Вы находитесь на странице: 1из 50

THIRD DIVISION

[G.R. No. 146006. February 23, 2004]



JOSE C. LEE AND ALMA AGGABAO, in their capacities as President and Corporate Secretary, respectively, of Philippines Internationl Life Insurance Company, and
FILIPINO LOAN ASSISTANCE GROUP,petitioners, vs. REGIONAL TRIAL COURT OF QUEZON CITY BRANCH 85 presided by JUDGE PEDRO M. AREOLA, BRANCH CLERK
OF COURT JANICE Y. ANTERO, DEPUTY SHERIFFS ADENAUER G. RIVERA and PEDRO L. BORJA, all of the Regional Trial Court of Quezon City Branch 85, MA. DIVINA
ENDERES claiming to be Special Administratrix, and other persons/ public officers acting for and in their behalf,respondents.
D E C I S I O N
CORONA, J .:
This is a petition for review under Rule 45 of the Rules of Court seeking to reverse and set aside the decision
[1]
of the Court of Appeals, First
Division, dated July 26, 2000, in CA G.R. 59736, which dismissed the petition for certiorari filed by petitioners Jose C. Lee and Alma Aggabao (in their
capacities as president and secretary, respectively, of Philippine International Life Insurance Company) and Filipino Loan Assistance Group.
The antecedent facts follow.
Dr. Juvencio P. Ortaez incorporated the Philippine International Life Insurance Company, Inc. on July 6, 1956. At the time of the companys
incorporation, Dr. Ortaez owned ninety percent (90%) of the subscribed capital stock.
On July 21, 1980, Dr. Ortaez died. He left behind a wife (Juliana Salgado Ortaez), three legitimate children (Rafael, Jose and Antonio Ortaez)
and five illegitimate children by Ligaya Novicio (herein private respondent Ma. Divina Ortaez-Enderes and her siblings Jose, Romeo, Enrico Manuel
and Cesar, all surnamed Ortaez).
[2]

On September 24, 1980, Rafael Ortaez filed before the Court of First Instance of Rizal, Quezon City Branch (now Regional Trial Court of Quezon
City) a petition for letters of administration of the intestate estate of Dr. Ortaez, docketed as SP Proc. Q-30884 (which petition to date remains pending
at Branch 85 thereof).
Private respondent Ma. Divina Ortaez-Enderes and her siblings filed an opposition to the petition for letters of administration and, in a
subsequent urgent motion, prayed that the intestate court appoint a special administrator.
On March 10, 1982, Judge Ernani Cruz Pao, then presiding judge of Branch 85, appointed Rafael and Jose Ortaez joint special administrators
of their fathers estate. Hearings continued for the appointment of a regular administrator (up to now no regular administrator has been appointed).
As ordered by the intestate court, special administrators Rafael and Jose Ortaez submitted an inventory of the estate of their father which
included, among other properties, 2,029
[3]
shares of stock in Philippine International Life Insurance Company (hereafter Philinterlife), representing
50.725% of the companys outstanding capital stock.
On April 15, 1989, the decedents wife, Juliana S. Ortaez, claiming that she owned 1,014
[4]
Philinterlife shares of stock as her conjugal share in
the estate, sold said shares with right to repurchase in favor of herein petitioner Filipino Loan Assistance Group (FLAG), represented by its president,
herein petitioner Jose C. Lee. Juliana Ortaez failed to repurchase the shares of stock within the stipulated period, thus ownership thereof was
consolidated by petitioner FLAG in its name.
On October 30, 1991, Special Administrator Jose Ortaez, acting in his personal capacity and claiming that he owned the remaining
1,011
[5]
Philinterlife shares of stocks as his inheritance share in the estate, sold said shares with right to repurchase also in favor of herein petitioner
FLAG, represented by its president, herein petitioner Jose C. Lee. After one year, petitioner FLAG consolidated in its name the ownership of the
Philinterlife shares of stock when Jose Ortaez failed to repurchase the same.
It appears that several years before (but already during the pendency of the intestate proceedings at the Regional Trial Court of Quezon City,
Branch 85), Juliana Ortaez and her two children, Special Administrators Rafael and Jose Ortaez, entered into a memorandum of agreement dated
March 4, 1982 for the extrajudicial settlement of the estate of Dr. Juvencio Ortaez, partitioning the estate (including the Philinterlife shares of stock)
among themselves. This was the basis of the number of shares separately sold by Juliana Ortaez on April 15, 1989 (1,014 shares) and by Jose
Ortaez on October 30, 1991 (1,011 shares) in favor of herein petitioner FLAG.
On July 12, 1995, herein private respondent Ma. Divina OrtaezEnderes and her siblings (hereafter referred to as private respondents
Enderes et al.) filed a motion for appointment of special administrator of Philinterlife shares of stock. This move was opposed by Special Administrator
Jose Ortaez.
On November 8, 1995, the intestate court granted the motion of private respondents Enderes et al. and appointed private respondent Enderes
special administratrix of the Philinterlife shares of stock.
On December 20, 1995, Special Administratrix Enderes filed an urgent motion to declare void ab initio the memorandum of agreement dated
March 4, 1982. On January 9, 1996, she filed a motion to declare the partial nullity of the extrajudicial settlement of the decedents estate. These
motions were opposed by Special Administrator Jose Ortaez.
On March 22, 1996, Special Administratrix Enderes filed an urgent motion to declare void ab initio the deeds of sale of Philinterlife shares of stock,
which move was again opposed by Special Administrator Jose Ortaez.
On February 4, 1997, Jose Ortaez filed an omnibus motion for (1) the approval of the deeds of sale of the Philinterlife shares of stock and (2) the
release of Ma. Divina Ortaez-Enderes as special administratrix of the Philinterlife shares of stock on the ground that there were no longer any shares of
stock for her to administer.
On August 11, 1997, the intestate court denied the omnibus motion of Special Administrator Jose Ortaez for the approval of the deeds of sale for
the reason that:
Under the Godoy case, supra, it was held in substance that a sale of a property of the estate without an Order of the probate court is void and passes no title to the
purchaser. Since the sales in question were entered into by Juliana S. Ortaez and Jose S. Ortaez in their personal capacity without prior approval of the Court, the
same is not binding upon the Estate.
WHEREFORE, the OMNIBUS MOTION for the approval of the sale of Philinterlife shares of stock and release of Ma. Divina Ortaez-Enderes as Special
Administratrix is hereby denied.
[6]

On August 29, 1997, the intestate court issued another order granting the motion of Special Administratrix Enderes for the annulment of the March
4, 1982 memorandum of agreement or extrajudicial partition of estate. The court reasoned that:
In consonance with the Order of this Court dated August 11, 1997 DENYING the approval of the sale of Philinterlife shares of stocks and release of Ma. Divina
Ortaez-Enderes as Special Administratrix, the Urgent Motion to Declare Void Ab Initio Memorandum of Agreement dated December 19, 1995. . . is hereby
impliedly partially resolved insofar as the transfer/waiver/renunciation of the Philinterlife shares of stock are concerned, in particular, No. 5, 9(c), 10(b) and 11(d)(ii) of
the Memorandum of Agreement.
WHEREFORE, this Court hereby declares the Memorandum of Agreement dated March 4, 1982 executed by Juliana S. Ortaez, Rafael S. Ortaez and Jose S. Ortaez
as partially void ab initio insofar as the transfer/waiver/renunciation of the Philinterlife shares of stocks are concerned.
[7]

Aggrieved by the above-stated orders of the intestate court, Jose Ortaez filed, on December 22, 1997, a petition for certiorari in the Court of
Appeals. The appellate court denied his petition, however, ruling that there was no legal justification whatsoever for the extrajudicial partition of the
estate by Jose Ortaez, his brother Rafael Ortaez and mother Juliana Ortaez during the pendency of the settlement of the estate of Dr. Ortaez,
without the requisite approval of the intestate court, when it was clear that there were other heirs to the estate who stood to be prejudiced thereby.
Consequently, the sale made by Jose Ortaez and his mother Juliana Ortaez to FLAG of the shares of stock they invalidly appropriated for themselves,
without approval of the intestate court, was void.
[8]

Special Administrator Jose Ortaez filed a motion for reconsideration of the Court of Appeals decision but it was denied. He elevated the case to
the Supreme Court via petition for review under Rule 45 which the Supreme Court dismissed on October 5, 1998, on a technicality. His motion for
reconsideration was denied with finality on January 13, 1999. On February 23, 1999, the resolution of the Supreme Court dismi ssing the petition of
Special Administrator Jose Ortaez became final and was subsequently recorded in the book of entries of judgments.
Meanwhile, herein petitioners Jose Lee and Alma Aggabao, with the rest of the FLAG-controlled board of directors, increased the authorized
capital stock of Philinterlife, diluting in the process the 50.725% controlling interest of the decedent, Dr. Juvencio Ortaez, in the insurance
company.
[9]
This became the subject of a separate action at the Securities and Exchange Commission filed by private respondent-Special Administratrix
Enderes against petitioner Jose Lee and other members of the FLAG-controlled board of Philinterlife on November 7, 1994. Thereafter, various cases
were filed by Jose Lee as president of Philinterlife and Juliana Ortaez and her sons against private respondent-Special Administratrix Enderes in the
SEC and civil courts.
[10]
Somehow, all these cases were connected to the core dispute on the legality of the sale of decedent Dr. Ortaezs Philinterli fe
shares of stock to petitioner FLAG, represented by its president, herein petitioner Jose Lee who later became the president of Philinterlife after the
controversial sale.
On May 2, 2000, private respondent-Special Administratrix Enderes and her siblings filed a motion for execution of the Orders of the intestate
court dated August 11 and August 29, 1997 because the orders of the intestate court nullifying the sale (upheld by the Court of Appeals and the
Supreme Court) had long became final. Respondent-Special Administratrix Enderes served a copy of the motion to petitioners Jose Lee and Alma
Aggabao as president and secretary, respectively, of Philinterlife,
[11]
but petitioners ignored the same.
On July 6, 2000, the intestate court granted the motion for execution, the dispositive portion of which read:
WHEREFORE, premises considered, let a writ of execution issue as follows:
1. Confirming the nullity of the sale of the 2,029 Philinterlife shares in the name of the Estate of Dr. Juvencio Ortaez to Filipino
Loan Assistance Group (FLAG);
2. Commanding the President and the Corporate Secretary of Philinterlife to reinstate in the stock and transfer book of Philinterlife
the 2,029 Philinterlife shares of stock in the name of the Estate of Dr. Juvencio P. Ortaez as the owner thereof without
prejudice to other claims for violation of pre-emptive rights pertaining to the said 2,029 Philinterlife shares;
3. Directing the President and the Corporate Secretary of Philinterlife to issue stock certificates of Philinterlife for 2,029 shares in
the name of the Estate of Dr. Juvencio P. Ortaez as the owner thereof without prejudice to other claims for violations of pre-
emptive rights pertaining to the said 2,029 Philinterlife shares and,
4. Confirming that only the Special Administratrix, Ma. Divina Ortaez-Enderes, has the power to exercise all the rights appurtenant
to the said shares, including the right to vote and to receive dividends.
5. Directing Philinterlife and/or any other person or persons claiming to represent it or otherwise, to acknowledge and allow the said
Special Administratrix to exercise all the aforesaid rights on the said shares and to refrain from resorting to any action which
may tend directly or indirectly to impede, obstruct or bar the free exercise thereof under pain of contempt.
6. The President, Corporate Secretary, any responsible officer/s of Philinterlife, or any other person or persons claiming to
represent it or otherwise, are hereby directed to comply with this order within three (3) days from receipt hereof under pain of
contempt.
7. The Deputy Sheriffs Adenauer Rivera and Pedro Borja are hereby directed to implement the writ of execution with dispatch to
forestall any and/or further damage to the Estate.
SO ORDERED.
[12]

In the several occasions that the sheriff went to the office of petitioners to execute the writ of execution, he was barred by the security guard upon
petitioners instructions. Thus, private respondent-Special Administratrix Enderes filed a motion to cite herein petitioners Jose Lee and Alma Aggabao
(president and secretary, respectively, of Philinterlife) in contempt.
[13]

Petitioners Lee and Aggabao subsequently filed before the Court of Appeals a petition for certiorari, docketed as CA G.R. SP No. 59736.
Petitioners alleged that the intestate court gravely abused its discretion in (1) declaring that the ownership of FLAG over the Philinterlife shares of stock
was null and void; (2) ordering the execution of its order declaring such nullity and (3) depriving the petitioners of their right to due process.
On July 26, 2000, the Court of Appeals dismissed the petition outright:
We are constrained to DISMISS OUTRIGHT the present petition for certiorari and prohibition with prayer for a temporary restraining order and/or writ of preliminary
injunction in the light of the following considerations:
1. The assailed Order dated August 11, 1997 of the respondent judge had long become final and executory;
2. The certification on non-forum shopping is signed by only one (1) of the three (3) petitioners in violation of the Rules; and
3. Except for the assailed orders and writ of execution, deed of sale with right to repurchase, deed of sale of shares of stocks and
omnibus motion, the petition is not accompanied by such pleadings, documents and other material portions of the record as
would support the allegations therein in violation of the second paragraph, Rule 65 of the 1997 Rules of Civil Procedure, as
amended.
Petition is DISMISSED.
SO ORDERED.
[14]

The motion for reconsideration filed by petitioners Lee and Aggabao of the above decision was denied by the Court of Appeals on October 30,
2000:
This resolves the urgent motion for reconsideration filed by the petitioners of our resolution of July 26, 2000 dismissing outrightly the above-entitled petition for the
reason, among others, that the assailed Order dated August 11, 1997 of the respondent Judge had long become final and executory.
Dura lex, sed lex.
WHEREFORE, the urgent motion for reconsideration is hereby DENIED, for lack of merit.
SO ORDERED.
[15]

On December 4, 2000, petitioners elevated the case to the Supreme Court through a petition for review under Rule 45 but on December 13, 2000,
we denied the petition because there was no showing that the Court of Appeals in CA G.R. SP No. 59736 committed any reversible error to warrant the
exercise by the Supreme Court of its discretionary appellate jurisdiction.
[16]

However, upon motion for reconsideration filed by petitioners Lee and Aggabao, the Supreme Court granted the motion and reinstated their
petition on September 5, 2001. The parties were then required to submit their respective memoranda.
Meanwhile, private respondent-Special Administratrix Enderes, on July 19, 2000, filed a motion to direct the branch clerk of court in lieu of herein
petitioners Lee and Aggabao to reinstate the name of Dr. Ortaez in the stock and transfer book of Philinterlife and issue the corresponding stock
certificate pursuant to Section 10, Rule 39 of the Rules of Court which provides that the court may direct the act to be done at the cost of the
disobedient party by some other person appointed by the court and the act when so done shall have the effect as if done by the party. Petitioners Lee
and Aggabao opposed the motion on the ground that the intestate court should refrain from acting on the motion because the issues raised therein were
directly related to the issues raised by them in their petition for certiorari at the Court of Appeals docketed as CA-G.R. SP No. 59736. On October 30,
2000, the intestate court granted the motion, ruling that there was no prohibition for the intestate court to execute its orders inasmuch as the appellate
court did not issue any TRO or writ of preliminary injunction.
On December 3, 2000, petitioners Lee and Aggabao filed a petition for certiorari in the Court of Appeals, docketed as CA-G.R. SP No. 62461,
questioning this time the October 30, 2000 order of the intestate court directing the branch clerk of court to issue the stock certificates. They also
questioned in the Court of Appeals the order of the intestate court nullifying the sale made in their favor by Juliana Ortaez and Jose Ortaez. On
November 20, 2002, the Court of Appeals denied their petition and upheld the power of the intestate court to execute its order. Petitioners Lee and
Aggabao then filed motion for reconsideration which at present is still pending resolution by the Court of Appeals.
Petitioners Jose Lee and Alma Aggabao (president and secretary, respectively, of Philinterlife) and FLAG now raise the following errors for our
consideration:
THE COURT OF APPEALS COMMITTED GRAVE REVERSIBLE ERROR:
A. IN FAILING TO RECONSIDER ITS PREVIOUS RESOLUTION DENYING THE PETITION DESPITE THE FACT THAT THE
APPELLATE COURTS MISTAKE IN APPREHENDING THE FACTS HAD BECOME PATENT AND EVIDENT FROM THE MOTION
FOR RECONSIDERATION AND THE COMMENT OF RESPONDENT ENDERES WHICH HAD ADMITTED THE FACTUAL
ALLEGATIONS OF PETITIONERS IN THE PETITION AS WELL AS IN THE MOTION FOR RECONSIDERATION. MOREOVER,
THE RESOLUTION OF THE APPELLATE COURT DENYING THE MOTION FOR RECONSIDERATION WAS CONTAINED IN
ONLY ONE PAGE WITHOUT EVEN TOUCHING ON THE SUBSTANTIVE MERITS OF THE EXHAUSTIVE DISCUSSION OF
FACTS AND SUPPORTING LAW IN THE MOTION FOR RECONSIDERATION IN VIOLATION OF THE RULE ON
ADMINISTRATIVE DUE PROCESS;
B. IN FAILING TO SET ASIDE THE VOID ORDERS OF THE INTESTATE COURT ON THE ERRONEOUS GROUND THAT THE
ORDERS WERE FINAL AND EXECUTORY WITH REGARD TO PETITIONERS EVEN AS THE LATTER WERE NEVER NOTIFIED
OF THE PROCEEDINGS OR ORDER CANCELING ITS OWNERSHIP;
C. IN NOT FINDING THAT THE INTESTATE COURT COMMITTED GRAVE ABUSE OF DISCRETION AMOUNTING TO EXCESS OF
JURISDICTION (1) WHEN IT ISSUED THE OMNIBUS ORDER NULLIFYING THE OWNERSHIP OF PETITIONER FLAG OVER
SHARES OF STOCK WHICH WERE ALLEGED TO BE PART OF THE ESTATE AND (2) WHEN IT ISSUED A VOID WRIT OF
EXECUTION AGAINST PETITIONER FLAG AS PRESENT OWNER TO IMPLEMENT MERELY PROVISIONAL ORDERS,
THEREBY VIOLATING FLAGS CONSTITUTIONAL RIGHT AGAINST DEPRIVATION OF PROPERTY WITHOUT DUE PROCESS;
D. IN FAILING TO DECLARE NULL AND VOID THE ORDERS OF THE INTESTATE COURT WHICH NULLIFIED THE SALE OF
SHARES OF STOCK BETWEEN THE LEGITIMATE HEIR JOSE S. ORTAEZ AND PETITIONER FLAG BECAUSE OF SETTLED
LAW AND JURISPRUDENCE, I.E., THAT AN HEIR HAS THE RIGHT TO DISPOSE OF THE DECEDENTS PROPERTY EVEN IF
THE SAME IS UNDER ADMINISTRATION PURSUANT TO CIVIL CODE PROVISION THAT POSSESSION OF HEREDITARY
PROPERTY IS TRANSMITTED TO THE HEIR THE MOMENT OF DEATH OF THE DECEDENT (ACEDEBO VS. ABESAMIS, 217
SCRA 194);
E. IN DISREGARDING THE FINAL DECISION OF THE SUPREME COURT IN G.R. NO. 128525 DATED DECEMBER 17, 1999
INVOLVING SUBSTANTIALLY THE SAME PARTIES, TO WIT, PETITIONERS JOSE C. LEE AND ALMA AGGABAO WERE
RESPONDENTS IN THAT CASE WHILE RESPONDENT MA. DIVINA ENDERES WAS THE PETITIONER THEREIN. THAT
DECISION, WHICH CAN BE CONSIDERED LAW OF THE CASE, RULED THAT PETITIONERS CANNOT BE ENJOINED BY
RESPONDENT ENDERES FROM EXERCISING THEIR POWER AS DIRECTORS AND OFFICERS OF PHILINTERLIFE AND THAT
THE INTESTATE COURT IN CHARGE OF THE INTESTATE PROCEEDINGS CANNOT ADJUDICATE TITLE TO PROPERTIES
CLAIMED TO BE PART OF THE ESTATE AND WHICH ARE EQUALLY CLAIMED BY PETITIONER FLAG.
[17]

The petition has no merit.
Petitioners Jose Lee and Alma Aggabao, representing Philinterlife and FLAG, assail before us not only the validity of the wri t of execution issued
by the intestate court dated July 7, 2000 but also the validity of the August 11, 1997 order of the intestate court nullifying the sale of the 2,029 Philinterlife
shares of stock made by Juliana Ortaez and Jose Ortaez, in their personal capacities and without court approval, in favor of petitioner FLAG.
We cannot allow petitioners to reopen the issue of nullity of the sale of the Philinterlife shares of stock in their favor because this was already
settled a long time ago by the Court of Appeals in its decision dated June 23, 1998 in CA-G.R. SP No. 46342. This decision was effectively upheld by us
in our resolution dated October 9, 1998 in G.R. No. 135177 dismissing the petition for review on a technicality and thereafter denying the motion for
reconsideration on January 13, 1999 on the ground that there was no compelling reason to reconsider said denial.
[18]
Our decision became final on
February 23, 1999 and was accordingly entered in the book of entry of judgments. For all intents and purposes therefore, the nullity of the sale of the
Philinterlife shares of stock made by Juliana Ortaez and Jose Ortaez in favor of petitioner FLAG is already a closed case. To reopen said issue would
set a bad precedent, opening the door wide open for dissatisfied parties to relitigate unfavorable decisions no end. This is completely inimical to the
orderly and efficient administration of justice.
The said decision of the Court of Appeals in CA-G.R. SP No. 46342 affirming the nullity of the sale made by Jose Ortaez and his mother Juliana
Ortaez of the Philinterlife shares of stock read:
Petitioners asseverations relative to said [memorandum] agreement were scuttled during the hearing before this Court thus:
JUSTICE AQUINO:
Counsel for petitioner, when the Memorandum of Agreement was executed, did the children of Juliana Salgado know already
that there was a claim for share in the inheritance of the children of Novicio?
ATTY. CALIMAG:
Your Honor please, at that time, Your Honor, it is already known to them.
JUSTICE AQUINO:
What can be your legal justification for extrajudicial settlement of a property subject of intestate proceedings when there i s an
adverse claim of another set of heirs, alleged heirs? What would be the legal justification for extra-judicially settling a property under
administration without the approval of the intestate court?
ATTY. CALIMAG:
Well, Your Honor please, in that extra-judicial settlement there is an approval of the honorable court as to the propertys
partition x x x. There were as mentioned by the respondents counsel, Your Honor.
ATTY. BUYCO:
No
JUSTICE AQUINO:
The point is, there can be no adjudication of a property under intestate proceedings without the approval of the court. That is
basic unless you can present justification on that. In fact, there are two steps: first, you ask leave and then execute the document and
then ask for approval of the document executed. Now, is there any legal justification to exclude this particular transaction from those
steps?
ATTY. CALIMAG:
None, Your Honor.
ATTY BUYCO:
With that admission that there is no legal justification, Your Honor, we rest the case for the private respondent. How can the
lower court be accused of abusing its discretion? (pages 33-35, TSN of January 29, 1998).
Thus, We find merit in the following postulation by private respondent:
What we have here is a situation where some of the heirs of the decedent without securing court approval have appropriated as their own personal property the
properties of [the] Estate, to the exclusion and the extreme prejudice of the other claimant/heirs. In other words, these heirs, without court approval, have distributed the
asset of the estate among themselves and proceeded to dispose the same to third parties even in the absence of an order of distribution by the Estate Court. As admitted
by petitioners counsel, there was absolutely no legal justification for this action by the heirs. There being no legal justification, petitioner has no basis for demanding
that public respondent [the intestate court] approve the sale of the Philinterlife shares of the Estate by Juliana and Jose Ortaez in favor of the Filipino Loan Assistance
Group.
It is an undisputed fact that the parties to the Memorandum of Agreement dated March 4, 1982 (see Annex 7 of the Comment). . . are not the only heirs claiming an
interest in the estate left by Dr. Juvencio P. Ortaez. The records of this case. . . clearly show that as early as March 3, 1981 an Opposition to the Application for
Issuance of Letters of Administration was filed by the acknowledged natural children of Dr. Juvencio P. Ortaez with Ligaya Novicio. . . This claim by the
acknowledged natural children of Dr. Juvencio P. Ortaez is admittedly known to the parties to the Memorandum of Agreement before they executed the same. This
much was admitted by petitioners counsel during the oral argument. xxx
Given the foregoing facts, and the applicable jurisprudence, public respondent can never be faulted for not approving. . . the subsequent sale by the petitioner [Jose
Ortaez] and his mother [Juliana Ortaez] of the Philinterlife shares belonging to the Estate of Dr. Juvencio P. Ortaez. (pages 3-4 of Private Respondents
Memorandum; pages 243-244 of the Rollo)
Amidst the foregoing, We found no grave abuse of discretion amounting to excess or want of jurisdiction committed by respondent judge.
[19]

From the above decision, it is clear that Juliana Ortaez, and her three sons, Jose, Rafael and Antonio, all surnamed Ortaez, invalidly entered
into a memorandum of agreement extrajudicially partitioning the intestate estate among themselves, despite their knowledge that there were other heirs
or claimants to the estate and before final settlement of the estate by the intestate court. Since the appropriation of the estate properties by Juliana
Ortaez and her children (Jose, Rafael and Antonio Ortaez) was invalid, the subsequent sale thereof by Juliana and Jose to a third party (FLAG),
without court approval, was likewise void.
An heir can sell his right, interest, or participation in the property under administration under Art. 533 of the Civil Code which provides that
possession of hereditary property is deemed transmitted to the heir without interruption from the moment of death of the decedent.
[20]
However, an heir
can only alienate such portion of the estate that may be allotted to him in the division of the estate by the probate or intestate court after final
adjudication, that is, after all debtors shall have been paid or the devisees or legatees shall have been given their shares.
[21]
This means that an heir may
only sell his ideal or undivided share in the estate, not any specific property therein. In the present case, Juliana Ortaez and Jose Ortaez sold specific
properties of the estate (1,014 and 1,011 shares of stock in Philinterlife) in favor of petitioner FLAG. This they could not lawfully do pending the final
adjudication of the estate by the intestate court because of the undue prejudice it would cause the other claimants to the estate, as what happened in the
present case.
Juliana Ortaez and Jose Ortaez sold specific properties of the estate, without court approval. It is well-settled that court approval is necessary
for the validity of any disposition of the decedents estate. In the early case of Godoy vs. Orellano,
[22]
we laid down the rule that the sale of the property of
the estate by an administrator without the order of the probate court is void and passes no title to the purchaser. And in the case of Dillena vs. Court of
Appeals,
[23]
we ruled that:
[I]t must be emphasized that the questioned properties (fishpond) were included in the inventory of properties of the estate submitted by then Administratrix Fausta
Carreon Herrera on November 14, 1974. Private respondent was appointed as administratrix of the estate on March 3, 1976 in lieu of Fausta Carreon Herrera. On
November 1, 1978, the questioned deed of sale of the fishponds was executed between petitioner and private respondent without notice and approval of the probate
court. Even after the sale, administratrix Aurora Carreon still included the three fishponds as among the real properties of the estate in her inventory submitted on
August 13, 1981. In fact, as stated by the Court of Appeals, petitioner, at the time of the sale of the fishponds in question, knew that the same were part of the estate
under administration.
x x x x x x x x x
The subject properties therefore are under the jurisdiction of the probate court which according to our settled jurisprudence has the authority to approve any disposition
regarding properties under administration. . . More emphatic is the declaration We made in Estate of Olave vs. Reyes (123 SCRA 767) where We stated that when the
estate of the deceased person is already the subject of a testate or intestate proceeding, the administrator cannot enter into any transaction involving it without prior
approval of the probate court.
Only recently, in Manotok Realty, Inc. vs. Court of Appeals (149 SCRA 174), We held that the sale of an immovable property belonging to the estate of a decedent, in a
special proceedings, needs court approval. . . This pronouncement finds support in the previous case of Dolores Vda. De Gil vs. Agustin Cancio (14 SCRA 797)
wherein We emphasized that it is within the jurisdiction of a probate court to approve the sale of properties of a deceased person by his prospective heirs before final
adjudication. x x x
It being settled that property under administration needs the approval of the probate court before it can be disposed of, any unauthorized disposition does not bind the
estate and is null and void. As early as 1921 in the case of Godoy vs. Orellano (42 Phil 347), We laid down the rule that a sale by an administrator of property of the
deceased, which is not authorized by the probate court is null and void and title does not pass to the purchaser.
There is hardly any doubt that the probate court can declare null and void the disposition of the property under administration, made by private respondent, the same
having been effected without authority from said court. It is the probate court that has the power to authorize and/or approve the sale (Section 4 and 7, Rule 89), hence,
a fortiori, it is said court that can declare it null and void for as long as the proceedings had not been closed or terminated. To uphold petitioners contention that the
probate court cannot annul the unauthorized sale, would render meaningless the power pertaining to the said court. (Bonga vs. Soler, 2 SCRA 755). (emphasis ours)
Our jurisprudence is therefore clear that (1) any disposition of estate property by an administrator or prospective heir pending final adjudication
requires court approval and (2) any unauthorized disposition of estate property can be annulled by the probate court, there being no need for a separate
action to annul the unauthorized disposition.
The question now is: can the intestate or probate court execute its order nullifying the invalid sale?
We see no reason why it cannot. The intestate court has the power to execute its order with regard to the nullity of an unauthorized sale of estate
property, otherwise its power to annul the unauthorized or fraudulent disposition of estate property would be meaningless. In other words, enforcement is
a necessary adjunct of the intestate or probate courts power to annul unauthorized or fraudulent transactions to prevent the dissipation of estate
property before final adjudication.
Moreover, in this case, the order of the intestate court nullifying the sale was affirmed by the appellate courts (the Court of Appeals in CA-G.R. SP
No. 46342 dated June 23, 1998 and subsequently by the Supreme Court in G.R. No. 135177 dated October 9, 1998). The finality of the decision of the
Supreme Court was entered in the book of entry of judgments on February 23, 1999. Considering the finality of the order of the intestate court nullifying
the sale, as affirmed by the appellate courts, it was correct for private respondent-Special Administratrix Enderes to thereafter move for a writ of
execution and for the intestate court to grant it.
Petitioners Jose Lee, Alma Aggabao and FLAG, however, contend that the probate court could not issue a writ of execution with regard to its order
nullifying the sale because said order was merely provisional:
The only authority given by law is for respondent judge to determine provisionally whether said shares are included or excluded in the inventory In ordering the
execution of the orders, respondent judge acted in excess of his jurisdiction and grossly violated settled law and jurisprudence, i.e., that the determination by a probate
or intestate court of whether a property is included or excluded in the inventory of the estate being provisional in nature, cannot be the subject of
execution.
[24]
(emphasis ours)
Petitioners argument is misplaced. There is no question, based on the facts of this case, that the Philinterlife shares of stock were part of the
estate of Dr. Juvencio Ortaez from the very start as in fact these shares were included in the inventory of the properties of the estate submitted by
Rafael Ortaez after he and his brother, Jose Ortaez, were appointed special administrators by the intestate court.
[25]

The controversy here actually started when, during the pendency of the settlement of the estate of Dr. Ortaez, his wife Juliana Ortaez sold the
1,014 Philinterlife shares of stock in favor petitioner FLAG without the approval of the intestate court. Her son Jose Ortaez later sold the remaining
1,011 Philinterlife shares also in favor of FLAG without the approval of the intestate court.
We are not dealing here with the issue of inclusion or exclusion of properties in the inventory of the estate because there is no question that, from
the very start, the Philinterlife shares of stock were owned by the decedent, Dr. Juvencio Ortaez. Rather, we are concerned here with the effect of
the sale made by the decedents heirs, Juliana Ortaez and Jose Ortaez, without the required approval of the intestate court. This being so,
the contention of petitioners that the determination of the intestate court was merely provisional and should have been threshed out in a separate
proceeding is incorrect.
The petitioners Jose Lee and Alma Aggabao next contend that the writ of execution should not be executed against them because they were not
notified, nor they were aware, of the proceedings nullifying the sale of the shares of stock.
We are not persuaded. The title of the purchaser like herein petitioner FLAG can be struck down by the intestate court after a clear showing of the
nullity of the alienation. This is the logical consequence of our ruling in Godoy and in several subsequent cases.
[26]
The sale of any property of the
estate by an administrator or prospective heir without order of the probate or intestate court is void and passes no title to the
purchaser. Thus, in Juan Lao et al. vs. Hon. Melencio Geneto, G.R. No. 56451, June 19, 1985, we ordered the probate court to cancel the transfer
certificate of title issued to the vendees at the instance of the administrator after finding that the sale of real property under probate proceedings was
made without the prior approval of the court. The dispositive portion of our decision read:
IN VIEW OF THE FOREGOING CONSIDERATIONS, the assailed Order dated February 18, 1981 of the respondent Judge approving the questioned Amicable
Settlement is declared NULL and VOID and hereby SET ASIDE. Consequently, the sale in favor of Sotero Dioniosio III and by the latter to William Go is likewise
declared NULL and VOID. The Transfer Certificate of Title issued to the latter is hereby ordered cancelled.
It goes without saying that the increase in Philinterlifes authorized capital stock, approved on the vote of petitioners non-existent shareholdings
and obviously calculated to make it difficult for Dr. Ortaezs estate to reassume its controlling interest in Philinterlife, was likewise void ab initio.
Petitioners next argue that they were denied due process.
We do not think so.
The facts show that petitioners, for reasons known only to them, did not appeal the decision of the intestate court nullifying the sale of shares of
stock in their favor. Only the vendor, Jose Ortaez, appealed the case. A careful review of the records shows that petitioners had actual knowledge of
the estate settlement proceedings and that they knew private respondent Enderes was questioning therein the sale to them of the Philinterlife shares of
stock.
It must be noted that private respondent-Special Administratrix Enderes filed before the intestate court (RTC of Quezon City, Branch 85) a Motion
to Declare Void Ab Initio Deeds of Sale of Philinterlife Shares of Stock on March 22, 1996. But as early as 1994, petitioners already knew of the pending
settlement proceedings and that the shares they bought were under the administration by the intestate court because private respondent Ma. Divina
Ortaez-Enderes and her mother Ligaya Novicio had filed a case against them at the Securities and Exchange Commission on November 7, 1994,
docketed as SEC No. 11-94-4909, for annulment of transfer of shares of stock, annulment of sale of corporate properties, annulment of subscriptions on
increased capital stocks, accounting, inspection of corporate books and records and damages with prayer for a writ of preliminary injunction and/or
temporary restraining order.
[27]
In said case, Enderes and her mother questioned the sale of the aforesaid shares of stock to petitioners. The SEC
hearing officer in fact, in his resolution dated March 24, 1995, deferred to the jurisdiction of the intestate court to rule on the validity of the sale of shares
of stock sold to petitioners by Jose Ortaez and Juliana Ortaez:
Petitioners also averred that. . . the Philinterlife shares of Dr. Juvencio Ortaez who died, in 1980, are part of his estate which is presently the subject matter of an
intestate proceeding of the RTC of Quezon City, Branch 85. Although, private respondents [Jose Lee et al.] presented the documents of partition whereby the foregoing
share of stocks were allegedly partitioned and conveyed to Jose S. Ortaez who allegedly assigned the same to the other private respondents, approval of the Court was
not presented. Thus, the assignments to the private respondents [Jose Lee et al.] of the subject shares of stocks are void.
x x x x x x x x x
With respect to the alleged extrajudicial partition of the shares of stock owned by the late Dr. Juvencio Ortaez, we rule that the matter properly belongs to the
jurisdiction of the regular court where the intestate proceedings are currently pending.
[28]

With this resolution of the SEC hearing officer dated as early as March 24, 1995 recognizing the jurisdiction of the intestate court to determine the
validity of the extrajudicial partition of the estate of Dr. Ortaez and the subsequent sale by the heirs of the decedent of the Philinterlife shares of stock to
petitioners, how can petitioners claim that they were not aware of the intestate proceedings?
Futhermore, when the resolution of the SEC hearing officer reached the Supreme Court in 1996 (docketed as G.R. 128525), herei n petitioners
who were respondents therein filed their answer which contained statements showing that they knew of the pending intestate proceedings:
[T]he subject matter of the complaint is not within the jurisdiction of the SEC but with the Regional Trial Court; Ligaya Novicio and children represented themselves to
be the common law wife and illegitimate children of the late Ortaez; that on March 4, 1982, the surviving spouse Juliana Ortaez, on her behalf and for her minor son
Antonio, executed a Memorandum of Agreement with her other sons Rafael and Jose, both surnamed Ortaez, dividing the estate of the deceased composed of his one-
half (1/2) share in the conjugal properties; that in the said Memorandum of Agreement, Jose S. Ortaez acquired as his share of the estate the 1,329 shares of stock in
Philinterlife; that on March 4, 1982, Juliana and Rafael assigned their respective shares of stock in Philinterlife to Jose; that contrary to the contentions of petitioners,
private respondents Jose Lee, Carlos Lee, Benjamin Lee and Alma Aggabao became stockholders of Philinterlife on March 23, 1983 when Jose S. Ortaez, the
principal stockholder at that time, executed a deed of sale of his shares of stock to private respondents; and that the right of petitioners to question the Memorandum of
Agreement and the acquisition of shares of stock of private respondent is barred by prescription.
[29]

Also, private respondent-Special Administratrix Enderes offered additional proof of actual knowledge of the settlement proceedings by petitioners
which petitioners never denied: (1) that petitioners were represented by Atty. Ricardo Calimag previously hired by the mother of private respondent
Enderes to initiate cases against petitioners Jose Lee and Alma Aggaboa for the nullification of the sale of the shares of stock but said counsel made a
conflicting turn-around and appeared instead as counsel of petitioners, and (2) that the deeds of sale executed between petitioners and the heirs of the
decedent (vendors Juliana Ortaez and Jose Ortaez) were acknowledged before Atty. Ramon Carpio who, during the pendency of the settlement
proceedings, filed a motion for the approval of the sale of Philinterlife shares of stock to the Knights of Columbus Fraternal Association, Inc. (which
motion was, however, later abandoned).
[30]
All this sufficiently proves that petitioners, through their counsels, knew of the pending settlement
proceedings.
Finally, petitioners filed several criminal cases such as libel (Criminal Case No. 97-7179-81), grave coercion (Criminal Case No. 84624) and
robbery (Criminal Case No. Q-96-67919) against private respondents mother Ligaya Novicio who was a director of Philinterlife,
[31]
all of which criminal
cases were related to the questionable sale to petitioners of the Philinterlife shares of stock.
Considering these circumstances, we cannot accept petitioners claim of denial of due process. The essence of due process is the reasonable
opportunity to be heard. Where the opportunity to be heard has been accorded, there is no denial of due process.
[32]
In this case, petitioners knew of the
pending instestate proceedings for the settlement of Dr. Juvencio Ortaezs estate but for reasons they alone knew, they never intervened. When the
court declared the nullity of the sale, they did not bother to appeal. And when they were notified of the motion for execution of the Orders of the intestate
court, they ignored the same. Clearly, petitioners alone should bear the blame.
Petitioners next contend that we are bound by our ruling in G.R. No. 128525 entitled Ma. Divina Ortaez-Enderes vs. Court of Appeals,dated
December 17, 1999, where we allegedly ruled that the intestate court may not pass upon the title to a certain property for the purpose of determining
whether the same should or should not be included in the inventory but such determination is not conclusive and is subject to final decision in a separate
action regarding ownership which may be constituted by the parties.
We are not unaware of our decision in G.R. No. 128525. The issue therein was whether the Court of Appeals erred in affirming the resolution of
the SEC that Enderes et al. were not entitled to the issuance of the writ of preliminary injunction. We ruled that the Court of Appeals was correct in
affirming the resolution of the SEC denying the issuance of the writ of preliminary injunction because injunction is not desi gned to protect contingent
rights. Said case did not rule on the issue of the validity of the sale of shares of stock belonging to the decedents estate without court approval nor of
the validity of the writ of execution issued by the intestate court. G.R. No. 128525 clearly involved a different issue and it does not therefore apply to the
present case.
Petitioners and all parties claiming rights under them are hereby warned not to further delay the execution of the Orders of the intestate court
dated August 11 and August 29, 1997.
WHEREFORE, the petition is hereby DENIED. The decision of the Court of Appeals in CA-G.R. S.P. No. 59736 dated July 26, 2000, dismissing
petitioners petition for certiorari and affirming the July 6, 2000 order of the trial court which ordered the execution of its (trial courts) August 11 and 29,
1997 orders, is hereby AFFIRMED.
SO ORDERED.













Republic of the Philippines
SUPREME COURT
Manila

EN BANC
G.R. No. L-43082 June 18, 1937

PABLO LORENZO, as trustee of the estate of Thomas Hanley, deceased, plaintiff-appellant,
vs.
JUAN POSADAS, JR., Collector of Internal Revenue, defendant-appellant.

Pablo Lorenzo and Delfin Joven for plaintiff-appellant.
Office of the Solicitor-General Hilado for defendant-appellant.

LAUREL, J.:
On October 4, 1932, the plaintiff Pablo Lorenzo, in his capacity as trustee of the estate of Thomas Hanley, deceased, brought this action in the Court of First Instance
of Zamboanga against the defendant, Juan Posadas, Jr., then the Collector of Internal Revenue, for the refund of the amount of P2,052.74, paid by the plaintiff as
inheritance tax on the estate of the deceased, and for the collection of interst thereon at the rate of 6 per cent per annum, computed from September 15, 1932, the
date when the aforesaid tax was [paid under protest. The defendant set up a counterclaim for P1,191.27 alleged to be interest due on the tax in question and which
was not included in the original assessment. From the decision of the Court of First Instance of Zamboanga dismissing both the plaintiff's complaint and the
defendant's counterclaim, both parties appealed to this court.

It appears that on May 27, 1922, one Thomas Hanley died in Zamboanga, Zamboanga, leaving a will (Exhibit 5) and considerable amount of real and personal
properties. On june 14, 1922, proceedings for the probate of his will and the settlement and distribution of his estate were begun in the Court of First Instance of
Zamboanga. The will was admitted to probate. Said will provides, among other things, as follows:

4. I direct that any money left by me be given to my nephew Matthew Hanley.

5. I direct that all real estate owned by me at the time of my death be not sold or otherwise disposed of for a period of ten (10) years after my death, and that the
same be handled and managed by the executors, and proceeds thereof to be given to my nephew, Matthew Hanley, at Castlemore, Ballaghaderine, County of
Rosecommon, Ireland, and that he be directed that the same be used only for the education of my brother's children and their descendants.

6. I direct that ten (10) years after my death my property be given to the above mentioned Matthew Hanley to be disposed of in the way he thinks most
advantageous.

x x x x x x x x x

8. I state at this time I have one brother living, named Malachi Hanley, and that my nephew, Matthew Hanley, is a son of my said brother, Malachi Hanley.

The Court of First Instance of Zamboanga considered it proper for the best interests of ther estate to appoint a trustee to administer the real properties which, under
the will, were to pass to Matthew Hanley ten years after the two executors named in the will, was, on March 8, 1924, appointed trustee. Moore took his oath of
office and gave bond on March 10, 1924. He acted as trustee until February 29, 1932, when he resigned and the plaintiff herein was appointed in his stead.

During the incumbency of the plaintiff as trustee, the defendant Collector of Internal Revenue, alleging that the estate left by the deceased at the time of his death
consisted of realty valued at P27,920 and personalty valued at P1,465, and allowing a deduction of P480.81, assessed against the estate an inheritance tax in the
amount of P1,434.24 which, together with the penalties for deliquency in payment consisting of a 1 per cent monthly interest from July 1, 1931 to the date of
payment and a surcharge of 25 per cent on the tax, amounted to P2,052.74. On March 15, 1932, the defendant filed a motion in the testamentary proceedings
pending before the Court of First Instance of Zamboanga (Special proceedings No. 302) praying that the trustee, plaintiff herein, be ordered to pay to the Government
the said sum of P2,052.74. The motion was granted. On September 15, 1932, the plaintiff paid said amount under protest, notifying the defendant at the same time
that unless the amount was promptly refunded suit would be brought for its recovery. The defendant overruled the plaintiff's protest and refused to refund the said
amount hausted, plaintiff went to court with the result herein above indicated.

In his appeal, plaintiff contends that the lower court erred:

I. In holding that the real property of Thomas Hanley, deceased, passed to his instituted heir, Matthew Hanley, from the moment of the death of the former, and that
from the time, the latter became the owner thereof.

II. In holding, in effect, that there was deliquency in the payment of inheritance tax due on the estate of said deceased.

III. In holding that the inheritance tax in question be based upon the value of the estate upon the death of the testator, and not, as it should have been held, upon the
value thereof at the expiration of the period of ten years after which, according to the testator's will, the property could be and was to be delivered to the instituted
heir.

IV. In not allowing as lawful deductions, in the determination of the net amount of the estate subject to said tax, the amounts allowed by the court as compensation
to the "trustees" and paid to them from the decedent's estate.

V. In not rendering judgment in favor of the plaintiff and in denying his motion for new trial.

The defendant-appellant contradicts the theories of the plaintiff and assigns the following error besides:

The lower court erred in not ordering the plaintiff to pay to the defendant the sum of P1,191.27, representing part of the interest at the rate of 1 per cent per month
from April 10, 1924, to June 30, 1931, which the plaintiff had failed to pay on the inheritance tax assessed by the defendant against the estate of Thomas Hanley.

The following are the principal questions to be decided by this court in this appeal: (a) When does the inheritance tax accrue and when must it be satisfied? (b)
Should the inheritance tax be computed on the basis of the value of the estate at the time of the testator's death, or on its value ten years later? (c) In determining
the net value of the estate subject to tax, is it proper to deduct the compensation due to trustees? (d) What law governs the case at bar? Should the provisions of Act
No. 3606 favorable to the tax-payer be given retroactive effect? (e) Has there been deliquency in the payment of the inheritance tax? If so, should the additional
interest claimed by the defendant in his appeal be paid by the estate? Other points of incidental importance, raised by the parties in their briefs, will be touched upon
in the course of this opinion.

(a) The accrual of the inheritance tax is distinct from the obligation to pay the same. Section 1536 as amended, of the Administrative Code, imposes the tax upon
"every transmission by virtue of inheritance, devise, bequest, gift mortis causa, or advance in anticipation of inheritance,devise, or bequest." The tax therefore is
upon transmission or the transfer or devolution of property of a decedent, made effective by his death. (61 C. J., p. 1592.) It is in reality an excise or privilege tax
imposed on the right to succeed to, receive, or take property by or under a will or the intestacy law, or deed, grant, or gift to become operative at or after death.
Acording to article 657 of the Civil Code, "the rights to the succession of a person are transmitted from the moment of his death." "In other words", said Arellano, C.
J., ". . . the heirs succeed immediately to all of the property of the deceased ancestor. The property belongs to the heirs at the moment of the death of the ancestor
as completely as if the ancestor had executed and delivered to them a deed for the same before his death." (Bondad vs. Bondad, 34 Phil., 232. See also, Mijares vs.
Nery, 3 Phil., 195; Suilong & Co., vs. Chio-Taysan, 12 Phil., 13; Lubrico vs. Arbado, 12 Phil., 391; Innocencio vs. Gat-Pandan, 14 Phil., 491; Aliasas vs.Alcantara, 16 Phil.,
489; Ilustre vs. Alaras Frondosa, 17 Phil., 321; Malahacan vs. Ignacio, 19 Phil., 434; Bowa vs. Briones, 38 Phil., 27; Osario vs. Osario & Yuchausti Steamship Co., 41 Phil.,
531; Fule vs. Fule, 46 Phil., 317; Dais vs. Court of First Instance of Capiz, 51 Phil., 396; Baun vs. Heirs of Baun, 53 Phil., 654.) Plaintiff, however, asserts that while article
657 of the Civil Code is applicable to testate as well as intestate succession, it operates only in so far as forced heirs are concerned. But the language of article 657 of
the Civil Code is broad and makes no distinction between different classes of heirs. That article does not speak of forced heirs; it does not even use the word "heir". It
speaks of the rights of succession and the transmission thereof from the moment of death. The provision of section 625 of the Code of Civil Procedure regarding the
authentication and probate of a will as a necessary condition to effect transmission of property does not affect the general rule laid down in article 657 of the Civil
Code. The authentication of a will implies its due execution but once probated and allowed the transmission is effective as of the death of the testator in accordance
with article 657 of the Civil Code. Whatever may be the time when actual transmission of the inheritance takes place, succession takes place in any event at the
moment of the decedent's death. The time when the heirs legally succeed to the inheritance may differ from the time when the heirs actually receive such
inheritance. "Poco importa", says Manresa commenting on article 657 of the Civil Code, "que desde el falleimiento del causante, hasta que el heredero o legatario
entre en posesion de los bienes de la herencia o del legado, transcurra mucho o poco tiempo, pues la adquisicion ha de retrotraerse al momento de la muerte, y asi lo
ordena el articulo 989, que debe considerarse como complemento del presente." (5 Manresa, 305; see also, art. 440, par. 1, Civil Code.) Thomas Hanley having died
on May 27, 1922, the inheritance tax accrued as of the date.

From the fact, however, that Thomas Hanley died on May 27, 1922, it does not follow that the obligation to pay the tax arose as of the date. The time for the
payment on inheritance tax is clearly fixed by section 1544 of the Revised Administrative Code as amended by Act No. 3031, in relation to section 1543 of the same
Code. The two sections follow:

SEC. 1543. Exemption of certain acquisitions and transmissions. The following shall not be taxed:

(a) The merger of the usufruct in the owner of the naked title.

(b) The transmission or delivery of the inheritance or legacy by the fiduciary heir or legatee to the trustees.

(c) The transmission from the first heir, legatee, or donee in favor of another beneficiary, in accordance with the desire of the predecessor.

In the last two cases, if the scale of taxation appropriate to the new beneficiary is greater than that paid by the first, the former must pay the difference.

SEC. 1544. When tax to be paid. The tax fixed in this article shall be paid:

(a) In the second and third cases of the next preceding section, before entrance into possession of the property.

(b) In other cases, within the six months subsequent to the death of the predecessor; but if judicial testamentary or intestate proceedings shall be instituted prior to
the expiration of said period, the payment shall be made by the executor or administrator before delivering to each beneficiary his share.

If the tax is not paid within the time hereinbefore prescribed, interest at the rate of twelve per centum per annum shall be added as part of the tax; and to the tax
and interest due and unpaid within ten days after the date of notice and demand thereof by the collector, there shall be further added a surcharge of twenty-five per
centum.

A certified of all letters testamentary or of admisitration shall be furnished the Collector of Internal Revenue by the Clerk of Court within thirty days after their
issuance.

It should be observed in passing that the word "trustee", appearing in subsection (b) of section 1543, should read "fideicommissary" or "cestui que trust". There was
an obvious mistake in translation from the Spanish to the English version.

The instant case does fall under subsection (a), but under subsection (b), of section 1544 above-quoted, as there is here no fiduciary heirs, first heirs, legatee or
donee. Under the subsection, the tax should have been paid before the delivery of the properties in question to P. J. M. Moore as trustee on March 10, 1924.

(b) The plaintiff contends that the estate of Thomas Hanley, in so far as the real properties are concerned, did not and could not legally pass to the instituted heir,
Matthew Hanley, until after the expiration of ten years from the death of the testator on May 27, 1922 and, that the inheritance tax should be based on the value of
the estate in 1932, or ten years after the testator's death. The plaintiff introduced evidence tending to show that in 1932 the real properties in question had a
reasonable value of only P5,787. This amount added to the value of the personal property left by the deceased, which the plaintiff admits is P1,465, would generate
an inheritance tax which, excluding deductions, interest and surcharge, would amount only to about P169.52.

If death is the generating source from which the power of the estate to impose inheritance taxes takes its being and if, upon the death of the decedent, succession
takes place and the right of the estate to tax vests instantly, the tax should be measured by the vlaue of the estate as it stood at the time of the decedent's death,
regardless of any subsequent contingency value of any subsequent increase or decrease in value. (61 C. J., pp. 1692, 1693; 26 R. C. L., p. 232; Blakemore and Bancroft,
Inheritance Taxes, p. 137. See also Knowlton vs. Moore, 178 U.S., 41; 20 Sup. Ct. Rep., 747; 44 Law. ed., 969.) "The right of the state to an inheritance tax accrues at
the moment of death, and hence is ordinarily measured as to any beneficiary by the value at that time of such property as passes to him. Subsequent appreciation or
depriciation is immaterial." (Ross, Inheritance Taxation, p. 72.)

Our attention is directed to the statement of the rule in Cyclopedia of Law of and Procedure (vol. 37, pp. 1574, 1575) that, in the case of contingent remainders,
taxation is postponed until the estate vests in possession or the contingency is settled. This rule was formerly followed in New York and has been adopted in Illinois,
Minnesota, Massachusetts, Ohio, Pennsylvania and Wisconsin. This rule, horever, is by no means entirely satisfactory either to the estate or to those interested in the
property (26 R. C. L., p. 231.). Realizing, perhaps, the defects of its anterior system, we find upon examination of cases and authorities that New York has varied and
now requires the immediate appraisal of the postponed estate at its clear market value and the payment forthwith of the tax on its out of the corpus of the estate
transferred. (In re Vanderbilt, 172 N. Y., 69; 69 N. E., 782; In re Huber, 86 N. Y. App. Div., 458; 83 N. Y. Supp., 769; Estate of Tracy, 179 N. Y., 501; 72 N. Y., 519; Estate
of Brez, 172 N. Y., 609; 64 N. E., 958; Estate of Post, 85 App. Div., 611; 82 N. Y. Supp., 1079. Vide also, Saltoun vs. Lord Advocate, 1 Peter. Sc. App., 970; 3 Macq. H. L.,
659; 23 Eng. Rul. Cas., 888.) California adheres to this new rule (Stats. 1905, sec. 5, p. 343).

But whatever may be the rule in other jurisdictions, we hold that a transmission by inheritance is taxable at the time of the predecessor's death, notwithstanding the
postponement of the actual possession or enjoyment of the estate by the beneficiary, and the tax measured by the value of the property transmitted at that time
regardless of its appreciation or depreciation.

(c) Certain items are required by law to be deducted from the appraised gross in arriving at the net value of the estate on which the inheritance tax is to be computed
(sec. 1539, Revised Administrative Code). In the case at bar, the defendant and the trial court allowed a deduction of only P480.81. This sum represents the expenses
and disbursements of the executors until March 10, 1924, among which were their fees and the proven debts of the deceased. The plaintiff contends that the
compensation and fees of the trustees, which aggregate P1,187.28 (Exhibits C, AA, EE, PP, HH, JJ, LL, NN, OO), should also be deducted under section 1539 of the
Revised Administrative Code which provides, in part, as follows: "In order to determine the net sum which must bear the tax, when an inheritance is concerned, there
shall be deducted, in case of a resident, . . . the judicial expenses of the testamentary or intestate proceedings, . . . ."

A trustee, no doubt, is entitled to receive a fair compensation for his services (Barney vs. Saunders, 16 How., 535; 14 Law. ed., 1047). But from this it does not follow
that the compensation due him may lawfully be deducted in arriving at the net value of the estate subject to tax. There is no statute in the Philippines which requires
trustees' commissions to be deducted in determining the net value of the estate subject to inheritance tax (61 C. J., p. 1705). Furthermore, though a testamentary
trust has been created, it does not appear that the testator intended that the duties of his executors and trustees should be separated. (Ibid.; In re Vanneck's Estate,
161 N. Y. Supp., 893; 175 App. Div., 363; In re Collard's Estate, 161 N. Y. Supp., 455.) On the contrary, in paragraph 5 of his will, the testator expressed the desire that
his real estate be handled and managed by his executors until the expiration of the period of ten years therein provided. Judicial expenses are expenses of
administration (61 C. J., p. 1705) but, in State vs. Hennepin County Probate Court (112 N. W., 878; 101 Minn., 485), it was said: ". . . The compensation of a trustee,
earned, not in the administration of the estate, but in the management thereof for the benefit of the legatees or devises, does not come properly within the class or
reason for exempting administration expenses. . . . Service rendered in that behalf have no reference to closing the estate for the purpose of a distribution thereof to
those entitled to it, and are not required or essential to the perfection of the rights of the heirs or legatees. . . . Trusts . . . of the character of that here before the
court, are created for the the benefit of those to whom the property ultimately passes, are of voluntary creation, and intended for the preservation of the estate. No
sound reason is given to support the contention that such expenses should be taken into consideration in fixing the value of the estate for the purpose of this tax."

(d) The defendant levied and assessed the inheritance tax due from the estate of Thomas Hanley under the provisions of section 1544 of the Revised Administrative
Code, as amended by section 3 of Act No. 3606. But Act No. 3606 went into effect on January 1, 1930. It, therefore, was not the law in force when the testator died
on May 27, 1922. The law at the time was section 1544 above-mentioned, as amended by Act No. 3031, which took effect on March 9, 1922.

It is well-settled that inheritance taxation is governed by the statute in force at the time of the death of the decedent (26 R. C. L., p. 206; 4 Cooley on Taxation, 4th
ed., p. 3461). The taxpayer can not foresee and ought not to be required to guess the outcome of pending measures. Of course, a tax statute may be made
retroactive in its operation. Liability for taxes under retroactive legislation has been "one of the incidents of social life." (Seattle vs. Kelleher, 195 U. S., 360; 49 Law.
ed., 232 Sup. Ct. Rep., 44.) But legislative intent that a tax statute should operate retroactively should be perfectly clear. (Scwab vs. Doyle, 42 Sup. Ct. Rep., 491;
Smietanka vs. First Trust & Savings Bank, 257 U. S., 602; Stockdale vs. Insurance Co., 20 Wall., 323; Lunch vs. Turrish, 247 U. S., 221.) "A statute should be considered
as prospective in its operation, whether it enacts, amends, or repeals an inheritance tax, unless the language of the statute clearly demands or expresses that it shall
have a retroactive effect, . . . ." (61 C. J., P. 1602.) Though the last paragraph of section 5 of Regulations No. 65 of the Department of Finance makes section 3 of Act
No. 3606, amending section 1544 of the Revised Administrative Code, applicable to all estates the inheritance taxes due from which have not been paid, Act No. 3606
itself contains no provisions indicating legislative intent to give it retroactive effect. No such effect can begiven the statute by this court.

The defendant Collector of Internal Revenue maintains, however, that certain provisions of Act No. 3606 are more favorable to the taxpayer than those of Act No.
3031, that said provisions are penal in nature and, therefore, should operate retroactively in conformity with the provisions of article 22 of the Revised Penal Code.
This is the reason why he applied Act No. 3606 instead of Act No. 3031. Indeed, under Act No. 3606, (1) the surcharge of 25 per cent is based on the tax only, instead
of on both the tax and the interest, as provided for in Act No. 3031, and (2) the taxpayer is allowed twenty days from notice and demand by rthe Collector of Internal
Revenue within which to pay the tax, instead of ten days only as required by the old law.

Properly speaking, a statute is penal when it imposes punishment for an offense committed against the state which, under the Constitution, the Executive has the
power to pardon. In common use, however, this sense has been enlarged to include within the term "penal statutes" all status which command or prohibit certain
acts, and establish penalties for their violation, and even those which, without expressly prohibiting certain acts, impose a penalty upon their commission (59 C. J., p.
1110). Revenue laws, generally, which impose taxes collected by the means ordinarily resorted to for the collection of taxes are not classed as penal laws, although
there are authorities to the contrary. (See Sutherland, Statutory Construction, 361; Twine Co. vs. Worthington, 141 U. S., 468; 12 Sup. Ct., 55; Rice vs. U. S., 4 C. C. A.,
104; 53 Fed., 910; Com. vs. Standard Oil Co., 101 Pa. St., 150; State vs. Wheeler, 44 P., 430; 25 Nev. 143.) Article 22 of the Revised Penal Code is not applicable to the
case at bar, and in the absence of clear legislative intent, we cannot give Act No. 3606 a retroactive effect.

(e) The plaintiff correctly states that the liability to pay a tax may arise at a certain time and the tax may be paid within another given time. As stated by this court,
"the mere failure to pay one's tax does not render one delinqent until and unless the entire period has eplased within which the taxpayer is authorized by law to
make such payment without being subjected to the payment of penalties for fasilure to pay his taxes within the prescribed period." (U. S. vs. Labadan, 26 Phil., 239.)

The defendant maintains that it was the duty of the executor to pay the inheritance tax before the delivery of the decedent's property to the trustee. Stated
otherwise, the defendant contends that delivery to the trustee was delivery to the cestui que trust, the beneficiery in this case, within the meaning of the first
paragraph of subsection (b) of section 1544 of the Revised Administrative Code. This contention is well taken and is sustained. The appointment of P. J. M. Moore as
trustee was made by the trial court in conformity with the wishes of the testator as expressed in his will. It is true that the word "trust" is not mentioned or used in
the will but the intention to create one is clear. No particular or technical words are required to create a testamentary trust (69 C. J., p. 711). The words "trust" and
"trustee", though apt for the purpose, are not necessary. In fact, the use of these two words is not conclusive on the question that a trust is created (69 C. J., p. 714).
"To create a trust by will the testator must indicate in the will his intention so to do by using language sufficient to separate the legal from the equitable estate, and
with sufficient certainty designate the beneficiaries, their interest in the ttrust, the purpose or object of the trust, and the property or subject matter thereof. Stated
otherwise, to constitute a valid testamentary trust there must be a concurrence of three circumstances: (1) Sufficient words to raise a trust; (2) a definite subject; (3)
a certain or ascertain object; statutes in some jurisdictions expressly or in effect so providing." (69 C. J., pp. 705,706.) There is no doubt that the testator intended to
create a trust. He ordered in his will that certain of his properties be kept together undisposed during a fixed period, for a stated purpose. The probate court certainly
exercised sound judgment in appointment a trustee to carry into effect the provisions of the will (see sec. 582, Code of Civil Procedure).

P. J. M. Moore became trustee on March 10, 1924. On that date trust estate vested in him (sec. 582 in relation to sec. 590, Code of Civil Procedure). The mere fact
that the estate of the deceased was placed in trust did not remove it from the operation of our inheritance tax laws or exempt it from the payment of the inheritance
tax. The corresponding inheritance tax should have been paid on or before March 10, 1924, to escape the penalties of the laws. This is so for the reason already
stated that the delivery of the estate to the trustee was in esse delivery of the same estate to the cestui que trust, the beneficiary in this case. A trustee is but an
instrument or agent for the cestui que trust (Shelton vs. King, 299 U. S., 90; 33 Sup. Ct. Rep., 689; 57 Law. ed., 1086). When Moore accepted the trust and took
possesson of the trust estate he thereby admitted that the estate belonged not to him but to his cestui que trust (Tolentino vs. Vitug, 39 Phil.,126, cited in 65 C. J., p.
692, n. 63). He did not acquire any beneficial interest in the estate. He took such legal estate only as the proper execution of the trust required (65 C. J., p. 528) and,
his estate ceased upon the fulfillment of the testator's wishes. The estate then vested absolutely in the beneficiary (65 C. J., p. 542).

The highest considerations of public policy also justify the conclusion we have reached. Were we to hold that the payment of the tax could be postponed or delayed
by the creation of a trust of the type at hand, the result would be plainly disastrous. Testators may provide, as Thomas Hanley has provided, that their estates be not
delivered to their beneficiaries until after the lapse of a certain period of time. In the case at bar, the period is ten years. In other cases, the trust may last for fifty
years, or for a longer period which does not offend the rule against petuities. The collection of the tax would then be left to the will of a private individual. The mere
suggestion of this result is a sufficient warning against the accpetance of the essential to the very exeistence of government. (Dobbins vs. Erie Country, 16 Pet., 435;
10 Law. ed., 1022; Kirkland vs. Hotchkiss, 100 U. S., 491; 25 Law. ed., 558; Lane County vs. Oregon, 7 Wall., 71; 19 Law. ed., 101; Union Refrigerator Transit Co. vs.
Kentucky, 199 U. S., 194; 26 Sup. Ct. Rep., 36; 50 Law. ed., 150; Charles River Bridge vs. Warren Bridge, 11 Pet., 420; 9 Law. ed., 773.) The obligation to pay taxes rests
not upon the privileges enjoyed by, or the protection afforded to, a citizen by the government but upon the necessity of money for the support of the state (Dobbins
vs. Erie Country, supra). For this reason, no one is allowed to object to or resist the payment of taxes solely because no personal benefit to him can be pointed out.
(Thomas vs. Gay, 169 U. S., 264; 18 Sup. Ct. Rep., 340; 43 Law. ed., 740.) While courts will not enlarge, by construction, the government's power of taxation (Bromley
vs. McCaughn, 280 U. S., 124; 74 Law. ed., 226; 50 Sup. Ct. Rep., 46) they also will not place upon tax laws so loose a construction as to permit evasions on merely
fanciful and insubstantial distictions. (U. S. vs. Watts, 1 Bond., 580; Fed. Cas. No. 16,653; U. S. vs. Wigglesirth, 2 Story, 369; Fed. Cas. No. 16,690, followed in Froelich &
Kuttner vs. Collector of Customs, 18 Phil., 461, 481; Castle Bros., Wolf & Sons vs. McCoy, 21 Phil., 300; Muoz & Co. vs. Hord, 12 Phil., 624; Hongkong & Shanghai
Banking Corporation vs. Rafferty, 39 Phil., 145; Luzon Stevedoring Co. vs. Trinidad, 43 Phil., 803.) When proper, a tax statute should be construed to avoid the
possibilities of tax evasion. Construed this way, the statute, without resulting in injustice to the taxpayer, becomes fair to the government.

That taxes must be collected promptly is a policy deeply intrenched in our tax system. Thus, no court is allowed to grant injunction to restrain the collection of any
internal revenue tax ( sec. 1578, Revised Administrative Code; Sarasola vs. Trinidad, 40 Phil., 252). In the case of Lim Co Chui vs. Posadas (47 Phil., 461), this court had
occassion to demonstrate trenchment adherence to this policy of the law. It held that "the fact that on account of riots directed against the Chinese on October 18,
19, and 20, 1924, they were prevented from praying their internal revenue taxes on time and by mutual agreement closed their homes and stores and remained
therein, does not authorize the Collector of Internal Revenue to extend the time prescribed for the payment of the taxes or to accept them without the additional
penalty of twenty five per cent." (Syllabus, No. 3.)

". . . It is of the utmost importance," said the Supreme Court of the United States, ". . . that the modes adopted to enforce the taxes levied should be interfered with
as little as possible. Any delay in the proceedings of the officers, upon whom the duty is developed of collecting the taxes, may derange the operations of
government, and thereby, cause serious detriment to the public." (Dows vs. Chicago, 11 Wall., 108; 20 Law. ed., 65, 66; Churchill and Tait vs. Rafferty, 32 Phil., 580.)

It results that the estate which plaintiff represents has been delinquent in the payment of inheritance tax and, therefore, liable for the payment of interest and
surcharge provided by law in such cases.

The delinquency in payment occurred on March 10, 1924, the date when Moore became trustee. The interest due should be computed from that date and it is error
on the part of the defendant to compute it one month later. The provisions cases is mandatory (see and cf. Lim Co Chui vs. Posadas, supra), and neither the Collector
of Internal Revenuen or this court may remit or decrease such interest, no matter how heavily it may burden the taxpayer.

To the tax and interest due and unpaid within ten days after the date of notice and demand thereof by the Collector of Internal Revenue, a surcharge of twenty-five
per centum should be added (sec. 1544, subsec. (b), par. 2, Revised Administrative Code). Demand was made by the Deputy Collector of Internal Revenue upon
Moore in a communiction dated October 16, 1931 (Exhibit 29). The date fixed for the payment of the tax and interest was November 30, 1931. November 30 being an
official holiday, the tenth day fell on December 1, 1931. As the tax and interest due were not paid on that date, the estate became liable for the payment of the
surcharge.

In view of the foregoing, it becomes unnecessary for us to discuss the fifth error assigned by the plaintiff in his brief.

We shall now compute the tax, together with the interest and surcharge due from the estate of Thomas Hanley inaccordance with the conclusions we have reached.

At the time of his death, the deceased left real properties valued at P27,920 and personal properties worth P1,465, or a total of P29,385. Deducting from this amount
the sum of P480.81, representing allowable deductions under secftion 1539 of the Revised Administrative Code, we have P28,904.19 as the net value of the estate
subject to inheritance tax.

The primary tax, according to section 1536, subsection (c), of the Revised Administrative Code, should be imposed at the rate of one per centum upon the first ten
thousand pesos and two per centum upon the amount by which the share exceed thirty thousand pesos, plus an additional two hundred per centum. One per centum
of ten thousand pesos is P100. Two per centum of P18,904.19 is P378.08. Adding to these two sums an additional two hundred per centum, or P965.16, we have as
primary tax, correctly computed by the defendant, the sum of P1,434.24.

To the primary tax thus computed should be added the sums collectible under section 1544 of the Revised Administrative Code. First should be added P1,465.31
which stands for interest at the rate of twelve per centum per annum from March 10, 1924, the date of delinquency, to September 15, 1932, the date of payment
under protest, a period covering 8 years, 6 months and 5 days. To the tax and interest thus computed should be added the sum of P724.88, representing a surhcarge
of 25 per cent on both the tax and interest, and also P10, the compromise sum fixed by the defendant (Exh. 29), giving a grand total of P3,634.43.

As the plaintiff has already paid the sum of P2,052.74, only the sums of P1,581.69 is legally due from the estate. This last sum is P390.42 more than the amount
demanded by the defendant in his counterclaim. But, as we cannot give the defendant more than what he claims, we must hold that the plaintiff is liable only in the
sum of P1,191.27 the amount stated in the counterclaim.

The judgment of the lower court is accordingly modified, with costs against the plaintiff in both instances. So ordered.


































Republic of the Philippines
SUPREME COURT
Manila

SECOND DIVISION

G.R. No. 82027 March 29, 1990

ROMARICO G. VITUG, petitioner,
vs.
THE HONORABLE COURT OF APPEALS and ROWENA FAUSTINO-CORONA, respondents.

Rufino B. Javier Law Office for petitioner.

Quisumbing, Torres & Evangelista for private respondent.



SARMIENTO, J.:

This case is a chapter in an earlier suit decided by this Court 1 involving the probate of the two wills of the late Dolores Luchangco Vitug, who died in New York, U.
S.A., on November 10, 1980, naming private respondent Rowena Faustino-Corona executrix. In our said decision, we upheld the appointment of Nenita Alonte as co-
special administrator of Mrs. Vitug's estate with her (Mrs. Vitug's) widower, petitioner Romarico G. Vitug, pending probate.

On January 13, 1985, Romarico G. Vitug filed a motion asking for authority from the probate court to sell certain shares of stock and real properties belonging to the
estate to cover allegedly his advances to the estate in the sum of P667,731.66, plus interests, which he claimed were personal funds. As found by the Court of
Appeals, 2 the alleged advances consisted of P58,147.40 spent for the payment of estate tax, P518,834.27 as deficiency estate tax, and P90,749.99 as "increment
thereto." 3 According to Mr. Vitug, he withdrew the sums of P518,834.27 and P90,749.99 from savings account No. 35342-038 of the Bank of America, Makati, Metro
Manila.

On April 12, 1985, Rowena Corona opposed the motion to sell on the ground that the same funds withdrawn from savings account No. 35342-038 were conjugal
partnership properties and part of the estate, and hence, there was allegedly no ground for reimbursement. She also sought his ouster for failure to include the sums
in question for inventory and for "concealment of funds belonging to the estate." 4

Vitug insists that the said funds are his exclusive property having acquired the same through a survivorship agreement executed with his late wife and the bank on
June 19, 1970. The agreement provides:

We hereby agree with each other and with the BANK OF AMERICAN NATIONAL TRUST AND SAVINGS ASSOCIATION (hereinafter referred to as the BANK), that all
money now or hereafter deposited by us or any or either of us with the BANK in our joint savings current account shall be the property of all or both of us and shall be
payable to and collectible or withdrawable by either or any of us during our lifetime, and after the death of either or any of us shall belong to and be the sole
property of the survivor or survivors, and shall be payable to and collectible or withdrawable by such survivor or survivors.

We further agree with each other and the BANK that the receipt or check of either, any or all of us during our lifetime, or the receipt or check of the survivor or
survivors, for any payment or withdrawal made for our above-mentioned account shall be valid and sufficient release and discharge of the BANK for such payment or
withdrawal. 5

The trial courts 6 upheld the validity of this agreement and granted "the motion to sell some of the estate of Dolores L. Vitug, the proceeds of which shall be used to
pay the personal funds of Romarico Vitug in the total sum of P667,731.66 ... ." 7

On the other hand, the Court of Appeals, in the petition for certiorari filed by the herein private respondent, held that the above-quoted survivorship agreement
constitutes a conveyance mortis causa which "did not comply with the formalities of a valid will as prescribed by Article 805 of the Civil Code," 8 and secondly,
assuming that it is a mere donation inter vivos, it is a prohibited donation under the provisions of Article 133 of the Civil Code. 9

The dispositive portion of the decision of the Court of Appeals states:

WHEREFORE, the order of respondent Judge dated November 26, 1985 (Annex II, petition) is hereby set aside insofar as it granted private respondent's motion to sell
certain properties of the estate of Dolores L. Vitug for reimbursement of his alleged advances to the estate, but the same order is sustained in all other respects. In
addition, respondent Judge is directed to include provisionally the deposits in Savings Account No. 35342-038 with the Bank of America, Makati, in the inventory of
actual properties possessed by the spouses at the time of the decedent's death. With costs against private respondent. 10

In his petition, Vitug, the surviving spouse, assails the appellate court's ruling on the strength of our decisions in Rivera v. People's Bank and Trust Co. 11 and Macam
v. Gatmaitan 12 in which we sustained the validity of "survivorship agreements" and considering them as aleatory contracts. 13

The petition is meritorious.

The conveyance in question is not, first of all, one of mortis causa, which should be embodied in a will. A will has been defined as "a personal, solemn, revocable and
free act by which a capacitated person disposes of his property and rights and declares or complies with duties to take effect after his death." 14 In other words, the
bequest or device must pertain to the testator. 15 In this case, the monies subject of savings account No. 35342-038 were in the nature of conjugal funds In the case
relied on, Rivera v. People's Bank and Trust Co., 16 we rejected claims that a survivorship agreement purports to deliver one party's separate properties in favor of
the other, but simply, their joint holdings:

... Such conclusion is evidently predicated on the assumption that Stephenson was the exclusive owner of the funds-deposited in the bank, which assumption was in
turn based on the facts (1) that the account was originally opened in the name of Stephenson alone and (2) that Ana Rivera "served only as housemaid of the
deceased." But it not infrequently happens that a person deposits money in the bank in the name of another; and in the instant case it also appears that Ana Rivera
served her master for about nineteen years without actually receiving her salary from him. The fact that subsequently Stephenson transferred the account to the
name of himself and/or Ana Rivera and executed with the latter the survivorship agreement in question although there was no relation of kinship between them but
only that of master and servant, nullifies the assumption that Stephenson was the exclusive owner of the bank account. In the absence, then, of clear proof to the
contrary, we must give full faith and credit to the certificate of deposit which recites in effect that the funds in question belonged to Edgar Stephenson and Ana
Rivera; that they were joint (and several) owners thereof; and that either of them could withdraw any part or the whole of said account during the lifetime of both,
and the balance, if any, upon the death of either, belonged to the survivor. 17

xxx xxx xxx

In Macam v. Gatmaitan, 18 it was held:

xxx xxx xxx

This Court is of the opinion that Exhibit C is an aleatory contract whereby, according to article 1790 of the Civil Code, one of the parties or both reciprocally bind
themselves to give or do something as an equivalent for that which the other party is to give or do in case of the occurrence of an event which is uncertain or will
happen at an indeterminate time. As already stated, Leonarda was the owner of the house and Juana of the Buick automobile and most of the furniture. By virtue of
Exhibit C, Juana would become the owner of the house in case Leonarda died first, and Leonarda would become the owner of the automobile and the furniture if
Juana were to die first. In this manner Leonarda and Juana reciprocally assigned their respective property to one another conditioned upon who might die first, the
time of death determining the event upon which the acquisition of such right by the one or the other depended. This contract, as any other contract, is binding upon
the parties thereto. Inasmuch as Leonarda had died before Juana, the latter thereupon acquired the ownership of the house, in the same manner as Leonarda would
have acquired the ownership of the automobile and of the furniture if Juana had died first. 19

xxx xxx xxx

There is no showing that the funds exclusively belonged to one party, and hence it must be presumed to be conjugal, having been acquired during the existence of
the marita. relations. 20

Neither is the survivorship agreement a donation inter vivos, for obvious reasons, because it was to take effect after the death of one party. Secondly, it is not a
donation between the spouses because it involved no conveyance of a spouse's own properties to the other.

It is also our opinion that the agreement involves no modification petition of the conjugal partnership, as held by the Court of Appeals, 21 by "mere stipulation" 22
and that it is no "cloak" 23 to circumvent the law on conjugal property relations. Certainly, the spouses are not prohibited by law to invest conjugal property, say, by
way of a joint and several bank account, more commonly denominated in banking parlance as an "and/or" account. In the case at bar, when the spouses Vitug
opened savings account No. 35342-038, they merely put what rightfully belonged to them in a money-making venture. They did not dispose of it in favor of the other,
which would have arguably been sanctionable as a prohibited donation. And since the funds were conjugal, it can not be said that one spouse could have pressured
the other in placing his or her deposits in the money pool.

The validity of the contract seems debatable by reason of its "survivor-take-all" feature, but in reality, that contract imposed a mere obligation with a term, the term
being death. Such agreements are permitted by the Civil Code. 24

Under Article 2010 of the Code:

ART. 2010. By an aleatory contract, one of the parties or both reciprocally bind themselves to give or to do something in consideration of what the other shall give or
do upon the happening of an event which is uncertain, or which is to occur at an indeterminate time.

Under the aforequoted provision, the fulfillment of an aleatory contract depends on either the happening of an event which is (1) "uncertain," (2) "which is to occur
at an indeterminate time." A survivorship agreement, the sale of a sweepstake ticket, a transaction stipulating on the value of currency, and insurance have been held
to fall under the first category, while a contract for life annuity or pension under Article 2021, et sequentia, has been categorized under the second. 25 In either case,
the element of risk is present. In the case at bar, the risk was the death of one party and survivorship of the other.

However, as we have warned:

xxx xxx xxx

But although the survivorship agreement is per se not contrary to law its operation or effect may be violative of the law. For instance, if it be shown in a given case
that such agreement is a mere cloak to hide an inofficious donation, to transfer property in fraud of creditors, or to defeat the legitime of a forced heir, it may be
assailed and annulled upon such grounds. No such vice has been imputed and established against the agreement involved in this case. 26

xxx xxx xxx

There is no demonstration here that the survivorship agreement had been executed for such unlawful purposes, or, as held by the respondent court, in order to
frustrate our laws on wills, donations, and conjugal partnership.

The conclusion is accordingly unavoidable that Mrs. Vitug having predeceased her husband, the latter has acquired upon her death a vested right over the amounts
under savings account No. 35342-038 of the Bank of America. Insofar as the respondent court ordered their inclusion in the inventory of assets left by Mrs. Vitug, we
hold that the court was in error. Being the separate property of petitioner, it forms no more part of the estate of the deceased.

WHEREFORE, the decision of the respondent appellate court, dated June 29, 1987, and its resolution, dated February 9, 1988, are SET ASIDE.
No costs. SO ORDERED.














Republic of the Philippines
SUPREME COURT
Manila
THIRD DIVISION
G.R. No. 122880 April 12, 2006
FELIX AZUELA, Petitioner,
vs.
COURT OF APPEALS, GERALDA AIDA CASTILLO substituted by ERNESTO G. CASTILLO, Respondents.
D E C I S I O N
TINGA, J .:
The core of this petition is a highly defective notarial will, purportedly executed by Eugenia E. Igsolo (decedent), who died on 16 December 1982 at the age of 80. In
refusing to give legal recognition to the due execution of this document, the Court is provided the opportunity to assert a few important doctrinal rules in the execution of
notarial wills, all self-evident in view of Articles 805 and 806 of the Civil Code.
A will whose attestation clause does not contain the number of pages on which the will is written is fatally defective. A will whose attestation clause is not
signed by the instrumental witnesses is fatally defective. And perhaps most importantly, a will which does not contain an acknowledgment, but a mere jurat,
is fatally defective. Any one of these defects is sufficient to deny probate. A notarial will with all three defects is just aching for judicial rejection.
There is a distinct and consequential reason the Civil Code provides a comprehensive catalog of imperatives for the proper execution of a notarial will. Full and faithful
compliance with all the detailed requisites under Article 805 of the Code leave little room for doubt as to the validity in the due execution of the notarial will. Article 806
likewise imposes another safeguard to the validity of notarial wills that they be acknowledged before a notary public by the testator and the witnesses. A notarial will
executed with indifference to these two codal provisions opens itself to nagging questions as to its legitimacy.
The case stems from a petition for probate filed on 10 April 1984 with the Regional Trial Court (RTC) of Manila. The petition filed by petitioner Felix Azuela sought to
admit to probate the notarial will of Eugenia E. Igsolo, which was notarized on 10 June 1981. Petitioner is the son of the cousin of the decedent.
The will, consisting of two (2) pages and written in the vernacular Pilipino, read in full:
HULING HABILIN NI EUGENIA E. IGSOLO
SA NGALAN NG MAYKAPAL, AMEN:
AKO, si EUGENIA E. IGSOLO, nakatira sa 500 San Diego St., Sampaloc, Manila, pitongput siyam (79) na gulang, nasa hustong pagi-isip, pag-unawa at memoria ay
nag-hahayag na ito na ang aking huling habilin at testamento, at binabali wala ko lahat ang naunang ginawang habilin o testamento:
Una-Hinihiling ko na ako ay mailibing sa Sementerio del Norte, La Loma sang-ayong sa kaugalian at patakaran ng simbahang katoliko at ang taga-pag-ingat (Executor)
ng habiling ito ay magtatayo ng bantayog upang silbing ala-ala sa akin ng aking pamilya at kaibigan;
Pangalawa-Aking ipinagkakaloob at isinasalin ang lahat ng karapatan sa aking pamangkin na si Felix Azuela, na siyang nag-alaga sa akin sa mahabang panahon, yaong
mga bahay na nakatirik sa lote numero 28, Block 24 at nakapangalan sa Pechaten Korporasyon, ganoon din ibinibigay ko ang lahat ng karapatan sa bahay na nakatirik
sa inoopahan kong lote, numero 43, Block 24 na pag-aari ng Pechaten Corporation. Ipinagkakaloob kong buong buo ang lahat ng karapatan sa bahay at lupa na nasa
500 San Diego St., Lot 42, Block 24, Sampaloc, Manila kay Felix Azuela at ang pagkakaloob kong ito ay walang pasubalit at kondiciones;
Pangatlo- Na ninunumbrahan ko si VART PAGUE na siyang nagpapatupad ng huling habiling ito at kagustuhan ko rin na hindi na kailanman siyang mag-lagak ng
piyansiya.
Aking nilagdaan ang Huling Habilin na ito dito sa Maynila ika 10 ng Hunyo, 1981.
(Sgd.)
EUGENIA E. IGSOLO
(Tagapagmana)
PATUNAY NG MGA SAKSI
Ang kasulatang ito, na binubuo ng ____ dahon pati ang huling dahong ito, na ipinahayag sa amin ni Eugenia E. Igsolo, tagapagmana na siya niyang Huling Habilin,
ngayon ika-10 ng Hunyo 1981, ay nilagdaan ng nasabing tagapagmana sa ilalim ng kasulatang nabanggit at sa kaliwang panig ng lahat at bawat dahon, sa harap ng
lahat at bawat sa amin, at kami namang mga saksi ay lumagda sa harap ng nasabing tagapagmana at sa harap ng lahat at bawat isa sa amin, sa ilalim ng nasabing
kasulatan at sa kaliwang panig ng lahat at bawat dahon ng kasulatan ito.
EUGENIA E. IGSOLO
address: 500 San Diego St.
Sampaloc, Manila Res. Cert. No. A-7717-37
Issued at Manila on March 10, 1981.
QUIRINO AGRAVA
address: 1228-Int. 3, Kahilum
Pandacan, Manila Res. Cert. No. A-458365
Issued at Manila on Jan. 21, 1981
LAMBERTO C. LEAO
address: Avenue 2, Blcok 7,
Lot 61, San Gabriel, G.MA., Cavite Res.
Cert. No. A-768277 issued at Carmona, Cavite on Feb. 7, 1981
JUANITO ESTRERA
address: City Court Compound,
City of Manila Res. Cert. No. A574829
Issued at Manila on March 2, 1981.
Nilagdaan ko at ninotario ko ngayong 10 ng Hunyo 10, 1981 dito sa Lungsod ng Maynila.
(Sgd.)
PETRONIO Y. BAUTISTA
Doc. No. 1232 ; NOTARIO PUBLIKO
Page No. 86 ; Until Dec. 31, 1981
Book No. 43 ; PTR-152041-1/2/81-Manila
Series of 1981 TAN # 1437-977-8
1

The three named witnesses to the will affixed their signatures on the left-hand margin of both pages of the will, but not at the bottom of the attestation clause.
The probate petition adverted to only two (2) heirs, legatees and devisees of the decedent, namely: petitioner himself, and one Irene Lynn Igsolo, who was alleged to
have resided abroad. Petitioner prayed that the will be allowed, and that letters testamentary be issued to the designated executor, Vart Prague.
The petition was opposed by Geralda Aida Castillo (Geralda Castillo), who represented herself as the attorney-in-fact of "the 12 legitimate heirs" of the
decedent.
2
Geralda Castillo claimed that the will is a forgery, and that the true purpose of its emergence was so it could be utilized as a defense in several court cases
filed by oppositor against petitioner, particularly for forcible entry and usurpation of real property, all centering on petitioners right to occupy the properties of the
decedent.
3
It also asserted that contrary to the representations of petitioner, the decedent was actually survived by 12 legitimate heirs, namely her grandchildren, who
were then residing abroad. Per records, it was subsequently alleged that decedent was the widow of Bonifacio Igsolo, who died in 1965,
4
and the mother of a legitimate
child, Asuncion E. Igsolo, who predeceased her mother by three (3) months.
5

Oppositor Geralda Castillo also argued that the will was not executed and attested to in accordance with law. She pointed out that decedents signature did not appear on
the second page of the will, and the will was not properly acknowledged. These twin arguments are among the central matters to this petition.
After due trial, the RTC admitted the will to probate, in an Order dated 10 August 1992.
6
The RTC favorably took into account the testimony of the three (3) witnesses to
the will, Quirino Agrava, Lamberto Leano, and Juanito Estrada. The RTC also called to fore "the modern tendency in respect to the formalities in the execution of a will x
x x with the end in view of giving the testator more freedom in expressing his last wishes;"
7
and from this perspective, rebutted oppositors arguments that the will was not
properly executed and attested to in accordance with law.
After a careful examination of the will and consideration of the testimonies of the subscribing and attesting witnesses, and having in mind the modern tendency in respect
to the formalities in the execution of a will, i.e., the liberalization of the interpretation of the law on the formal requirements of a will with the end in view of giving the
testator more freedom in expressing his last wishes, this Court is persuaded to rule that the will in question is authentic and had been executed by the testatrix in
accordance with law.
On the issue of lack of acknowledgement, this Court has noted that at the end of the will after the signature of the testatrix, the following statement is made under the
sub-title, "Patunay Ng Mga Saksi":
"Ang kasulatang ito, na binubuo ng _____ dahon pati ang huling dahong ito, na ipinahayag sa amin ni Eugenia N. Igsolo, tagapagmana na siya niyang Huling Habilin,
ngayong ika-10 ng Hunyo 1981, ay nilagdaan ng nasabing tagapagmana sa ilalim ng kasulatang nabanggit at sa kaliwang panig ng lahat at bawat dahon, sa harap ng
lahat at bawat sa amin, at kami namang mga saksi ay lumagda sa harap ng nasabing tagapagmana at sa harap ng lahat at bawat isa sa amin, sa ilalim ng nasabing
kasulatan at sa kaliwang panig ng lahat at bawat dahon ng kasulatan ito."
The aforequoted declaration comprises the attestation clause and the acknowledgement and is considered by this Court as a substantial compliance with the
requirements of the law.
On the oppositors contention that the attestation clause was not signed by the subscribing witnesses at the bottom thereof, this Court is of the view that the signing by
the subscribing witnesses on the left margin of the second page of the will containing the attestation clause and acknowledgment, instead of at the bottom thereof,
substantially satisfies the purpose of identification and attestation of the will.
With regard to the oppositors argument that the will was not numbered correlatively in letters placed on upper part of each page and that the attestation did not state the
number of pages thereof, it is worthy to note that the will is composed of only two pages. The first page contains the entire text of the testamentary dispositions, and the
second page contains the last portion of the attestation clause and acknowledgement. Such being so, the defects are not of a serious nature as to invalidate the will. For
the same reason, the failure of the testatrix to affix her signature on the left margin of the second page, which contains only the last portion of the attestation clause and
acknowledgment is not a fatal defect.
As regards the oppositors assertion that the signature of the testatrix on the will is a forgery, the testimonies of the three subscribing witnesses to the will are convincing
enough to establish the genuineness of the signature of the testatrix and the due execution of the will.
8

The Order was appealed to the Court of Appeals by Ernesto Castillo, who had substituted his since deceased mother-in-law, Geralda Castillo. In a Decision dated 17
August 1995, the Court of Appeals reversed the trial court and ordered the dismissal of the petition for probate.
9
The Court of Appeals noted that the attestation clause
failed to state the number of pages used in the will, thus rendering the will void and undeserving of probate.
10

Hence, the present petition.
Petitioner argues that the requirement under Article 805 of the Civil Code that "the number of pages used in a notarial will be stated in the attestation clause" is merely
directory, rather than mandatory, and thus susceptible to what he termed as "the substantial compliance rule."
11

The solution to this case calls for the application of Articles 805 and 806 of the Civil Code, which we replicate in full.
Art. 805. Every will, other than a holographic will, must be subscribed at the end thereof by the testator himself or by the testator's name written by some other person in
his presence, and by his express direction, and attested and subscribed by three or more credible witnesses in the presence of the testator and of one another.
The testator or the person requested by him to write his name and the instrumental witnesses of the will, shall also sign, as aforesaid, each and every page thereof,
except the last, on the left margin, and all the pages shall be numbered correlatively in letters placed on the upper part of each page.
The attestation shall state the number of pages used upon which the will is written, and the fact that the testator signed the will and every page thereof, or caused some
other person to write his name, under his express direction, in the presence of the instrumental witnesses, and that the latter witnessed and signed the will and all the
pages thereof in the presence of the testator and of one another.
If the attestation clause is in a language not known to the witnesses, it shall be interpreted to them.
Art. 806. Every will must be acknowledged before a notary public by the testator and the witnesses. The notary public shall not be required to retain a copy of the will, or
file another with the office of the Clerk of Court.
The appellate court, in its Decision, considered only one defect, the failure of the attestation clause to state the number of pages of the will. But an examination of the will
itself reveals several more deficiencies.
As admitted by petitioner himself, the attestation clause fails to state the number of pages of the will.
12
There was an incomplete attempt to comply with this requisite, a
space having been allotted for the insertion of the number of pages in the attestation clause. Yet the blank was never filled in; hence, the requisite was left uncomplied
with.
The Court of Appeals pounced on this defect in reversing the trial court, citing in the process Uy Coque v. Navas L. Sioca
13
and In re: Will of Andrada.
14
In Uy Coque, the
Court noted that among the defects of the will in question was the failure of the attestation clause to state the number of pages contained in the will.
15
In ruling that the will
could not be admitted to probate, the Court made the following consideration which remains highly relevant to this day: "The purpose of requiring the number of sheets to
be stated in the attestation clause is obvious; the document might easily be so prepared that the removal of a sheet would completely change the testamentary
dispositions of the will and in the absence of a statement of the total number of sheets such removal might be effected by taking out the sheet and changing
the numbers at the top of the following sheets or pages. If, on the other hand, the total number of sheets is stated in the attestation clause the falsification of the
document will involve the inserting of new pages and the forging of the signatures of the testator and witnesses in the margin, a matter attended with much greater
difficulty."
16

The case of In re Will of Andrada concerned a will the attestation clause of which failed to state the number of sheets or pages used. This consideration alone was
sufficient for the Court to declare "unanim[ity] upon the point that the defect pointed out in the attesting clause is fatal."
17
It was further observed that "it cannot be denied
that the x x x requirement affords additional security against the danger that the will may be tampered with; and as the Legislature has seen fit to prescribe this
requirement, it must be considered material."
18

Against these cited cases, petitioner cites Singson v. Florentino
19
and Taboada v. Hon. Rosal,
20
wherein the Court allowed probate to the wills concerned therein despite
the fact that the attestation clause did not state the number of pages of the will. Yet the appellate court itself considered the import of these two cases, and made the
following distinction which petitioner is unable to rebut, and which we adopt with approval:
Even a cursory examination of the Will (Exhibit "D"), will readily show that the attestation does not state the number of pages used upon which the will is written. Hence,
the Will is void and undeserving of probate.
We are not impervious of the Decisions of the Supreme Court in "Manuel Singson versus Emilia Florentino, et al., 92 Phil. 161 and Apolonio [Taboada] versus Hon.
Avelino Rosal, et al., 118 SCRA 195," to the effect that a will may still be valid even if the attestation does not contain the number of pages used upon which the Will is
written. However, the Decisions of the Supreme Court are not applicable in the aforementioned appeal at bench. This is so because, in the case of "Manuel Singson
versus Emilia Florentino, et al., supra," although the attestation in the subject Will did not state the number of pages used in the will, however, the same was found in the
last part of the body of the Will:
"x x x
The law referred to is article 618 of the Code of Civil Procedure, as amended by Act No. 2645, which requires that the attestation clause shall state the number of pages
or sheets upon which the will is written, which requirement has been held to be mandatory as an effective safeguard against the possibility of interpolation or omission of
some of the pages of the will to the prejudice of the heirs to whom the property is intended to be bequeathed (In re Will of Andrada, 42 Phil. 180; Uy Coque vs. Navas L.
Sioca, 43 Phil., 405; Gumban vs. Gorcho, 50 Phil. 30; Quinto vs. Morata, 54 Phil. 481; Echevarria vs. Sarmiento, 66 Phil. 611). The ratio decidendi of these cases seems
to be that the attestation clause must contain a statement of the number of sheets or pages composing the will and that if this is missing or is omitted, it will have the
effect of invalidating the will if the deficiency cannot be supplied, not by evidence aliunde, but by a consideration or examination of the will itself. But here the situation is
different. While the attestation clause does not state the number of sheets or pages upon which the will is written, however, the last part of the body of the will contains a
statement that it is composed of eight pages, which circumstance in our opinion takes this case out of the rigid rule of construction and places it within the realm of similar
cases where a broad and more liberal view has been adopted to prevent the will of the testator from being defeated by purely technical considerations." (page 165-165,
supra) (Underscoring supplied)
In "Apolonio Tabaoda versus Hon. Avelino Rosal, et al." supra, the notarial acknowledgement in the Will states the number of pages used in the:
"x x x
We have examined the will in question and noticed that the attestation clause failed to state the number of pages used in writing the will. This would have been a fatal
defect were it not for the fact that, in this case, it is discernible from the entire will that it is really and actually composed of only two pages duly signed by the testatrix and
her instrumental witnesses. As earlier stated, the first page which contains the entirety of the testamentary dispositions is signed by the testatrix at the end or at the
bottom while the instrumental witnesses signed at the left margin. The other page which is marked as "Pagina dos" comprises the attestation clause and the
acknowledgment. The acknowledgment itself states that "this Last Will and Testament consists of two pages including this page" (pages 200-201, supra) (Underscoring
supplied).
However, in the appeal at bench, the number of pages used in the will is not stated in any part of the Will. The will does not even contain any notarial acknowledgment
wherein the number of pages of the will should be stated.
21

Both Uy Coque and Andrada were decided prior to the enactment of the Civil Code in 1950, at a time when the statutory provision governing the formal requirement of
wills was Section
618 of the Code of Civil Procedure.
22
Reliance on these cases remains apropos, considering that the requirement that the attestation state the number of pages of the will
is extant from Section 618.
23
However, the enactment of the Civil Code in 1950 did put in force a rule of interpretation of the requirements of wills, at least insofar as the
attestation clause is concerned, that may vary from the philosophy that governed these two cases. Article 809 of the Civil Code states: "In the absence of bad faith,
forgery, or fraud, or undue and improper pressure and influence, defects and imperfections in the form of attestation or in the language used therein shall not render the
will invalid if it is proved that the will was in fact executed and attested in substantial compliance with all the requirements of article 805."
In the same vein, petitioner cites the report of the Civil Code Commission, which stated that "the underlying and fundamental objective permeating the provisions on the
[law] on [wills] in this project consists in the [liberalization] of the manner of their execution with the end in view of giving the testator more [freedom] in [expressing] his
last wishes. This objective is in accord with the [modern tendency] in respect to the formalities in the execution of wills."
24
However, petitioner conveniently omits the
qualification offered by the Code Commission in the very same paragraph he cites from their report, that such liberalization be "but with sufficient safeguards and
restrictions to prevent the commission of fraud and the exercise of undue and improper pressure and influence upon the testator."
25

Caneda v. Court of Appeals
26
features an extensive discussion made by Justice Regalado, speaking for the Court on the conflicting views on the manner of interpretation of the legal
formalities required in the execution of the attestation clause in wills.
27
Uy Coque and Andrada are cited therein, along with several other cases, as examples of the application of the rule
of strict construction.
28
However, the Code Commission opted to recommend a more liberal construction through the "substantial compliance rule" under Article 809. A cautionary note
was struck though by Justice J.B.L. Reyes as to how Article 809 should be applied:
x x x The rule must be limited to disregarding those defects that can be supplied by an examination of the will itself: whether all the pages are consecutively numbered;
whether the signatures appear in each and every page; whether the subscribing witnesses are three or the will was notarized. All these are facts that the will itself can
reveal, and defects or even omissions concerning them in the attestation clause can be safely disregarded. But the total number of pages, and whether all persons
required to sign did so in the presence of each other must substantially appear in the attestation clause, being the only check against perjury in the probate
proceedings.
29
(Emphasis supplied.)
The Court of Appeals did cite these comments by Justice J.B.L. Reyes in its assailed decision, considering that the failure to state the number of pages of the will in the
attestation clause is one of the defects which cannot be simply disregarded. In Caneda itself, the Court refused to allow the probate of a will whose attestation clause
failed to state that the witnesses subscribed their respective signatures to the will in the presence of the testator and of each other,
30
the other omission cited by Justice
J.B.L. Reyes which to his estimation cannot be lightly disregarded.
Caneda suggested: "[I]t may thus be stated that the rule, as it now stands, is that omission which can be supplied by an examination of the will itself, without the need of
resorting to extrinsic evidence, will not be fatal and, correspondingly, would not obstruct the allowance to probate of the will being assailed. However, those omissions
which cannot be supplied except by evidence aliunde would result in the invalidation of the attestation clause and ultimately, of the will itself."
31
Thus, a failure by the
attestation clause to state that the testator signed every page can be liberally construed, since that fact can be checked by a visual examination; while a failure by the
attestation clause to state that the witnesses signed in one anothers presence should be considered a fatal flaw since the attestation is the only textual guarantee of
compliance.
32

The failure of the attestation clause to state the number of pages on which the will was written remains a fatal flaw, despite Article 809. The purpose of the law in
requiring the clause to state the number of pages on which the will is written is to safeguard against possible interpolation or omission of one or some of its pages and to
prevent any increase or decrease in the pages.
33
The failure to state the number of pages equates with the absence of an averment on the part of the instrumental
witnesses as to how many pages consisted the will, the execution of which they had ostensibly just witnessed and subscribed to. Following Caneda, there is substantial
compliance with this requirement if the will states elsewhere in it how many pages it is comprised of, as was the situation in Singson andTaboada. However, in this case,
there could have been no substantial compliance with the requirements under Article 805 since there is no statement in the attestation clause or anywhere in the will itself
as to the number of pages which comprise the will.
At the same time, Article 809 should not deviate from the need to comply with the formal requirements as enumerated under Article 805. Whatever the inclinations of the
members of the Code Commission in incorporating Article 805, the fact remains that they saw fit to prescribe substantially the same formal requisites as enumerated in
Section 618 of the Code of Civil Procedure, convinced that these remained effective safeguards against the forgery or intercalation of notarial wills.
34
Compliance with
these requirements, however picayune in impression, affords the public a high degree of comfort that the testator himself or herself had decided to convey property post
mortem in the manner established in the will.
35
The transcendent legislative intent, even as expressed in the cited comments of the Code Commission, is for the
fruition of the testators incontestable desires, and not for the indulgent admission of wills to probate.
The Court could thus end here and affirm the Court of Appeals. However, an examination of the will itself reveals a couple of even more critical defects that should
necessarily lead to its rejection.
For one, the attestation clause was not signed by the instrumental witnesses. While the signatures of the instrumental witnesses appear on the left-hand margin of
the will, they do not appear at the bottom of the attestation clause which after all consists of their averments before the notary public.
Cagro v. Cagro
36
is material on this point. As in this case, "the signatures of the three witnesses to the will do not appear at the bottom of the attestation clause, although
the page containing the same is signed by the witnesses on the left-hand margin."
37
While three (3) Justices
38
considered the signature requirement had been
substantially complied with, a majority of six (6), speaking through Chief Justice Paras, ruled that the attestation clause had not been duly signed, rendering the will fatally
defective.
There is no question that the signatures of the three witnesses to the will do not appear at the bottom of the attestation clause, although the page containing the same is
signed by the witnesses on the left-hand margin.
We are of the opinion that the position taken by the appellant is correct. The attestation clause is "a memorandum of the facts attending the execution of the will" required
by law to be made by the attesting witnesses, and it must necessarily bear their signatures. An unsigned attestation clause cannot be considered as an act of the
witnesses, since the omission of their signatures at the bottom thereof negatives their participation.
The petitioner and appellee contends that signatures of the three witnesses on the left-hand margin conform substantially to the law and may be deemed as their
signatures to the attestation clause. This is untenable, because said signatures are in compliance with the legal mandate that the will be signed on the left-hand margin of
all its pages. If an attestation clause not signed by the three witnesses at the bottom thereof, be admitted as sufficient, it would be easy to add such clause to a will on a
subsequent occasion and in the absence of the testator and any or all of the witnesses.
39

The Court today reiterates the continued efficacy of Cagro. Article 805 particularly segregates the requirement that the instrumental witnesses sign each page of the will,
from the requisite that the will be "attested and subscribed by [the instrumental witnesses]." The respective intents behind these two classes of signature are distinct from
each other. The signatures on the left-hand corner of every page signify, among others, that the witnesses are aware that the page they are signing forms part of the will.
On the other hand, the signatures to the attestation clause establish that the witnesses are referring to the statements contained in the attestation clause itself. Indeed,
the attestation clause is separate and apart from the disposition of the will. An unsigned attestation clause results in an unattested will. Even if the instrumental witnesses
signed the left-hand margin of the page containing the unsigned attestation clause, such signatures cannot demonstrate these witnesses undertakings in the clause,
since the signatures that do appear on the page were directed towards a wholly different avowal.
The Court may be more charitably disposed had the witnesses in this case signed the attestation clause itself, but not the left-hand margin of the page containing such
clause. Without diminishing the value of the instrumental witnesses signatures on each and every page, the fact must be noted that it is the attestation clause which
contains the utterances reduced into writing of the testamentary witnesses themselves. It is the witnesses, and not the testator, who are required under Article 805 to
state the number of pages used upon which the will is written; the fact that the testator had signed the will and every page thereof; and that they witnessed and signed the
will and all the pages thereof in the presence of the testator and of one another. The only proof in the will that the witnesses have stated these elemental facts would be
their signatures on the attestation clause.
Thus, the subject will cannot be considered to have been validly attested to by the instrumental witnesses, as they failed to sign the attestation clause.
Yet, there is another fatal defect to the will on which the denial of this petition should also hinge. The requirement under Article 806 that "every will must be acknowledged
before a notary public by the testator and the witnesses" has also not been complied with. The importance of this requirement is highlighted by the fact that it had been
segregated from the other requirements under Article 805 and entrusted into a separate provision, Article 806. The non-observance of Article 806 in this case is equally
as critical as the other cited flaws in compliance with Article 805, and should be treated as of equivalent import.
In lieu of an acknowledgment, the notary public, Petronio Y. Bautista, wrote "Nilagdaan ko at ninotario ko ngayong10 ng Hunyo 10 (sic), 1981 dito sa Lungsod ng
Maynila."
40
By no manner of contemplation can those words be construed as an acknowledgment. An acknowledgment is the act of one who has executed a deed in
going before some competent officer or court and declaring it to be his act or deed.
41
It involves an extra step undertaken whereby the signor actually declares to the



















notary that the executor of a document has attested to the notary that the same is his/her own free act and deed.
It might be possible to construe the averment as a jurat, even though it does not hew to the usual language thereof. A jurat is that part of an affidavit where the notary
certifies that before him/her, the document was subscribed and sworn to by the executor.
42
Ordinarily, the language of the jurat should avow that the document was
subscribed and sworn before the notary public, while in this case, the notary public averred that he himself "signed and notarized" the document. Possibly though, the
word "ninotario" or "notarized" encompasses the signing of and swearing in of the executors of the document, which in this case would involve the decedent and the
instrumental witnesses.
Yet even if we consider what was affixed by the notary public as a jurat, the will would nonetheless remain invalid, as the express requirement of Article 806 is that the
will be "acknowledged", and not merely subscribed and sworn to. The will does not present any textual proof, much less one under oath, that the decedent and the
instrumental witnesses executed or signed the will as their own free act or deed. The acknowledgment made in a will provides for another all-important legal safeguard
against spurious wills or those made beyond the free consent of the testator. An acknowledgement is not an empty meaningless act.
43
The acknowledgment coerces the
testator and the instrumental witnesses to declare before an officer of the law that they had executed and subscribed to the will as their own free act or deed. Such
declaration is under oath and under pain of perjury, thus allowing for the criminal prosecution of persons who participate in the execution of spurious wills, or those
executed without the free consent of the testator. It also provides a further degree of assurance that the testator is of certain mindset in making the testamentary
dispositions to those persons he/she had designated in the will.
It may not have been said before, but we can assert the rule, self-evident as it is under Article 806. A notarial will that is not acknowledged before a notary public by
the testator and the witnesses is fatally defective, even if it is subscribed and sworn to before a notary public.
There are two other requirements under Article 805 which were not fully satisfied by the will in question. We need not discuss them at length, as they are no longer
material to the
disposition of this case. The provision requires that the testator and the instrumental witnesses sign each and every page of the will on the left margin, except the last;
and that all the pages shall be numbered correlatively in letters placed on the upper part of each page. In this case, the decedent, unlike the witnesses, failed to sign both
pages of the will on the left margin, her only signature appearing at the so-called "logical end"
44
of the will on its first page. Also, the will itself is not numbered correlatively
in letters on each page, but instead numbered with Arabic numerals. There is a line of thought that has disabused the notion that these two requirements be construed as
mandatory.
45
Taken in isolation, these omissions, by themselves, may not be sufficient to deny probate to a will. Yet even as these omissions are not decisive to the
adjudication of this case, they need not be dwelt on, though indicative as they may be of a general lack of due regard for the requirements under Article 805 by whoever
executed the will.
All told, the string of mortal defects which the will in question suffers from makes the probate denial inexorable.
WHEREFORE, the petition is DENIED. Costs against petitioner.
SO ORDERED.
DANTE O. TINGA
Associate Justice



Republic of the Philippines
SUPREME COURT
Manila
EN BANC
C.A. No. 8075 March 25, 1946
TRINIDAD NEYRA, plaintiff-appellant,
vs.
ENCARNACION NEYRA, defendant-appellee.
Alejandro M. Panis for appellant.
Lucio Javillonar for appellee.
DE JOYA, J .:
On October 25, 1939, Trinidad Neyra filed a complaint against her sister, Encarnacion Neyra, in the Court of First Instance of the City of Manila, for the recovery of one-
half () of the property mentioned and described therein, which had been left by their deceased father, Severo Neyra, and which had been previously divided equally
between the two extrajudicially, demanding at the same time one-half () of the rents collected on the said property by the defendant Encarnacion Neyra. The defendant
filed an answer admitting that the property mentioned and described therein was community property, and at the same time set up counterclaims amounting to over
P1,000, for money spent, during the last illness of their father, and for money loaned to the plaintiff.
After the trial of the case, the court found that the plaintiff was really entitled to one-half () of the said property, adjudicating the same to her, but at the same time
ordered said plaintiff to pay to the defendant the sum of P727.77, plus interests, by virtue of said counterclaims.
Plaintiff Trinidad Neyra appealed from the said decision, to the Court of Appeals for Manila, alleging several errors, attacking the execution and validity of said agreement;
and on November 10, 1942, said appeal was dismissed, pursuant to the to an agreement or compromise entered into by the parties, as shown by the corresponding
document, dated November 3, 1942, which was filed in the case the following day, November 4, 1942.
In the meanwhile, Encarnacion Neyra, who had been sickly for about two years, unexpectedly died, on November 4, 1942 at the age of 48, allegedly from heart attack, as
a consequence of Addison's disease from which, it was claimed, she had been suffering for sometime.
In view of the decision of the Court of Appeals, dated November 10, 1942, dismissing the appeal, by virtue of said agreement or compromise, Atty. Lucio Javillonar,
claiming to represent Encarnacion Neyra, who had died since November 4, 1942, and other relatives of hers, filed a petition, dated November 23, 1942, asking for the
reconsideration of said decision of the Court of Appeals, dismissing the appeal, claiming that the alleged compromise or agreement, dated November 3, 1942, could not
have been understood by Encarnacion Neyra, as she was already then at the threshold of death, and that as a matter of fact she died the following day; and that if it had
been signed at all by said Encarnacion Neyra, her thumbmark appearing on said document must have been affixed thereto by Trinidad Neyra's attorney, against
Encarnacion's will; and that the court had no more jurisdiction over the case, when the alleged agreement was filed on November 4, 1942, at the instance of Trinidad
Neyra, as Encarnacion was already dead at the time.
The principal question to be decided, in connection with said petition for reconsideration, is whether or not said compromise or agreement had been legally executed and
signed by Encarnacion Neyra, on November 3, 1942. Trinidad Neyra maintains the affirmative.
The voluminous evidence, testimonial and documentary, adduced by the parties, in this case, has fully established the following facts:
That Severo Nayra died intestate in the City of Manila, on May 6, 1938, leaving certain properties and two children, by his first marriage, named Encarnacion Neyra and
Trinidad Neyra, and other children by his second marriage; That after the death of Severo Neyra, the two sisters, Encarnacion Neyra and Trinidad Neyra, had serious
misunderstandings, in connection with the properties left by their deceased father, and so serious were their dissensions that, after March 31, 1939, they had two
litigations in the Court of First Instance of Manila, concerning said properties. In the first case, filed in March 31, 1939, Trinidad Neyra and others demanded by
Encarnacion Neyra and others the annulment of the sale of the property located at No. 366 Raon Street, Manila which was finally decided in favor of the defendants, in
the court of first instance, and in the Court of Appeals, on December 21, 1943 (G.R. No. 8162); and the second is the instance case.
That Encarnacion Neyra, who had remained single, and who had no longer any ascendants, executed a will on September 14, 1939, marked Exhibit 16, disposing of her
properties in favor of the "Congregacion de Religiosas de la Virgen Maria" and her other relatives, named Teodora Neyra, Pilar de Guzman and Maria Jacobo Vda. de
Blanco, making no provision whatsoever in said will, in favor of her only sister of the whole blood, Trinidad Neyra, who had become her bitter enemy; that when the said
will was brought to the attention of the authorities of said Congregation, after due deliberation and consideration, said religious organization declined the bounty offered
by Encarnacion Neyra, and said decision of the Congregation was duly communicated to her; that in order to overcome the difficulties encountered by said religious
organization in not accepting the generosity of Encarnacion Neyra, the latter decided to make a new will, and for that purpose, about one week before her death, sent for
Atty. Ricardo Sikat, and gave him instructions for the preparation of a new will; that Atty. Sikat, instead of preparing a new will, merely prepared a draft of a codicil,
amending said will, dated September 14, 1939, again naming said religious organization, among others as beneficiary, and said draft of a codicil was also forwarded to
the authorities of religious organization, for their consideration and acceptance; but it was also rejected.
In the meanwhile, Encarnacion Neyra had become seriously ill, suffering from Addison's disease, and on October 31, 1942, she sent for her religious adviser and
confessor, Mons. Vicente Fernandez of the Quiapo Church to make confession, after which she requested that holy mass be celebrated in her house at No. 366 Raon
Street, City of Manila, so that she might take holy communion; that Mons. Fernandez caused the necessary arrangements to be made, and, as a matter of fact, on
November 1, 1942, holy mass was solemnized in her house by Father Teodoro Garcia, also of the Quiapo Church, on which occasion, Encarnacion Neyra, who remained
in bed, took holy communion; that after the mass, Father Garcia talked to Encarnacion Neyra and advised reconciliation between the two sisters, Encarnacion and
Trinidad Neyra. Encarnacion accepted said advise and, at about noon of the same day (November 1, 1942), sent Eustaquio Mendoza to fetch her sister Trinidad, who
came at about 2:30 that same afternoon; that the two sisters greeted each other in most affectionate manner, and became reconciled and two had a long and cordial
conversation, in the course of which they also talked about the properties left by their father and their litigations which had reached the Court of Appeals for the City of
Manila, the instant case being the second, and they agreed to have the latter dismissed, on the condition that the property involved therein should be given exclusively to
Trinidad Neyra, that the latter should waive her share in the rents of said property collected by Encarnacion, and the Trinidad had no more indebtedness to Encarnacion.
They also agreed to send for Atty. Alejandro M. Panis, to prepare the necessary document embodying the said agreement, but Attorney Panis could come only in the
afternoon of the following day, November 2, 1942, when Encarnacion gave him instructions for the preparation of the document embodying their agreement, and other
instructions for the preparation of her last will and testament; that Attorney Panis prepared said document of compromise as well as the new will and testament, naming
Trinidad Neyra and Eustaquio Mendoza beneficiaries therein, pursuant to Encarnacion's express instructions, and the two documents were prepared, in duplicate, and
were ready for signature, since the morning of November 3, 1942; that in the afternoon of that day, of compromise and last will and testament to Encarnacion Neyra,
slowly and in a loud voice, in the presence of Father Teodoro Garcia, Dr. Moises B. Abad, Dr. Eladio Aldecoa, Trinidad Neyra, and others, after which he asked her if
their terms were in accordance with her wishes, or if she wanted any change made in said documents; that Encarnacion Neyra did not suggest any change, and asked
for the pad and the two documents, and, with the help of a son of Trinidad, placed her thumbmark at the foot of each one of the two documents, in duplicate, on her bed
in the sala, in the presence of attesting witnesses, Dr. Moises B. Abad, Dr. Eladio R. Aldecoa and Atty. Alejandro M. Panis, after which said witnesses signed at the foot
of the will, in the presence of Encarnacion Neyra, and of each other. The agreement was also signed by Trinidad Neyra, as party, and by Dr. M. B. Abad and Eustaquio
Mendoza, a protege, as witnesses.
Father Teodoro Garcia was also present at the signing of the two documents, at the request of Encarnacion Neyra.
The foregoing facts have been established by the witnesses presented by Trinidad Neyra, who are all trustworthy men, and who had absolutely no interest in the final
outcome of this case. Two of them are ministers of the Gospel, while three of the attesting witnesses are professional men of irreproachable character, who had known
and seen and actually talked to the testatrix.
Petitioner Teodora Neyra, half sister of Encarnacion, and her young daughter Ceferina de la Cruz, and Presentacion Blanco, daughter of petitioner Maria Jacobo Vda. de
Blanco, substantially corroborated the testimony of the witnesses presented by Trinidad Neyra, with reference to the signing of documents, in the bedroom of
Encarnacion Neyra, in the afternoon of November 3, 1942.
Teodora Neyra, Presentacion Blanco and Ceferina de la Cruz testified, however, that when the thumbmark of Encarnacion Neyra was affixed to the agreement in
question, dated November 3, 1942, she was sleeping on her bed in the sala; and that the attesting witnesses were not present, as they were in the caida.
But Ceferina de la Cruz also stated that the attesting witnesses signed the documents thumbmarked by Encarnacion Neyra, in the sala near her bed, thus contradicting
herself and Teodora Neyra and Presentacion Blanco.
Strange to say, Teodora Neyra, Presentacion Blanco and Ceferina de la Cruz also testified that Encarnacion Neyra's, thumbmark was affixed to the will, only in the
morning of November 4, 1942, by Trinidad Neyra and one Ildefonso del Barrio, when Encarnacion was already dead.
The testimony of Dr. Dionisio Parulan, alleged medical expert, as to the nature of effects of Addison's disease, is absolutely unreliable. He had never seen or talked to the
testatrix Encarnacion Neyra.
According to medical authorities, persons suffering from Addison's disease often live as long as ten (10) years, while others die after a few weeks only, and that as the
disease progresses, asthenia sets in, and from 80 per cent to 90 per cent of the patients develop tuberculosis, and complications of the heart also appear. (Cecil,
Textbook of Medicine, 3d ed., 1935, pp. 1250-1253; McCrae, Osler's Modern Medicine, 3d ed., Vol. V, pp. 272-279.)
And it has been conclusively shown that Encarnacion Neyra died on November 4, 1942, due to a heart attack, at the age of 48, after an illness of about two (2) years.
In connection with mental capacity, in several cases, this court has considered the testimony of witnesses, who had known and talked to the testators, more trustworthy
than the testimony of the alleged medical experts.
Insomnia, in spite of the testimony of two doctors, who testified for the opponents to the probate of a will, to the effect that it tended to destroy mental capacity, was held
not to effect the full possession of mental faculties deemed necessary and sufficient for its execution. (Caguioa vs. Calderon, 20 Phil., 400.) The testatrix was held to
have been compos mentis, in spite of the physician's testimony to the contrary, to the effect that she was very weak, being in the third or last stage of tuberculosis. (Yap
Tua vs. Yap Ca Kuan and Yap Ca Llu, 27 Phil., 579.) The testimony of the attending physician that the deceased was suffering from diabetes and had been in a
comatose condition for several days, prior to his death, was held not sufficient to establish testamentary incapacity, in view of the positive statement of several credible
witnesses that he was conscious and able to understand what was said to him and to communicate his desires. (Samson vs. Corrales Tan Quintin, 44 Phil., 573.) Where
the mind of the testator is in perfectly sound condition, neither old age, nor ill health, nor the fact that somebody had to guide his hand in order that he might sign, is
sufficient to invalidate his will (Amata and Almojuelavs. Tablizo, 48 Phil., 485.)
Where it appears that a few hours and also a few days after the execution of the will, the testator intelligently and intelligibly conversed with other persons, although lying
down and unable to move or stand up unassisted, but could still effect the sale of property belonging to him, these circumstances show that the testator was in a perfectly
sound mental condition at the time of the execution of the will. (Amata and Almojuela vs. Tablizo, 48 Phil., 485.)
Presentacion Blanco, in the course of her cross-examination, frankly admitted that, in the morning and also at about 6 o'clock in he afternoon of November 3, 1942,
Encarnacion Neyra talked to her that they understood each other clearly, thus showing that the testatrix was really of sound mind, at the time of signing and execution of
the agreement and will in question.
It may, therefore, be reasonably concluded that the mental faculties of persons suffering from Addison's disease, like the testatrix in this case, remain unimpaired, partly
due to the fact that, on account of the sleep they enjoy, they necessarily receive the benefit of physical and mental rest. And that like patients suffering from tuberculosis,
insomnia or diabetes, they preserve their mental faculties until the moments of their death.
Judging by the authorities above cited, the logical conclusion is that Encarnacion Neyra was of sound mind and possessed the necessary testamentary and mental
capacity, at the time of the execution of the agreement and will, dated November 3, 1942. The contention that the attesting witnesses were not present, at the time
Encarnacion Neyra thumbmarked the agreement and will in question, on her bed, in the sala of the house, as they were allegedly in the caida, is untenable. It has been
fully shown that said witnesses were present, at the time of the signing and execution of the agreement and will in question, in the sala, where the testatrix was lying on
her bed. The true test is not whether they actually saw each other at the time of the signing of the documents, but whether they might have seen each other sign, had
they chosen to do so; and the attesting witnesses actually saw it all in this case. (Jaboneta vs.Gustilo, 5 Phil., 541.) And the thumbmark placed by the testatrix on the
agreement and will in question is equivalent to her signature. (Yap Tua vs. Yap Ca Kuan and Yap Ca Llu, 27 Phil., 579.)
Teodora Neyra and her principal witnesses are all interested parties, as they are children of legatees named in the will, dated September 14, 1939, but eliminated from
the will, dated November 3, 1942. Furthermore, the testimony of Teodora Neyra and her witnesses, to the effect that there could have been no reconciliation between the
two sisters, and that the thumbmark of Encarnacion Neyra was affixed to the documents embodying the agreement, while she was sleeping, on November 3, 1942, in
their presence; and that her thumbmark was affixed to the will in question, when she was already dead, in the morning of November 4, 1942, within their view, is
absolutely devoid of any semblance of truth. Said testimony is contrary to common sense. It violates all sense of proportion. Teodora Neyra and her witnesses could not
have told the truth; they have testified to deliberate falsefoods; and they are, therefore, absolutely unworthy of belief. And to the evidence of the petitioners is completely
applicable the legal aphorism falsus in uno, falsus in omnibus. (Gonzales vs. Mauricio, 53 Phil., 728, 735.)
To show the alleged improbability of reconciliation, and the execution of the two documents, dated November 3, 1942, petitioners have erroneously placed great
emphasis on the fact that, up to October 31, 1942, the two sisters Encarnacion and Trinidad Neyra were bitter enemies. They were banking evidently on the common
belief that the hatred of relatives is the most violent. Terrible indeed are the feuds of relatives and difficult the reconciliation; and yet not impossible. They had forgotten
that Encarnacion Neyra was a religious woman instructed in the ancient virtues of the Christian faith, and hope and charity, and that to forgive is a divine attribute. They
had also forgotten that there could be no more sublime love than that embalmed in tears, as in the case of a reconciliation. It was most natural that there should have
been reconciliation between the two sisters, Encarnacion and Trinidad Neyra, as the latter is the nearest relative of the former, her only sister of the whole blood. The
approach of imminent death must have evoked in her the tenderest recollections of family life. And believing perhaps that her little triumphs had not always brought her
happiness, and that she had always been just to her sister, who had been demanding insistently what was her due, Encarnacion finally decided upon reconciliation, as
she did not want to go to her eternal rest, with hatred in her heart or wrath upon her head. It was, therefore, most logical that Encarnacion should make Trinidad the
benificiary of her generosity, under her last will and testament, and end all her troubles with her, by executing said agreement, and thus depart in perfect peace from the
scenes of her earthly labors. It having been shown that the said compromise or agreement had been legally signed and executed by Encarnacion Neyra on November 3,
1942, in the presence of credible and trustworthy witnesses, and that she wascompos mentis and possessed the necessary testamentary and mental capacity of the
time; the petition for the reconsideration filed by Atty. Lucio Javillonar, on November 23, 1942, on behalf of a client, Encarnacion Neyra, who had been dead since
November 4, 1942, and some of her relatives, who have appeared, in accordance with the provisions of section 17 of Rule 3 of the Rules of Court, is hereby denied; and
the decision of the Court of Appeals for Manila, dated November 10, 1942, dismissing the appeal, is hereby re-affirmed, without costs. So ordered.


Republic of the Philippines
SUPREME COURT
Manila

EN BANC

October 18, 1939

G.R. No. 46097
TEOFILA ADEVA VIUDA DE LEYNEZ, petitioner,
vs.
IGNACIO LEYNEZ, respondent.

Conrado V. Sanchez and Ambrosio Padilla for petitioner.
Ilao and Enriquez for respondent.

LAUREL, J.:

This is a petition for a writ of certiorari to review the decision of the Court of Appeals affirming the decision of the Court of First Instance of Mindoro denying probate
of the will of the deceased Valerio Leynez, on the ground that its attestation clause does not conform to the requirements section 618, as amended, of the Code of Civil
Procedure.

The attestation clause of the will is worded as follows:

Suscrito y declarado por el testador Valerio Leynez, como su ultima voluntad y testamento en presencia de todos y cada uno de nosotros, y a ruego de dicho testador,
firmamos el presente cada uno en presencia de los otros, o de los demas y de la del mismo testador Valerio Leynez . El testamento consta de los (2) paginas solamente.

The question presented is, under section 618, as amended, of the Code of Civil Procedure, is this attestation clause legally sufficient? The pertinent portion of this
section of the Code is as follows:

. . . the attestation shall state the number of sheets or pages used, upon which the will is written, and the fact that the testator signed the will and every page thereof, or
caused some other person to write his name, under his express direction, in the presence of three witnesses, an the later witnessed and signed the will and all pages
thereof in the presence of the testator and of each other.

The alleged defect in the attestation clause of the controverted will is that it fails to state that the testator and the three witnesses signed each and every page of the will
in the manner prescribed by law, because it merely states "firmanos el presente cada uno en presencia de los otros, o de los demas y de la del mismo testador Valerio
Leynez." In deciding this question the Court of Appeals, however, ruled:

A la luz de las jurisprudencias arriba citadas en la clausada de atestiguamiento discutida en el asunto de autos no encontramos un cumplimiento sustantial del requisito
exigido por la ley, de que en ella se haga constar que el testador y los testigos han firmado unos en presencia de otros, todas y cada una de las paginas usadas del
testamento, requisito que no se puede establecer por medio de su prueba aliunde.

Against this conclusion of the Court of Appeals, petitioner puts forward the contention that it has decided a question of substance in a way not probably in accord with
the law and the applicable decisions of this court (Rule 47, paragraph e [1] of Supreme Court.) The rule of liberal construction of the applicable law should, petitioner
avers, be held to apply in the case at bar, and in support of her content on she invokes a long array of cases (Abangan vs. Abangan, 40 Phil., 476; Avera vs. Garcia and
Rodriguez, 42 Phil., 145; Aldaba vs. Roque, 43 Phil., 378; Unson vs. Abella, 43 Phil., 494; Fernandez vs. Vergel de Dios, 46 Phil., 922; Nayve vs. Mojal, 47 Phil., 152;
De Gala vs. Gonzalez, 53 Phil., 104; Rey vs. Cartagena, 56 Phil., 282; Dichoso de Ticson vs. De Gorostiza, 57 Phil., 437; Sebastian vs. Paganiban, 59 Phil., 653; De
Guzman vs. Celestino, G.R. No. 35273, April 25, 1932; Policarpio vs. Baltazar, G.R. No. 36349, November 14, 1932; Malate vs. Olea, G.R. No. 36154, December 16,
1932; In re Estate of Jennings, 1933, G.R. No. 38758). To this line of cases those of Rodriguez vs. Yap, G.R. No. 45924, May 18, 1939, and Grey vs. Fabie, G.R. No.
45160, May 23, 1939, may perhaps be added. Respondent, on the other hand, equally invokes a number of cases wherein, he contends, the rule of strict construction
was made to prevail. (Uy Coque vs. Navas L. Sioca, 43 Phil., 405; In re Estate of Neuark, 46 Phil., 841; Sao vs. Quintana, 48 Phil., 506; Gumban vs. Gorecho, 50
Phil., 30; Quinto vs. Morata, 54 Phil., 481; Rodriguez vs. Alcala, 55 Phil., 150.)

This Court has already taken notice of these different views within, in Dichoso de Ticson vs. De Gorostiza (57 Phil., 437, 439-440), it frankly made the following
observation : "The truth is that there have been, noticeable in the Philippines two divergent tendencies in the law of wills the one being planted on strict construction
and the other on liberal construction. A late example of the former views may be found in the decision in Rodriguez vs. Alcala ([1930], 55 Phil., 150), sanctioning a
literal enforcement of the law. The basic case in the other direction, predicated on reason, is Abangan vs. Abangan ([1919], 40 Phil., 476), oft-cited approvingly in later
decisions." It is fairness to recognize the existence of opposing currents of legal thought, a situation which perhaps has brought about a certain degree of confusion in
this field. It is also fairness to avow, however, that a more careful examination of the cases will show that, while the two tendencies mentioned in easily discernible, the
conflict in many cases is more apparent than real, and the variance, if at all, in the application of the principles involved was due in some instances to the marked
differentiation of facts and the consequent personal or collective criteria in particular cases.

We have taken pains to examine the numerous cases relied upon by the petitioner and those relied upon by the respondent, and while we do not deem it necessary to
make a detailed comparison between them, we find no difficulty in selecting what we consider is the reasonable rule to apply in this case at bar. It is, of course, not
possible to lay down a general rule, rigid and inflexible, which would be applicable to all cases. More than anything else, the facts and circumstances of record are to be
considered in the application of any given rule. If the surrounding circumstances point to a regular execution of the will, and the instrument appears to have been
executed substantially in accordance with the requirements of the law, the inclination should, in the absence of any suggestion of bad faith, forgery or fraud, lean
towards its admission to probate, although the document may suffer from some imperfection of language, or other non-essential defect. This, in our opinion, is the
situation in the present case, and we, therefore, hold that the requirement that the attestation clause, among other things, shall state "that the testator signed the will and
every page thereof in the presence of three witnesses, and that the witnesses signed the will in the presence of the testator and of each other," is sufficiently complied
with, it appearing that the testator and the witnesses signed each and every page of the will according to the stipulation of the parties. (Record on Appeal, stipulation,
pp. 10, 14, 15); and this fact being shown in the will itself, and there being, furthermore, no question raised as to the authenticity of the signature of the testator and the
witnesses.

An attestation clause is made for the purpose of preserving, in permanent form, a record of the facts attending the execution of the will, so that in case of failure of the
memory of the subscribing witnesses, or other casualty, they may still be proved. (Thompson on Wills, 2 ed., sec. 132.) A will, therefore, should not be rejected where
its attestation clause serves the purpose of the law. The law-making body, in recognition of the dangers to which testamentary dispositions are apt to be subject in the
hands of unscrupulous individuals, has surrounded the execution of the wills with every solemnity deemed necessary to safeguard it. This purpose was indicated when
our legislature provided for the exclusion of evidence aliunde to prove the execution of the will. We should not, however, attribute the prohibition as indicative of a
desire to impose unreasonable restraint or beyond what reason and justice permit. It could not have been the intention of the legislature in providing for the essential
safeguards in the execution of a will to shackle the very right of testamentary disposition which the law recognizes and holds sacred. The pronouncement of this Court
in Abangan vs. Abangan (40 Phil., 476, 479), expresses the sound rule to which we have recently adhered in principle. (Rodriguez vs. Yap, G.R. No. 45924,
promulgated May 18, 1939; and Grey vs. Fabie, G.R. No. 45160, promulgated May 23, 1939):

The object of the solemnities surrounding the execution of wills is to close the door against bad faith and fraud, to avoid substitution of wills and testaments and to
guaranty their truth and authenticity. Therefore the laws on this subject should be interpreted in such a way as to attain these primordial ends. But, on the other hand,
also one must not lose sight of the fact that it is not the object of the law to restrain and curtail the exercise of the right to make a will. So when an interpretation already
given assures such ends, any other interpretation whatsoever, that adds nothing but demands more requisites entirely unnecessary, useless and frustrative of the testator's
last will, must be disregarded.

It follows that the writ of certiorari should be, as it is hereby, granted and the judgment of the Court of Appeals reversed, with the result that the controverted will,
Exhibit A, of the deceased Valerio Leynez, shall be admitted to probate. So ordered, with costs against the respondent-appellee. So ordered.
Guerrero vs BihisG.R. No. 174144 April 17, 2007Facts:
Felisa Tamio de Buenaventura, mother of petitioner Bella A. Guerrero andrespondent Resurreccion A. Bihis, died. Guerrero filed for probate in the RTCQC.
Respondent Bihis
opposed her elder sisters petition on the following
grounds: the will was not executed and attested as required by law; itsattestation clause and acknowledgment did not comply with the requirementsof the law; the
signature of the testatrix was procured by fraud and petitioner and her children procured the will through undue and improper pressure andinfluence. The trial court
denied the probate of the will ruling that Article 806 of
the Civil Code was not complied with because the will was acknowledged bythe testatrix and the witnesses at the testatrixs residence at No. 40 Kanlaon
Street, Quezon City before Atty. Macario O. Directo who was a commissionednotary public for and in Caloocan City.
ISSUE:

Did the will acknowledged by the testatrix and t
he instrumental witnessesbefore a notary public acting outside the place of his commission satisfy therequirement under Article 806 of the Civil Code?
HELD:
No. One of the formalities required by law in connection with the execution of a notarial will is that it must be acknowledged before a notary public by thetestator and
the witnesses. This formal requirement is one of theindispensable requisites for the validity of a will. In other words, a notarial willthat is not acknowledged before a
notary public by the testator and theinstrumental witnesses is void and cannot be accepted for probate.The Notarial law provides: SECTION 240.Territorial jurisdiction.

The jurisdiction of a notary public in a province shall be co-extensive with theprovince. The jurisdiction of a notary public in the City of Manila shall be co-extensive
with said city. No notary shall possess authority to do any notarialact beyond the limits of his jurisdiction.Sine Atty. Directo was not a commissioned notary public for
and in QuezonCity, he lacked the authority to take the acknowledgment of the testratix andthe instrumental witnesses. In the same vain, the testratix and theinstrumental
witnesses could not have validly acknowledged the will beforehim. Thus, Felisa Tamio de Bu
enaventuras last will and testament was, in
effect, not acknowledged as required by law.






Republic of the Philippines
SUPREME COURT
Manila
EN BANC
G.R. No. L-7188 August 9, 1954
In re: Will and Testament of the deceased REVEREND SANCHO ABADIA.
SEVERINA A. VDA. DE ENRIQUEZ, ET AL., petitioners-appellees,
vs.
MIGUEL ABADIA, ET AL., oppositors-appellants.
Manuel A. Zosa, Luis B. Ladonga, Mariano A. Zosa and B. G. Advincula for appellants.
C. de la Victoria for appellees.
MONTEMAYOR, J .:
On September 6, 1923, Father Sancho Abadia, parish priest of Talisay, Cebu, executed a document purporting to be his Last Will and Testament now marked Exhibit
"A". Resident of the City of Cebu, he died on January 14, 1943, in the municipality of Aloguinsan, Cebu, where he was an evacuee. He left properties estimated at P8,000
in value. On October 2, 1946, one Andres Enriquez, one of the legatees in Exhibit "A", filed a petition for its probate in the Court of First Instance of Cebu. Some cousins
and nephews who would inherit the estate of the deceased if he left no will, filed opposition.
During the hearing one of the attesting witnesses, the other two being dead, testified without contradiction that in his presence and in the presence of his co-witnesses,
Father Sancho wrote out in longhand Exhibit "A" in Spanish which the testator spoke and understood; that he (testator) signed on he left hand margin of the front page of
each of the three folios or sheets of which the document is composed, and numbered the same with Arabic numerals, and finally signed his name at the end of his writing
at the last page, all this, in the presence of the three attesting witnesses after telling that it was his last will and that the said three witnesses signed their names on the
last page after the attestation clause in his presence and in the presence of each other. The oppositors did not submit any evidence.
The learned trial court found and declared Exhibit "A" to be a holographic will; that it was in the handwriting of the testator and that although at the time it was executed
and at the time of the testator's death, holographic wills were not permitted by law still, because at the time of the hearing and when the case was to be decided the new
Civil Code was already in force, which Code permitted the execution of holographic wills, under a liberal view, and to carry out the intention of the testator which
according to the trial court is the controlling factor and may override any defect in form, said trial court by order dated January 24, 1952, admitted to probate Exhibit "A",
as the Last Will and Testament of Father Sancho Abadia. The oppositors are appealing from that decision; and because only questions of law are involved in the appeal,
the case was certified to us by the Court of Appeals.
The new Civil Code (Republic Act No. 386) under article 810 thereof provides that a person may execute a holographic will which must be entirely written, dated and
signed by the testator himself and need not be witnessed. It is a fact, however, that at the time that Exhibit "A" was executed in 1923 and at the time that Father Abadia
died in 1943, holographic wills were not permitted, and the law at the time imposed certain requirements for the execution of wills, such as numbering correlatively each
page (not folio or sheet) in letters and signing on the left hand margin by the testator and by the three attesting witnesses, requirements which were not complied with in
Exhibit "A" because the back pages of the first two folios of the will were not signed by any one, not even by the testator and were not numbered, and as to the three front
pages, they were signed only by the testator.
Interpreting and applying this requirement this Court in the case of In re Estate of Saguinsin, 41 Phil., 875, 879, referring to the failure of the testator and his witnesses to
sign on the left hand margin of every page, said:
. . . . This defect is radical and totally vitiates the testament. It is not enough that the signatures guaranteeing authenticity should appear upon two
folios or leaves; three pages having been written on, the authenticity of all three of them should be guaranteed by the signature of the alleged
testatrix and her witnesses.
And in the case of Aspe vs. Prieto, 46 Phil., 700, referring to the same requirement, this Court declared:
From an examination of the document in question, it appears that the left margins of the six pages of the document are signed only by Ventura
Prieto. The noncompliance with section 2 of Act No. 2645 by the attesting witnesses who omitted to sign with the testator at the left margin of each of
the five pages of the document alleged to be the will of Ventura Prieto, is a fatal defect that constitutes an obstacle to its probate.
What is the law to apply to the probate of Exh. "A"? May we apply the provisions of the new Civil Code which not allows holographic wills, like Exhibit "A" which provisions
were invoked by the appellee-petitioner and applied by the lower court? But article 795 of this same new Civil Code expressly provides: "The validity of a will as to its
form depends upon the observance of the law in force at the time it is made." The above provision is but an expression or statement of the weight of authority to the affect
that the validity of a will is to be judged not by the law enforce at the time of the testator's death or at the time the supposed will is presented in court for probate or when
the petition is decided by the court but at the time the instrument was executed. One reason in support of the rule is that although the will operates upon and after the
death of the testator, the wishes of the testator about the disposition of his estate among his heirs and among the legatees is given solemn expression at the time the will
is executed, and in reality, the legacy or bequest then becomes a completed act. This ruling has been laid down by this court in the case of In re Will of Riosa, 39 Phil.,
23. It is a wholesome doctrine and should be followed.
Of course, there is the view that the intention of the testator should be the ruling and controlling factor and that all adequate remedies and interpretations should be
resorted to in order to carry out said intention, and that when statutes passed after the execution of the will and after the death of the testator lessen the formalities
required by law for the execution of wills, said subsequent statutes should be applied so as to validate wills defectively executed according to the law in force at the time
of execution. However, we should not forget that from the day of the death of the testator, if he leaves a will, the title of the legatees and devisees under it becomes a
vested right, protected under the due process clause of the constitution against a subsequent change in the statute adding new legal requirements of execution of wills
which would invalidate such a will. By parity of reasoning, when one executes a will which is invalid for failure to observe and follow the legal requirements at the time of
its execution then upon his death he should be regarded and declared as having died intestate, and his heirs will then inherit by intestate succession, and no subsequent
law with more liberal requirements or which dispenses with such requirements as to execution should be allowed to validate a defective will and thereby divest the heirs
of their vested rights in the estate by intestate succession. The general rule is that the Legislature can not validate void wills (57 Am. Jur., Wills, Sec. 231, pp. 192-193).In
view of the foregoing, the order appealed from is reversed, and Exhibit "A" is denied probate. With costs.


Republic of the Philippines
SUPREME COURT
Manila

FIRST DIVISION

G.R. No. L-58509 December 7, 1982

IN THE MATTER OF THE PETITION TO APPROVE THE WILL OF RICARDO B. BONILLA deceased, MARCELA RODELAS, petitioner-appellant,
vs.
AMPARO ARANZA, ET AL., oppositors-appellees, ATTY. LORENZO SUMULONG, intervenor.

Luciano A. Joson for petitioner-appellant.

Cesar Paralejo for oppositor-appellee.



RELOVA, J.:
This case was certified to this Tribunal by the Court of Appeals for final determination pursuant to Section 3, Rule 50 of the Rules of Court.
As found by the Court of Appeals:

... On January 11, 1977, appellant filed a petition with the Court of First Instance of Rizal for the probate of the holographic will of Ricardo B. Bonilla and the issuance
of letters testamentary in her favor. The petition, docketed as Sp. Proc. No. 8432, was opposed by the appellees Amparo Aranza Bonilla, Wilferine Bonilla Treyes
Expedita Bonilla Frias and Ephraim Bonilla on the following grounds:

(1) Appellant was estopped from claiming that the deceased left a will by failing to produce the will within twenty days of the death of the testator as required by
Rule 75, section 2 of the Rules of Court;

(2) The alleged copy of the alleged holographic will did not contain a disposition of property after death and was not intended to take effect after death, and
therefore it was not a will

(3) The alleged hollographic will itself,and not an alleged copy thereof, must be produced, otherwise it would produce no effect, as held in Gam v. Yap, 104 Phil. 509;
and

(4 ) The deceased did not leave any will, holographic or otherwise, executed and attested as required by law.

The appellees likewise moved for the consolidation of the case with another case Sp. Proc. No, 8275). Their motion was granted by the court in an order dated April 4,
1977.

On November 13, 1978, following the consolidation of the cases, the appellees moved again to dismiss the petition for the probate of the will. They argued that:

(1) The alleged holographic was not a last will but merely an instruction as to the management and improvement of the schools and colleges founded by
decedent Ricardo B. Bonilla; and

(2) Lost or destroyed holographic wills cannot be proved by secondary evidence unlike ordinary wills.

Upon opposition of the appellant, the motion to dismiss was denied by the court in its order of February 23, 1979.

The appellees then filed a motion for reconsideration on the ground that the order was contrary to law and settled pronouncements and rulings of the Supreme
Court, to which the appellant in turn filed an opposition. On July 23, 1979, the court set aside its order of February 23, 1979 and dismissed the petition for the
probate of the will of Ricardo B. Bonilla. The court said:

... It is our considered opinion that once the original copy of the holographic will is lost, a copy thereof cannot stand in lieu of the original.

In the case of Gam vs. Yap, 104 Phil. 509, 522, the Supreme Court held that 'in the matter of holographic wills the law, it is reasonable to suppose, regards the
document itself as the material proof of authenticity of said wills.

MOREOVER, this Court notes that the alleged holographic will was executed on January 25, 1962 while Ricardo B. Bonilla died on May 13, 1976. In view of the lapse of
more than 14 years from the time of the execution of the will to the death of the decedent, the fact that the original of the will could not be located shows to our
mind that the decedent had discarded before his death his allegedly missing Holographic Will.

Appellant's motion for reconsideration was denied. Hence, an appeal to the Court of Appeals in which it is contended that the dismissal of appellant's petition is
contrary to law and well-settled jurisprudence.

On July 7, 1980, appellees moved to forward the case to this Court on the ground that the appeal does not involve question of fact and alleged that the trial court
committed the following assigned errors:

I. THE LOWER COURT ERRED IN HOLDING THAT A LOST HOLOGRAPHIC WILL MAY NOT BE PROVED BY A COPY THEREOF;

II. THE LOWER COURT ERRED IN HOLDING THAT THE DECEDENT HAS DISCARDED BEFORE HIS DEATH THE MISSING HOLOGRAPHIC WILL;

III. THE LOWER COURT ERRED IN DISMISSING APPELLANT'S WILL.

The only question here is whether a holographic will which was lost or cannot be found can be proved by means of a photostatic copy. Pursuant to Article 811 of the
Civil Code, probate of holographic wills is the allowance of the will by the court after its due execution has been proved. The probate may be uncontested or not. If
uncontested, at least one Identifying witness is required and, if no witness is available, experts may be resorted to. If contested, at least three Identifying witnesses
are required. However, if the holographic will has been lost or destroyed and no other copy is available, the will can not be probated because the best and only
evidence is the handwriting of the testator in said will. It is necessary that there be a comparison between sample handwritten statements of the testator and the
handwritten will. But, a photostatic copy or xerox copy of the holographic will may be allowed because comparison can be made with the standard writings of the
testator. In the case of Gam vs. Yap, 104 PHIL. 509, the Court ruled that "the execution and the contents of a lost or destroyed holographic will may not be proved by
the bare testimony of witnesses who have seen and/or read such will. The will itself must be presented; otherwise, it shall produce no effect. The law regards the
document itself as material proof of authenticity." But, in Footnote 8 of said decision, it says that "Perhaps it may be proved by a photographic or photostatic copy.
Even a mimeographed or carbon copy; or by other similar means, if any, whereby the authenticity of the handwriting of the deceased may be exhibited and tested
before the probate court," Evidently, the photostatic or xerox copy of the lost or destroyed holographic will may be admitted because then the authenticity of the
handwriting of the deceased can be determined by the probate court.

WHEREFORE, the order of the lower court dated October 3, 1979, denying appellant's motion for reconsideration dated August 9, 1979, of the Order dated July 23,
1979, dismissing her petition to approve the will of the late Ricardo B. Bonilla, is hereby SET ASIDE. SO ORDERED.



Republic of the Philippines
SUPREME COURT
Manila

FIRST DIVISION



G.R. No. L-32213 November 26, 1973

AGAPITA N. CRUZ, petitioner,
vs.
HON. JUDGE GUILLERMO P. VILLASOR, Presiding Judge of Branch I, Court of First Instance of Cebu, and MANUEL B. LUGAY, respondents.

Paul G. Gorrez for petitioner.

Mario D. Ortiz for respondent Manuel B. Lugay.



ESGUERRA, J.:

Petition to review on certiorari the judgment of the Court First Instance of Cebu allowing the probate of the last will a testament of the late Valente Z. Cruz.
Petitioner-appellant Agapita N. Cruz, the surviving spouse of the said decease opposed the allowance of the will (Exhibit "E"), alleging the will was executed through
fraud, deceit, misrepresentation and undue influence; that the said instrument was execute without the testator having been fully informed of the content thereof,
particularly as to what properties he was disposing and that the supposed last will and testament was not executed in accordance with law. Notwithstanding her
objection, the Court allowed the probate of the said last will and testament Hence this appeal by certiorari which was given due course.

The only question presented for determination, on which the decision of the case hinges, is whether the supposed last will and testament of Valente Z. Cruz (Exhibit
"E") was executed in accordance with law, particularly Articles 805 and 806 of the new Civil Code, the first requiring at least three credible witnesses to attest and
subscribe to the will, and the second requiring the testator and the witnesses to acknowledge the will before a notary public.

Of the three instrumental witnesses thereto, namely Deogracias T. Jamaloas Jr., Dr. Francisco Paares and Atty. Angel H. Teves, Jr., one of them, the last named, is at
the same time the Notary Public before whom the will was supposed to have been acknowledged. Reduced to simpler terms, the question was attested and
subscribed by at least three credible witnesses in the presence of the testator and of each other, considering that the three attesting witnesses must appear before
the notary public to acknowledge the same. As the third witness is the notary public himself, petitioner argues that the result is that only two witnesses appeared
before the notary public to acknowledge the will. On the other hand, private respondent-appellee, Manuel B. Lugay, who is the supposed executor of the will,
following the reasoning of the trial court, maintains that there is substantial compliance with the legal requirement of having at least three attesting witnesses even if
the notary public acted as one of them, bolstering up his stand with 57 American Jurisprudence, p. 227 which, insofar as pertinent, reads as follows:

It is said that there are, practical reasons for upholding a will as against the purely technical reason that one of the witnesses required by law signed as certifying to an
acknowledgment of the testator's signature under oath rather than as attesting the execution of the instrument. After weighing the merits of the conflicting claims of
the parties, We are inclined to sustain that of the appellant that the last will and testament in question was not executed in accordance with law. The notary public
before whom the will was acknowledged cannot be considered as the third instrumental witness since he cannot acknowledge before himself his having signed the
will. To acknowledge before means to avow (Javellana v. Ledesma, 97 Phil. 258, 262; Castro v. Castro, 100 Phil. 239, 247); to own as genuine, to assent, to admit; and
"before" means in front or preceding in space or ahead of. (The New Webster Encyclopedic Dictionary of the English Language, p. 72; Funk & Wagnalls New Standard
Dictionary of the English Language, p. 252; Webster's New International Dictionary 2d. p. 245.) Consequently, if the third witness were the notary public himself, he
would have to avow assent, or admit his having signed the will in front of himself. This cannot be done because he cannot split his personality into two so that one
will appear before the other to acknowledge his participation in the making of the will. To permit such a situation to obtain would be sanctioning a sheer absurdity.

Furthermore, the function of a notary public is, among others, to guard against any illegal or immoral arrangement Balinon v. De Leon, 50 0. G. 583.) That function
would defeated if the notary public were one of the attesting instrumental witnesses. For them he would be interested sustaining the validity of the will as it directly
involves him and the validity of his own act. It would place him in inconsistent position and the very purpose of acknowledgment, which is to minimize fraud (Report
of Code Commission p. 106-107), would be thwarted. Admittedly, there are American precedents holding that notary public may, in addition, act as a witness to the
executive of the document he has notarized. (Mahilum v. Court Appeals, 64 0. G. 4017; 17 SCRA 482; Sawyer v. Cox, 43 Ill. 130). There are others holding that his
signing merely as notary in a will nonetheless makes him a witness thereon (Ferguson v. Ferguson, 47 S. E. 2d. 346; In Re Douglas Will, N. Y. S. 2d. 641; Ragsdal v. Hill,
269 S. W. 2d. 911, Tyson Utterback, 122 So. 496; In Re Baybee's Estate 160 N. 900; W. Merill v. Boal, 132 A. 721; See also Trenwith v. Smallwood, 15 So. 1030). But
these authorities do not serve the purpose of the law in this jurisdiction or are not decisive of the issue herein because the notaries public and witnesses referred to
aforecited cases merely acted as instrumental, subscribing attesting witnesses, and not as acknowledging witnesses. He the notary public acted not only as attesting
witness but also acknowledging witness, a situation not envisaged by Article 805 of the Civil Code which reads:
ART. 806. Every will must be acknowledged before a notary public by the testator and the witnesses. The notary public shall not be required to retain a copy of the
will or file another with the office of the Clerk of Court. [Emphasis supplied]
To allow the notary public to act as third witness, or one the attesting and acknowledging witnesses, would have the effect of having only two attesting witnesses to
the will which would be in contravention of the provisions of Article 80 be requiring at least three credible witnesses to act as such and of Article 806 which requires
that the testator and the required number of witnesses must appear before the notary public to acknowledge the will. The result would be, as has been said, that
only two witnesses appeared before the notary public for or that purpose. In the circumstances, the law would not be duly in observed.

FOR ALL THE FOREGOING, the judgment appealed from is hereby reversed and the probate of the last will and testament of Valente Z. Cruz (Exhibit "E") is declared
not valid and hereby set aside. Cost against the appellee.
Republic of the Philippines
SUPREME COURT
Manila

EN BANC

G.R. No. L-2538 September 21, 1951

Testate Estate of the Deceased MARIANO MOLO Y LEGASPI. JUANA JUAN VDA. DE MOLO, petitioner-appellee,
vs.
LUZ, GLICERIA and CORNELIO MOLO, oppositors-appellants.

Claro M. Recto and Serafin C. Dizon for appellants.
Delgado & Flores for appellee.

BAUTISTA ANGELO, J.:

This is an appeal from an order of the Court of First Instance of Rizal admitting to probate the last will and testament of the deceased Mariano Molo y Legaspi
executed on August 17, 1918. The oppositors-appellants brought the case on appeal to this Court for the reason that the value of the properties involved exceeds
P50,000.

Mariano Molo y Legaspi died on January 24, 1941, in the municipality of Pasay, province of Rizal, without leaving any forced heir either in the descending or ascending
line. He was survived, however, by his wife, the herein petitioner Juana Juan Vda. de Molo, and by his nieces and nephew, the oppositors-appellants, Luz Gliceria and
Cornelio, all surnamed Molo, who were the legitimate children of Candido Molo y Legaspi, deceased brother of the testator. Mariano Molo y Legaspi left two wills,
one executed on August 17, 1918, (Exhibit A) and another executed on June 20, 1939. (Exhibit I). The later will executed in 1918.

On February 7, 1941, Juana Juan Vda. de Molo, filed in the Court of First Instance of Rizal a petition, which was docketed as special proceeding No. 8022 seeking the
probate of the will executed by the deceased on June 20, 1939. There being no opposition, the will was probated. However, upon petition filed by the herein
oppositors, the order of the court admitting the will to probate was set aside and the case was reopened. After hearing, at which both parties presented their
evidence, the court rendered decision denying the probate of said will on the ground that the petitioner failed to prove that the same was executed in accordance
with law.

In view of the disallowance of the will executed on June 20, 1939, the widow on February 24, 1944, filed another petition for the probate of the will executed by the
deceased on August 17, 1918, which was docketed as special proceeding No. 56, in the same court. Again, the same oppositors filed an opposition to the petition
based on three grounds: (1) that petitioner is now estopped from seeking the probate of the will of 1918; (2) that said will has not been executed in the manner
required by law and (3) that the will has been subsequently revoked. But before the second petition could be heard, the battle for liberation came and the records of
the case were destroyed. Consequently, a petition for reconstitution was filed, but the same was found to be impossible because neither petitioner nor oppositors
could produce the copies required for its reconstitution. As a result, petitioner filed a new petition on September 14, 1946, similar to the one destroyed, to which the
oppositors filed an opposition based on the same grounds as those contained in their former opposition. Then, the case was set for trial, and on May 28, 1948, the
court issued an order admitting the will to probate already stated in the early part of this decision. From this order the oppositors appealed assigning six errors, to
wit.

I. The probate court erred in not holding that the present petitioner voluntarily and deliberately frustrated the probate of the will dated June 20, 1939, in
special proceeding No. 8022, in order to enable her to obtain the probate of another alleged will of Molo dated 191.

II. The court a quo erred in not holding that the petitioner is now estopped from seeking the probate of Molo's alleged will of 1918.

III. The lower court erred in not holding that petitioner herein has come to court with "unclean hands" and as such is not entitled to relief.

IV. The probate court erred in not holding that Molo's alleged will of August 17, 1918 was not executed in the manner required by law.

V. The probate court erred in not holding that the alleged will of 1918 was deliberately revoked by Molo himself.

VI. The lower court erred in not holding that Molo's will of 1918 was subsequently revoked by the decedent's will of 1939.

In their first assignment of error, counsel for oppositors contend that the probate court erred in not holding that the petitioner voluntarily and deliberately frustrated
the probate of the will dated June 20, 1939, in order to enable her to obtain the probate of the will executed by the deceased on August 17, 1918, pointing out
certain facts and circumstances with their opinion indicate that petitioner connived with the witness Canuto Perez in an effort to defeat and frustrate the probate of
the 1939 will because of her knowledge that said will intrinsically defective in that "the one and only testamentory disposition thereof was a "disposicion captatoria".
These circumstances, counsel for the appellants contend, constitute a series of steps deliberately taken by petitioner with a view to insuring the realization of her
plan of securing the probate of the 1918 will which she believed would better safeguard her right to inherit from the decease.

These imputations of fraud and bad faith allegedly committed in connection with special proceedings No. 8022, now closed and terminated, are vigorously met by
counsel for petitioner who contends that to raise them in these proceedings which are entirely new and distinct and completely independent from the other is
improper and unfair as they find no support whatsoever in any evidence submitted by the parties in this case. They are merely based on the presumptions and
conjectures not supported by any proof. For this reason, counsel, contends, the lower court was justified in disregarding them and in passing them sub silentio in its
decision.

A careful examination of the evidence available in this case seems to justify this contention. There is indeed no evidence which may justify the insinuation that
petitioner had deliberately intended to frustrate the probate of the 1939 will of the deceased to enable her to seek the probate of another will other than a mere
conjecture drawn from the apparently unexpected testimony of Canuto Perez that he went out of the room to answer an urgent call of nature when Artemio Reyes
was signing the will and the failure of petitioner later to impeach the character of said witness in spite of the opportunity given her by the court to do so. Apart from
this insufficiency of evidence, the record discloses that this failure has been explained by petitioner when she informed the court that she was unable to impeach the
character of her witness Canuto Perez because of her inability to find witnesses who may impeach him, and this explanation stands uncontradicted. Whether this
explanation is satisfactory or not, it is not now, for us to determine. It is an incident that comes within the province of the former case. The failure of petitioner to
present the testimony of Artemio Reyes at the hearing has also been explained, and it appears that petitioner has filed because his whereabouts could not be found.
Whether this is true or not is also for this Court to determine. It is likewise within the province and function of the court in the former case. And the unfairness of this
imputation becomes more glaring when we stock of the developments that had taken place in these proceedings which show in bold relief the true nature of the
conduct, behavior and character of the petitioner so bitterly assailed and held in disrepute by the oppositors.

It should be recalled that the first petition for the probate of the will executed on June 20, 1939, was filed on February 7, 1941, by the petitioner. There being no
opposition, the will was probated. Subsequently, however, upon petition of the herein oppositors, the order of the court admitting said will to probate was set aside,
over the vigorous opposition of the herein petitioner, and the case was reopened. The reopening was ordered because of the strong opposition of the oppositors who
contended that he will had not been executed as required by law. After the evidence of both parties had been presented, the oppositors filed an extensive
memorandum wherein they reiterated their view that the will should be denied probate. And on the strenght of this opposition, the court disallowed the will.

If petitioner then knew that the 1939 will was inherently defective and would make the testamentary disposition in her favor invalid and ineffective, because it is a
"disposicion captatoria", which knowledge she may easily acquire through consultation with a lawyer, there was no need her to go through the order of filing the
petition for the probate of the will. She could accomplish her desire by merely suppressing the will or tearing or destroying it, and then take steps leading to the
probate of the will executed in 1918. But for her conscience was clear and bade her to take the only proper step possible under the circumstances, which is to
institute the necessary proceedings for the probate of the 1939 will. This she did and the will was admitted to probate. But then the unexpected happened. Over her
vigorous opposition, the herein appellants filed a petition for reopening, and over her vigorous objection, the same was granted and the case was reopened. Her
motion for reconsideration was denied. Is it her fault that the case was reopened? Is it her fault that the order admitting the will to probate was set aside? That was a
contingency which petitioner never expected. Had appellants not filed their opposition to the probate of the will and had they limited their objection to the intrinsic
validity of said will, their plan to defeat the will and secure the intestacy of the deceased would have perhaps been accomplished. But they failed in their strategy. If
said will was denied probate it is due to their own effort. It is now unfair to impute bad faith petitioner simply because she exerted every effort to protect her own
interest and prevent the intestacy of the deceased to happen.

Having reached the foregoing conclusions, it is obvious that the court did not commit the second and third errors imputed to it by the counsel for appellants. Indeed,
petitioner cannot be considered guilty or estoppel which would prevent her from seeking the probate of the 1918 will simply because of her effort to obtain the
allowance of the 1939 will has failed considering that in both the 1918 and 1939 wills she was in by her husband as his universal heir. Nor can she be charged with
bad faith far having done so because of her desire to prevent the intestacy of her husband. She cannot be blamed being zealous in protecting her interest.

The next contention of appellants refers to the revocatory clause contained in 1939 will of the deceased which was denied probate. They contend that,
notwithstanding the disallowance of said will, the revocatory clause is valid and still has the effect of nullifying the prior of 1918.

Counsel for petitioner meets this argument by invoking the doctrine laid down in the case of Samson vs. Naval, (41 Phil., 838). He contends that the facts involved in
that case are on all fours with the facts of this case. Hence, the doctrine is that case is here controlling.

There is merit in this contention. We have carefully read the facts involved in the Samson case we are indeed impressed by their striking similarity with the facts of
this case. We do not need to recite here what those facts are; it is enough to point out that they contain many points and circumstances in common. No reason,
therefore, is seen by the doctrine laid down in that case (which we quote hereunder) should not apply and control the present case.

A subsequent will, containing a clause revoking a previous will, having been disallowed, for the reason that it was not executed in conformity with the provisions of
section 618 of the Code of Civil Procedure as to the making of wills, cannot produce the effect of annulling the previous will, inasmuch as said revocatory clause is
void. (41 Phil., 838.)

Apropos of this question, counsel for oppositors make the remark that, while they do not disagree with the soundness of the ruling laid down in the Samson case,
there is reason to abandon said ruling because it is archaic or antiquated and runs counter to the modern trend prevailing in American jurisprudence. They maintain
that said ruling is no longer controlling but merely represents the point of view of the minority and should, therefore, be abandoned, more so if we consider the fact
that section 623 of our Code of Civil Procedure, which governs the revocation of wills, is of American origin and as such should follow the prevailing trend of the
majority view in the United States. A long line of authorities is cited in support of this contention. And these authorities hold the view, that "an express revocation is
immediately effective upon the execution of the subsequent will, and does not require that it first undergo the formality of a probate proceeding". (p. 63, appellants'
brief .

While they are many cases which uphold the view entertained by counsel for oppositors, and that view appears to be in controlling the states where the decisions
had been promulgated, however, we are reluctant to fall in line with the assertion that is now the prevailing view in the United States. In the search we have made of
American authorities on the subject, we found ourselves in a pool of conflicting opinions perhaps because of the peculiar provisions contained in the statutes adopted
by each State in the subject of revocation of wills. But the impression we gathered from a review and the study of the pertinent authorities is that the doctrine laid
down in the Samson case is still a good law. On page 328 of the American Jurisprudence Vol. 57, which is a revision Published in 1948, we found the following
passages which in our opinion truly reflect the present trend of American jurisprudence on this matter affecting the revocation of wills:

SEC. 471. Observance of Formalities in Execution of Instrument. Ordinarily, statutes which permit the revocation of a will by another writing provide that to be
effective as a revocation, the writing must be executed with the same formalities which are required to be observed in the execution of a will. Accordingly, where,
under the statutes, attestation is necessary to the making of a valid will, an unattested non testamentary writing is not effective to revoke a prior will. It has been held
that a writing fails as a revoking instrument where it is not executed with the formalities requisite for the execution of a will, even though it is inscribed on the will
itself, although it may effect a revocation by cancellation or obliteration of the words of the will. A testator cannot reserve to himself the power to modify a will by a
written instrument subsequently prepared but not executed in the manner required for a will.

SEC, 472. Subsequent Unexecuted, Invalid, or Ineffective Will or Codicil. A will which is invalid because of the incapacity of the testator, or of undue influence can
have no effect whatever as a revoking will. Moreover, a will is not revoked by the unexecuted draft of a later one. Nor is a will revoked by a defectively executed will
or codicil, even though the latter contains a clause expressly revoking the former will, in a jurisdiction where it is provided by a controlling statute that no writing
other than a testamentary instrument is sufficient to revoke a will, for the simple reason that there is no revoking will. Similarly where the statute provides that a will
may be revoked by a subsequent will or other writing executed with the same formalities as are required in the execution of wills, a defectively executed will does not
revoke a prior will, since it cannot be said that there is a writing which complies with the statute. Moreover, a will or codicil which, on account of the manner in which
it is executed, is sufficient to pass only personally does not affect dispositions of real estate made by a former will, even though it may expressly purport to do so. The
intent of the testator to revoke is immaterial, if he has not complied with the statute. (57 Am. Jur., 328, 329.)

We find the same opinion in the American Law Reports, Annotated, edited in 1939. On page 1400, Volume 123, there appear many authorities on the "application of
rules where second will is invalid", among which a typical one is the following:

It is universally agreed that where the second will is invalid on account of not being executed in accordance with the provisions of the statute, or where the testator
who has not sufficient mental capacity to make a will or the will is procured through undue influence, or the such, in other words, where the second will is really no
will, it does not revoke the first will or affect it in any manner. Mort vs. Baker University (193-5) 229 Mo. App., 632, 78 S.W. (2d), 498.

These treaties cannot be mistaken. They uphold the view on which the ruling in the Samson case is predicated. They reflect the opinion that this ruling is sound and
good and for this reason, we see no justification for abondoning it as now suggested by counsel for the oppositors.

It is true that our law on the matter (sec. 623, Code Civil Procedure) provides that a will may be some will, codicil, or other writing executed as proved in case of wills"
but it cannot be said that the 1939 will should be regarded, not as a will within the meaning of said word, but as "other writing executed as provided in the case of
wills", simply because it was denied probate. And even if it be regarded as any other writing within the meaning of said clause, there is authority for holding that
unless said writing is admitted to probate, it cannot have the effect of revocation. (See 57 Am. Jur. pp. 329-330).

But counsel for oppositors contemned that, regardless of said revocatory clause, said will of 1918 cannot still be given effect because of the presumption that it was
deliberately revoked by the testator himself. The oppositors contend that the testator, after executing the 1939 will, and with full knowledge of the recovatory clause
contained said will, himself deliberately destroyed the original of the 1918 will, and for that reason the will submitted by petitioner for probate in these proceedings
is only a duplicate of said original.

There is no evidence which may directly indicate that the testator deliberately destroyed the original of the 1918 will because of his knowledge of the revocatory
clause contained in the will he executed in 1939. The only evidence we have is that when the first will was executed in 1918, Juan Salcedo, who prepared it, gave the
original and copies to the testator himself and apparently they remained in his possession until he executed his second will in 1939. And when the 1939 will was
denied probate on November 29, 1943, and petitioner was asked by her attorney to look for another will, she found the duplicate copy (Exhibit A) among the papers
or files of the testator. She did not find the original.

If it can be inferred that the testator deliberately destroyed the 1918 will because of his knowledge of the revocatory clause of the 1939 will, and it is true that he
gave a duplicate copy thereof to his wife, the herein petitioner, the most logical step for the testator to take is to recall said duplicate copy in order that it may
likewise be destroyed. But this was not done as shown by the fact that said duplicate copy remained in the possession of petitioner. It is possible that because of the
long lapse of twenty-one (21) years since the first will was executed, the original of the will had been misplaced or lost, and forgetting that there was a copy, the
testator deemed it wise to execute another will containing exactly the same testamentary dispositions. Whatever may be the conclusion we may draw from this chain
of circumstances, the stubborn fact is that there is no direct evidence of voluntary or deliberate destruction of the first will by the testator. This matter cannot be
inference or conjectur.

Granting for the sake of argument that the earlier will was voluntarily destroyed by the testator after the execution of the second will, which revoked the first, could
there be any doubt, under this theory, that said earlier will was destroyed by the testator in the honest belief that it was no longer necessary because he had
expressly revoked it in his will of 1939? In other words, can we not say that the destruction of the earlier will was but the necessary consequence of the testator's
belief that the revocatory clause contained in the subsequent will was valid and the latter would be given effect? If such is the case, then it is our opinion that the
earlier will can still be admitted to probate under the principle of "dependent relative revocation".

This doctrine is known as that of dependent relative revocation, and is usually applied where the testator cancels or destroys a will or executes an instrument
intended to revoke a will with a present intention to make a new testamentary disposition as a substitute for the old, and the new disposition is not made or, if made,
fails of effect for same reason. The doctrine is n limited to the existence of some other document, however, and has been applied where a will was destroyed as a
consequence of a mistake of law. . . . (68 C.J.P. 799).

The rule is established that where the act of destruction is connected with the making of another will so as fairly to raise the inference that the testator meant the
revocation of the old to depend upon the efficacy of a new disposition intended to be substituted, the revocation will be conditional and dependent upon the efficacy
of the new disposition; and if, for any reason, the new will intended to be made as a substitute is inoperative, the revocation fails and the original will remains in full
force. (Gardner, pp. 232, 233.)

This is the doctrine of dependent relative revocation. The failure of a new testamentary disposition upon whose validity the revocation depends, is equivalent to the
non-fulfillment of a suspensive conditions, and hence prevents the revocation of the original will. But a mere intent to make at some time a will in the place of that
destroyed will not render the destruction conditional. It must appear that the revocation is dependent upon the valid execution of a new will. (1 Alexander, p. 751;
Gardner, p. 253.)

We hold therefore, that even in the supposition that the destruction of the original will by the testator could be presumed from the failure of the petitioner to
produce it in court, such destruction cannot have the effect of defeating the prior will of 1918 because of the fact that it is founded on the mistaken belief that the
will of 1939 has been validly executed and would be given due effect. The theory on which this principle is predicated is that the testator did not intend to die
intestate. And this intention is clearly manifest when he executed two wills on two different occasion and instituted his wife as his universal heir. There can therefore
be no mistake as to his intention of dying testate.

The remaining question to be determined refers to the sufficiency of the evidence to prove the due execution of the will.

The will in question was attested, as required by law, by three witnesses, Lorenzo Morales, Rufino Enriquez, and Angel Cuenca. The first two witnesses died before
the commencement of the present proceedings. So the only instrumental witness available was Angel Cuenca and under our law and precedents, his testimony is
sufficient to prove the due execution of the will. However, petitioner presented not only the testimony of Cuenca but placed on the witness stand Juan Salcedo, the
notary public who prepared and notarized the will upon the express desire and instruction of the testator, The testimony of these witnesses shows that the will had
been executed in the manner required by law. We have read their testimony and we were impressed by their readiness and sincerity. We are convinced that they
told the truth.

Wherefore, the order appealed from is hereby affirmed, with costs against the appellants.1wphl.nt












Republic of the Philippines
SUPREME COURT
Manila

EN BANC

G.R. No. L-23445 June 23, 1966

REMEDIOS NUGUID, petitioner and appellant,
vs.
FELIX NUGUID and PAZ SALONGA NUGUID, oppositors and appellees.

Custodio O. Partade for petitioner and appellant.
Beltran, Beltran and Beltran for oppositors and appellees.

SANCHEZ, J.:

Rosario Nuguid, a resident of Quezon City, died on December 30, 1962, single, without descendants, legitimate or illegitimate. Surviving her were her legitimate
parents, Felix Nuguid and Paz Salonga Nuguid, and six (6) brothers and sisters, namely: Alfredo, Federico, Remedios, Conrado, Lourdes and Alberto, all surnamed
Nuguid.

On May 18, 1963, petitioner Remedios Nuguid filed in the Court of First Instance of Rizal a holographic will allegedly executed by Rosario Nuguid on November 17,
1951, some 11 years before her demise. Petitioner prayed that said will be admitted to probate and that letters of administration with the will annexed be issued to
her.

On June 25, 1963, Felix Nuguid and Paz Salonga Nuguid, concededly the legitimate father and mother of the deceased Rosario Nuguid, entered their opposition to the
probate of her will. Ground therefor, inter alia, is that by the institution of petitioner Remedios Nuguid as universal heir of the deceased, oppositors who are
compulsory heirs of the deceased in the direct ascending line were illegally preterited and that in consequence the institution is void.

On August 29, 1963, before a hearing was had on the petition for probate and objection thereto, oppositors moved to dismiss on the ground of absolute preterition.

On September 6, 1963, petitioner registered her opposition to the motion to dismiss.1wph1.t

The court's order of November 8, 1963, held that "the will in question is a complete nullity and will perforce create intestacy of the estate of the deceased Rosario
Nuguid" and dismissed the petition without costs.

A motion to reconsider having been thwarted below, petitioner came to this Court on appeal.

1. Right at the outset, a procedural aspect has engaged our attention. The case is for the probate of a will. The court's area of inquiry is limited to an examination
of, and resolution on, the extrinsic validity of the will. The due execution thereof, the testatrix's testamentary capacity, and the compliance with the requisites or
solemnities by law prescribed, are the questions solely to be presented, and to be acted upon, by the court. Said court at this stage of the proceedings is not called
upon to rule on the intrinsic validity or efficacy of the provisions of the will, the legality of any devise or legacy therein.1

A peculiar situation is here thrust upon us. The parties shunted aside the question of whether or not the will should be allowed probate. For them, the meat of the
case is the intrinsic validity of the will. Normally, this comes only after the court has declared that the will has been duly authenticated.2 But petitioner and
oppositors, in the court below and here on appeal, travelled on the issue of law, to wit: Is the will intrinsically a nullity?

We pause to reflect. If the case were to be remanded for probate of the will, nothing will be gained. On the contrary, this litigation will be protracted. And for aught
that appears in the record, in the event of probate or if the court rejects the will, probability exists that the case will come up once again before us on the same issue
of the intrinsic validity or nullity of the will. Result: waste of time, effort, expense, plus added anxiety. These are the practical considerations that induce us to a belief
that we might as well meet head-on the issue of the validity of the provisions of the will in question.3 After all, there exists a justiciable controversy crying for
solution.

2. Petitioner's sole assignment of error challenges the correctness of the conclusion below that the will is a complete nullity. This exacts from us a study of the
disputed will and the applicable statute.

Reproduced hereunder is the will:

Nov. 17, 1951

I, ROSARIO NUGUID, being of sound and disposing mind and memory, having amassed a certain amount of property, do hereby give, devise, and bequeath all of the
property which I may have when I die to my beloved sister Remedios Nuguid, age 34, residing with me at 38-B Iriga, Q.C. In witness whereof, I have signed my name
this seventh day of November, nineteen hundred and fifty-one.

(Sgd.) Illegible

T/ ROSARIO NUGUID

The statute we are called upon to apply in Article 854 of the Civil Code which, in part, provides:

ART. 854. The preterition or omission of one, some, or all of the compulsory heirs in the direct line, whether living at the time of the execution of the will or born after
the death of the testator, shall annul the institution of heir; but the devises and legacies shall be valid insofar as they are not inofficious. ...

Except for inconsequential variation in terms, the foregoing is a reproduction of Article 814 of the Civil Code of Spain of 1889, which is similarly herein copied, thus

Art. 814. The preterition of one or all of the forced heirs in the direct line, whether living at the time of the execution of the will or born after the death of the
testator, shall void the institution of heir; but the legacies and betterments4 shall be valid, in so far as they are not inofficious. ...

A comprehensive understanding of the term preterition employed in the law becomes a necessity. On this point Manresa comments:

La pretericion consiste en omitar al heredero en el testamento. O no se le nombra siquiera o aun nombrandole como padre, hijo, etc., no se le instituya heredero ni
se le deshereda expresamente ni se le asigna parte alguna de los bienes, resultando privado de un modo tacito de su derecho a legitima.

Para que exista pretericion, con arreglo al articulo 814, basta que en el testamento omita el testador a uno cualquiera de aquellos a quienes por su muerte
corresponda la herencia forzosa.

Se necesita, pues, a) Que la omision se refiera a un heredero forzoso. b) Que la omision sea completa; que el heredero forzoso nada reciba en el testamento.

It may now appear trite bat nonetheless helpful in giving us a clear perspective of the problem before us, to have on hand a clear-cut definition of the word annul:

To "annul" means to abrogate, to make void ... In re Morrow's Estate, 54 A. 342, 343, 204 Pa. 484.6

The word "annul" as used in statute requiring court to annul alimony provisions of divorce decree upon wife's remarriage means to reduce to nothing; to annihilate;
obliterate; blot out; to make void or of no effect; to nullify; to abolish. N.J.S.A. 2:50 38 (now N.J.S. 2A:34-35). Madden vs. Madden, 40 A. 2d 611, 614, 136 N..J Eq.
132.7

ANNUL. To reduce to nothing; annihilate; obliterate; to make void or of no effect; to nullify; to abolish; to do away with. Ex parte Mitchell, 123 W. Va. 283, 14 S.E. 2d.
771, 774.8

And now, back to the facts and the law. The deceased Rosario Nuguid left no descendants, legitimate or illegitimate. But she left forced heirs in the direct ascending
line her parents, now oppositors Felix Nuguid and Paz Salonga Nuguid. And, the will completely omits both of them: They thus received nothing by the testament;
tacitly, they were deprived of their legitime; neither were they expressly disinherited. This is a clear case of preterition. Such preterition in the words of Manresa
"anulara siempre la institucion de heredero, dando caracter absoluto a este ordenamiento referring to the mandate of Article 814, now 854 of the Civil Code.9 The
one-sentence will here institutes petitioner as the sole, universal heir nothing more. No specific legacies or bequests are therein provided for. It is in this posture
that we say that the nullity is complete. Perforce, Rosario Nuguid died intestate. Says Manresa:

En cuanto a la institucion de heredero, se anula. Lo que se anula deja de existir, en todo o en parte? No se aade limitacion alguna, como en el articulo 851, en el que
se expresa que se anulara la institucion de heredero en cuanto prejudique a la legitima del deseheredado Debe, pues, entenderse que la anulacion es completa o
total, y que este articulo como especial en el caso que le motiva rige con preferencia al 817. 10

The same view is expressed by Sanchez Roman:

La consecuencia de la anulacion o nulidad de la institucion de heredero por pretericion de uno, varios o todos los forzosos en linea recta, es la apertura de la sucesion
intestada total o parcial. Sera total, cuando el testador que comete la pretericion, hubiese dispuesto de todos los bienes por titulo universal de herencia en favor de
los herederos instituidos, cuya institucion se anula, porque asi lo exige la generalidad del precepto legal del art. 814, al determinar, como efecto de la pretericion, el
de que "anulara la institucion de heredero." ... 11

Really, as we analyze the word annul employed in the statute, there is no escaping the conclusion that the universal institution of petitioner to the entire inheritance
results in totally abrogating the will. Because, the nullification of such institution of universal heir without any other testamentary disposition in the will
amounts to a declaration that nothing at all was written. Carefully worded and in clear terms, Article 854 offers no leeway for inferential interpretation. Giving it an
expansive meaning will tear up by the roots the fabric of the statute. On this point, Sanchez Roman cites the "Memoria annual del Tribunal Supreme, correspondiente
a 1908", which in our opinion expresses the rule of interpretation, viz:

... El art. 814, que preceptua en tales casos de pretericion la nulidad de la institucion de heredero, no consiente interpretacion alguna favorable a la persona instituida
en el sentido antes expuesto aun cuando parezca, y en algun caso pudiera ser, mas o menos equitativa, porque una nulidad no significa en Derecho sino la suposicion
de que el hecho o el acto no se ha realizado, debiendo por lo tanto procederse sobre tal base o supuesto, y consiguientemente, en un testamento donde falte la
institucion, es obligado llamar a los herederos forzosos en todo caso, como habria que llamar a los de otra clase, cuando el testador no hubiese distribudo todos sus
bienes en legados, siendo tanto mas obligada esta consecuencia legal cuanto que, en materia de testamentos, sabido es, segun tiene declarado la jurisprudencia, con
repeticion, que no basta que sea conocida la voluntad de quien testa si esta voluntad no aparece en la forma y en las condiciones que la ley ha exigido para que sea
valido y eficaz, por lo que constituiria una interpretacion arbitraria, dentro del derecho positivo, reputar como legatario a un heredero cuya institucion fuese anulada
con pretexto de que esto se acomodaba mejor a la voluntad del testador, pues aun cuando asi fuese, sera esto razon para modificar la ley, pero no autoriza a una
interpretacion contraria a sus terminos y a los principios que informan la testamentifaccion, pues no porque parezca mejor una cosa en el terreno del Derecho
constituyente, hay razon para convereste juicio en regla de interpretacion, desvirtuando y anulando por este procedimiento lo que el legislador quiere establecer. 12

3. We should not be led astray by the statement in Article 854 that, annullment notwithstanding, "the devises and legacies shall be valid insofar as they are not
inofficious". Legacies and devises merit consideration only when they are so expressly given as such in a will. Nothing in Article 854 suggests that the mere institution
of a universal heir in a will void because of preterition would give the heir so instituted a share in the inheritance. As to him, the will is inexistent. There must be,
in addition to such institution, a testamentary disposition granting him bequests or legacies apart and separate from the nullified institution of heir. Sanchez Roman,
speaking of the two component parts of Article 814, now 854, states that preterition annuls the institution of the heir "totalmente por la pretericion"; but added (in
reference to legacies and bequests) "pero subsistiendo ... todas aquellas otras disposiciones que no se refieren a la institucion de heredero ... . 13 As Manresa puts it,
annulment throws open to intestate succession the entire inheritance including "la porcion libre (que) no hubiese dispuesto en virtud de legado, mejora o donacion.
14

As aforesaid, there is no other provision in the will before us except the institution of petitioner as universal heir. That institution, by itself, is null and void. And,
intestate succession ensues.

4. Petitioner's mainstay is that the present is "a case of ineffective disinheritance rather than one of preterition". 15 From this, petitioner draws the conclusion that
Article 854 "does not apply to the case at bar". This argument fails to appreciate the distinction between pretention and disinheritance.

Preterition "consists in the omission in the testator's will of the forced heirs or anyone of them, either because they are not mentioned therein, or, though
mentioned, they are neither instituted as heirs nor are expressly disinherited." 16 Disinheritance, in turn, "is a testamentary disposition depriving any compulsory heir
of his share in the legitime for a cause authorized by law. " 17 In Manresa's own words: "La privacion expresa de la legitima constituye la desheredacion. La privacion
tacita de la misma se denomina pretericion." 18 Sanchez Roman emphasizes the distinction by stating that disinheritance "es siempre voluntaria"; preterition, upon
the other hand, is presumed to be "involuntaria". 19 Express as disinheritance should be, the same must be supported by a legal cause specified in the will itself. 20

The will here does not explicitly disinherit the testatrix's parents, the forced heirs. It simply omits their names altogether. Said will rather than be labeled ineffective
disinheritance is clearly one in which the said forced heirs suffer from preterition.

On top of this is the fact that the effects flowing from preterition are totally different from those of disinheritance. Preterition under Article 854 of the Civil Code, we
repeat, "shall annul the institution of heir". This annulment is in toto, unless in the will there are, in addition, testamentary dispositions in the form of devises or
legacies. In ineffective disinheritance under Article 918 of the same Code, such disinheritance shall also "annul the institution of heirs", put only "insofar as it may
prejudice the person disinherited", which last phrase was omitted in the case of preterition. 21 Better stated yet, in disinheritance the nullity is limited to that portion
of the estate of which the disinherited heirs have been illegally deprived. Manresa's expressive language, in commenting on the rights of the preterited heirs in the
case of preterition on the one hand and legal disinheritance on the other, runs thus: "Preteridos, adquiren el derecho a todo; desheredados, solo les corresponde un
tercio o dos tercios, 22 el caso. 23

5. Petitioner insists that the compulsory heirs ineffectively disinherited are entitled to receive their legitimes, but that the institution of heir "is not invalidated,"
although the inheritance of the heir so instituted is reduced to the extent of said legitimes. 24

This is best answered by a reference to the opinion of Mr. Chief Justice Moran in the Neri case heretofore cited, viz:

But the theory is advanced that the bequest made by universal title in favor of the children by the second marriage should be treated as legado and mejora and,
accordingly, it must not be entirely annulled but merely reduced. This theory, if adopted, will result in a complete abrogation of Articles 814 and 851 of the Civil Code.
If every case of institution of heirs may be made to fall into the concept of legacies and betterments reducing the bequest accordingly, then the provisions of Articles
814 and 851 regarding total or partial nullity of the institution, would. be absolutely meaningless and will never have any application at all. And the remaining
provisions contained in said article concerning the reduction of inofficious legacies or betterments would be a surplusage because they would be absorbed by Article
817. Thus, instead of construing, we would be destroying integral provisions of the Civil Code.

The destructive effect of the theory thus advanced is due mainly to a failure to distinguish institution of heirs from legacies and betterments, and a general from a
special provision. With reference to article 814, which is the only provision material to the disposition of this case, it must be observed that the institution of heirs is
therein dealt with as a thing separate and distinct from legacies or betterments. And they are separate and distinct not only because they are distinctly and
separately treated in said article but because they are in themselves different. Institution of heirs is a bequest by universal title of property that is undetermined.
Legacy refers to specific property bequeathed by a particular or special title. ... But again an institution of heirs cannot be taken as a legacy. 25

The disputed order, we observe, declares the will in question "a complete nullity". Article 854 of the Civil Code in turn merely nullifies "the institution of heir".
Considering, however, that the will before us solely provides for the institution of petitioner as universal heir, and nothing more, the result is the same. The entire will
is null.

Upon the view we take of this case, the order of November 8, 1963 under review is hereby affirmed. No costs allowed. So ordered.

Concepcion, C.J., Reyes, J.B.L., Barrera, Dizon, Regala, Makalintal, Bengzon, J.P. and Zaldivar, JJ., concur.



















EN BANC

[G.R. No. L-12207. December 24, 1959.]

JUAN PALACIOS, Petitioner-Appellant, v. MARIA CATIMBANG PALACIOS, Oppositor-Appellee.

Augusto Francisco and Vicente Reyes Villavicencio for Appellant.

Laureano C. Alano and Enrique A. Amador for Appellee.


SYLLABUS

1. WILLS; PROBATE DURING LIFETIME OF TESTATOR; OPPOSITION TO INTRINSIC VALIDITY OF THE WILL NOT ALLOWED. Opposition to the intrinsic validity or to the
legality of the provisions of the will cannot be entertained in probate proceeding because its only purpose is merely to determine if the will has been executed in
accordance with the requirements of the law, much less if the purpose of the opposition is to show that the oppositor is an acknowledged natural child who allegedly
has been ignored in the will for such issue cannot be raised here but in a separate action. This is especially so when the testator, as in the present case, is still alive
and has merely filed a petition for the allowance of his will leaving the effects thereof after his death.

2. ID.; ID.; RIGHT OR POWER OF TESTATOR. "After a will has been probated during the lifetime of a testator it does not necessarily mean that he cannot alter or
revoke the same before his death. Should he make a new will, it would also be allowable on his petition, and if he should die before he had a chance to present such
petition, the ordinary probate proceedings after the testators death would be in order" (Report of the Code Commission, pp. 53-54). The reason for this comment is
that the rights to the succession are transmitted from the moment of the death of the decedent (Article 777, New Civil Code).
D E C I S I O N


BAUTISTA ANGELO, J.:


Juan Palacios executed his last will and testament on June 25, 1946 and availing himself of the provisions of the new Civil Code, he filed on May 23, 1956 before the
Court of First Instance of Batangas a petition for its approval. In said will, he instituted as his sole heirs his natural children Antonio C. Palacios and Andrea C. Palacios.

On June 21, 1956, Maria Catimbang filed an opposition to the probate of the will alleging that she is the acknowledged natural daughter of petitioner but that she
was completely ignored in said will thus impairing her legitime.

After the presentation of petitioners evidence relative to the essential requisites and formalities provided by law for the validity of a will, the court on July 6, 1956
issued an order admitting the will to probate. The court, however, set a date for the hearing of the opposition relative to the intrinsic validity of the will and, after
proper hearing concerning this incident, the court issued another order declaring oppositor to be the natural child of petitioner and annulling the will insofar as it
impairs her legitime, with costs against petitioner.

From this last order, petitioner gave notice of his intention to appeal directly to the Supreme Court, and accordingly, the record was elevated to this Court.

It should be noted that petitioner instituted the present proceeding in order to secure the probate of his will availing himself of the provisions of Article 838,
paragraph 2, of the new Civil Code, which permit a testator to petition the proper court during his lifetime for the allowance of his will, but to such petition one Maria
Catimbang filed an opposition alleging that she is the acknowledged natural daughter of petitioner but that she was completely ignored in the will thus impairing her
legitime. In other words, Maria Catimbang does not object to the probate of the will insofar as its due execution is concerned or on the ground that it has not
complied with the formalities prescribed by law; rather she objects to its intrinsic validity or to the legality of the provisions of the will.

We hold that such opposition cannot be entertained in this proceeding because its only purpose is merely to determine if the will has been executed in accordance
with the requirements of the law, much less if the purpose of the opposition is to show that the oppositor is an acknowledged natural child who allegedly has been
ignored in the will for such issue cannot be raised here but in a separate action. This is especially so when the testator, as in the present case, is still alive and has
merely filed a petition for the allowance of his will leaving the effects thereof after his death. This is in line with our ruling in Montaano v. Suesa, 14 Phil., 676,
wherein we said: "The authentication of the will decides no other questions than such as touch upon the capacity of the testator and the compliance with those
requisites or solemnities which the law prescribes for the validity of a will. It does not determine nor even by implication prejudge the validity or efficiency of the
provisions; that may be impugned as being vicious or null, notwithstanding its authentication. The questions relating to these points remain entirely un-affected, and
may be raised even after the will has been authenticated."cralaw virtua1aw library

On the other hand, "after a will has been probated during the lifetime of a testator it does not necessarily mean that he cannot alter or revoke the same before his
death. Should he make a new will, it would also be allowable on his petition, and if he should die before he has had a chance to present such petition, the ordinary
probate proceedings after the testators death would be in order" (Report of the Code Commission, pp. 53-54). The reason for this comment is that the rights to the
succession are transmitted from the moment of the death of the decedent (Article 777, new Civil Code). It is clear that the trial court erred in entertaining the
opposition and in annulling the portion of the will which allegedly impairs the legitime of the oppositor on the ground that, as it has found, she is an acknowledged
natural daughter of the testator. This is an extraneous matter which should be threshed out in a separate action. Wherefore, the order appealed from is set aside,
without pronouncement as to costs.
Republic of the Philippines
SUPREME COURT
Manila

EN BANC

G.R. No. 45629 September 22, 1938

ANTILANO G. MERCADO, petitioner,
vs.
ALFONSO SANTOS, Judge of First Instance of Pampanga, respondents.
ROSARIO BASA DE LEON, ET AL., intervenors.

Claro M. Recto and Benigno S. Aquino for petitioner.
Esperanza de la Cruz and Heracio Abistao for respondents.
Sotto and Sotto for intervenors.

LAUREL, J.:

On May 28, 1931, the petitioner herein filed in the Court of First Instance of Pampanga a petition for the probate of the will of his deceased wife, Ines Basa. Without
any opposition, and upon the testimony of Benigno F. Gabino, one of the attesting witnesses, the probate court, on June 27,1931, admitted the will to probate.
Almost three years later, on April 11, 1934, the five intervenors herein moved ex parte to reopen the proceedings, alleging lack of jurisdiction of the court to probate
the will and to close the proceedings. Because filed ex parte, the motion was denied. The same motion was filed a second time, but with notice to the adverse party.
The motion was nevertheless denied by the probate court on May 24, 1934. On appeal to this court, the order of denial was affirmed on July 26, 1935. (Basa vs.
Mercado, 33 Off. Gaz., 2521.)

It appears that on October 27, 1932, i. e., sixteen months after the probate of the will of Ines Basa, intervenor Rosario Basa de Leon filed with the justice of the peace
court of San Fernando, Pampanga, a complaint against the petitioner herein, for falsification or forgery of the will probated as above indicated. The petitioner was
arrested. He put up a bond in the sum of P4,000 and engaged the services of an attorney to undertake his defense. Preliminary investigation of the case was
continued twice upon petition of the complainant. The complaint was finally dismissed, at the instance of the complainant herself, in an order dated December 8,
1932. Three months later, or on March 2, 1933, the same intervenor charged the petitioner for the second time with the same offense, presenting the complaint this
time in the justice of the peace court of Mexico, Pampanga. The petitioner was again arrested, again put up a bond in the sum of P4,000, and engaged the services of
counsel to defend him. This second complaint, after investigation, was also dismissed, again at the instance of the complainant herself who alleged that the petitioner
was in poor health. That was on April 27, 1933. Some nine months later, on February 2, 1934, to be exact, the same intervenor accused the same petitioner for the
third time of the same offense. The information was filed by the provincial fiscal of Pampanga in the justice of the peace court of Mexico. The petitioner was again
arrested, again put up a bond of P4,000, and engaged the services of defense counsel. The case was dismissed on April 24, 1934, after due investigation, on the
ground that the will alleged to have been falsified had already been probated and there was no evidence that the petitioner had forged the signature of the testatrix
appearing thereon, but that, on the contrary, the evidence satisfactorily established the authenticity of the signature aforesaid. Dissatisfied with the result, the
provincial fiscal, on May 9, 1934, moved in the Court of First Instance of Pampanga for reinvestigation of the case. The motion was granted on May 23, 1934, and, for
the fourth time, the petitioner was arrested, filed a bond and engaged the services of counsel to handle his defense. The reinvestigation dragged on for almost a year
until February 18, 1934, when the Court of First Instance ordered that the case be tried on the merits. The petitioner interposed a demurrer on November 25, 1935,
on the ground that the will alleged to have been forged had already been probated. This demurrer was overruled on December 24, 1935, whereupon an exception
was taken and a motion for reconsideration and notice of appeal were filed. The motion for reconsideration and the proposed appeal were denied on January 14,
1936. The case proceeded to trial, and forthwith petitioner moved to dismiss the case claiming again that the will alleged to have been forged had already been
probated and, further, that the order probating the will is conclusive as to the authenticity and due execution thereof. The motion was overruled and the petitioner
filed with the Court of Appeals a petition for certiorari with preliminary injunction to enjoin the trial court from further proceedings in the matter. The injunction was
issued and thereafter, on June 19, 1937, the Court of Appeals denied the petition for certiorari, and dissolved the writ of preliminary injunction. Three justices
dissented in a separate opinion. The case is now before this court for review on certiorari.

Petitioner contends (1) that the probate of the will of his deceased wife is a bar to his criminal prosecution for the alleged forgery of the said will; and, (2) that he has
been denied the constitutional right to a speedy trial.

1. Section 306 of our Code of Civil Procedure provides as to the effect of judgments.

SEC. 306. Effect of judgment. The effect of a judgment or final order in an action or special proceeding before a court or judge of the Philippine Islands or of the
United States, or of any State or Territory of the United States, having jurisdiction to pronounce the judgment or order, may be as follows.

1. In case of a judgment or order against a specific thing, or in respect to the probate of a will, or the administration of the estate of a deceased person, or in
respect to the personal, political, or legal condition or relation of a particular person, the judgment or order is conclusive upon the title of the thing, the will or
administration, or the condition or relation of the person Provided, That the probate of a will or granting of letters of administration shall only be prima facie
evidence of the death of the testator or intestate.

xxx xxx xxx

(Emphasis ours.)

Section 625 of the same Code is more explicit as to the conclusiveness of the due execution of a probate will. It says.

SEC. 625. Allowance Necessary, and Conclusive as to Execution. No will shall pass either the real or personal estate, unless it is proved and allowed in the Court of
First Instance, or by appeal to the Supreme Court; and the allowance by the court of a will of real and personal estate shall be conclusive as to its due execution.
(Emphasis ours.)

(In Manahan vs. Manahan 58 Phil., 448, 451), we held:

. . . The decree of probate is conclusive with respect to the due execution thereof and it cannot be impugned on any of the grounds authorized by law, except that of
fraud, in any separate or independent action or proceeding. Sec. 625, Code of Civil Procedure; Castaeda vs. Alemany, 3 Phil., 426; Pimentel vs. Palanca, 5 Phil., 436;
Sahagun vs. De Gorostiza, 7 Phil., 347; Limjuco vs. Ganara, 11 Phil., 393; Montaano vs. Suesa, 14 Phil., 676; in re Estate of Johnson, 39 Phil, 156; Riera vs. Palmaroli,
40 Phil., 105; Austria vs. Ventenilla, 21 Phil., 180; Ramirez vs. Gmur, 42 Phil., 855; and Chiong Jocsoy vs. Vano, 8 Phil., 119.

In 28 R. C. L., p. 377, section 378, it is said.

The probate of a will by the probate court having jurisdiction thereof is usually considered as conclusive as to its due execution and validity, and is also conclusive that
the testator was of sound and disposing mind at the time when he executed the will, and was not acting under duress, menace, fraud, or undue influence, and that
the will is genuine and not a forgery. (Emphasis ours.)

As our law on wills, particularly section 625 of our Code of Civil Procedure aforequoted, was taken almost bodily from the Statutes of Vermont, the decisions of the
Supreme Court of the State relative to the effect of the probate of a will are of persuasive authority in this jurisdiction. The Vermont statute as to the conclusiveness
of the due execution of a probated will reads as follows.

SEC. 2356. No will shall pass either real or personal estate, unless it is proved and allowed in the probate court, or by appeal in the county or supreme court; and the
probate of a will of real or personal estate shall be conclusive as to its due execution. (Vermont Statutes, p. 451.)

Said the Supreme Court of Vermont in the case of Missionary Society vs. Eells (68 Vt., 497, 504): "The probate of a will by the probate court having jurisdiction
thereof, upon the due notice, is conclusive as to its due execution against the whole world. (Vt. St., sec. 2336; Fosters Exrs. vs. Dickerson, 64 Vt., 233.)"

The probate of a will in this jurisdiction is a proceeding in rem. The provision of notice by Publication as a prerequisite to the allowance of a will is constructive notice
to the whole world, and when probate is granted, the judgment of the court is binding upon everybody, even against the State. This court held in the case of Manalo
vs. Paredes and Philippine Food Co. (47 Phil., 938):

The proceeding for the probate of a will is one in rem (40 Cyc., 1265), and the court acquires jurisdiction over all the persons interested, through the publication of
the notice prescribed by section 630 of the Code of Civil Procedure, and any order that may be entered therein is binding against all of them.

Through the publication of the petition for the probate of the will, the court acquires jurisdiction over all such persons as are interested in said will; and any judgment
that may be rendered after said proceeding is binding against the whole world.

In Everrett vs. Wing (103 Vt., 488, 492), the Supreme Court of Vermont held.

In this State the probate of a will is a proceeding in rem being in form and substance upon the will itself to determine its validity. The judgment determines the status
of the instrument, whether it is or is not the will of the testator. When the proper steps required by law have been taken the judgment is binding upon everybody,
and makes the instrument as to all the world just what the judgment declares it to be. (Woodruff vs. Taylor, 20 Vt., 65, 73; Burbeck vs. Little, 50 Vt., 713, 715;
Missionary Society vs. Eells, 68 Vt., 497, 504; 35 Atl., 463.) The proceedings before the probate court are statutory and are not governed by common law rules as to
parties or causes of action. (Holdrige vs. Holdriges Estate, 53 Vt., 546, 550; Purdy vs. Estate of Purdy, 67 Vt. 50, 55; 30 Atl., 695.) No process is issued against anyone in
such proceedings, but all persons interested in determining the state or conditions of the instrument are constructively notified by the publication of notice as
required by G. L. 3219. (Woodruff vs. Taylor, supra; In re Warners Estate 98 Vt., 254; 271; 127 Atl., 362.)

Section 333, paragraph 4, of the Code of Civil Procedure establishes an incontrovertible presumption in favor of judgments declared by it to be conclusive.

SEC. 333. Conclusive Presumptions. The following presumptions or deductions, which the law expressly directs to be made from particular facts, are deemed
conclusive.

xxx xxx xxx

4. The judgment or order of a court, when declared by this code to be conclusive.

Conclusive presumptions are inferences which the law makes so peremptory that it will not allow them to be overturned by any contrary proof however strong.
(Brant vs. Morning Journal Assn., 80 N.Y.S., 1002, 1004; 81 App. Div., 183; see, also, Joslyn vs. Puloer, 59 Hun., 129, 140, 13 N.Y.S., 311.) The will in question having
been probated by a competent court, the law will not admit any proof to overthrow the legal presumption that it is genuine and not a forgery.

The majority decision of the Court of Appeals cites English decisions to bolster up its conclusion that "the judgment admitting the will to probate is binding upon the
whole world as to the due execution and genuineness of the will insofar as civil rights and liabilities are concerned, but not for the purpose of punishment of a crime."
The cases of Dominus Rex vs. Vincent, 93 English Reports, Full Reprint, 795, the first case being decided in 1721, were cited to illustrate the earlier English decisions to
the effect that upon indictment for forging a will, the probating of the same is conclusive evidence in the defendants favor of its genuine character. Reference is
made, however, to the cases of Rex vs. Gibson, 168 English Reports, Full Reprint, 836, footnote (a), decided in 1802, and Rex vs. Buttery and Macnamarra, 168 English
Reports, Full Reprint, 836, decided in 1818, which establish a contrary rule. Citing these later cases, we find the following quotation from Black on Judgments, Vol. II,
page 764.

A judgment admitting a will to probate cannot be attacked collaterally although the will was forged; and a payment to the executor named therein of a debt due the
decedent will discharge the same, notwithstanding the spurious character of the instrument probated. It has also been held that, upon an indictment for forging a
will, the probate of the paper in question is conclusive evidence in the defendants favor of its genuine character. But this particular point has lately been ruled
otherwise.

It was the case of Rex vs. Buttery, supra, which induced the Supreme Court of Massachussetts in the case of Waters vs. Stickney (12 Allen 1; 90 Am. Dec., 122) also
cited by the majority opinion, to hold that "according to later and sounder decisions, the probate, though conclusive until set aside of the disposition of the property,
does not protect the forger from punishment." This was reproduced in 28 R.C.L., p. 376, and quoted in Barry vs. Walker (103 Fla., 533; 137 So., 711, 715), and
Thompson vs. Freeman (149 So., 740, 742), also cited in support of the majority opinion of the Court of Appeals. The dissenting opinion of the Court of Appeals in the
instant case under review makes a cursory study of the statutes obtaining in England, Massachussetts and Florida, and comes to the conclusion that the decisions
cited in the majority opinion do not appear to "have been promulgated in the face of statutes similar to ours." The dissenting opinion cites Whartons Criminal
Evidence (11th ed., sec. 831), to show that the probate of a will in England is only prima facie proof of the validity of the will (Op. Cit. quoting Marriot vs. Marriot, 93
English Reprint, 770); and 21 L.R.A. (pp. 686689 and note), to show that in Massachussetts there is no statute making the probate of a will conclusive, and that in
Florida the statute(sec. 1810, Revised Statutes) makes the probate conclusive evidence as to the validity of the will with regard to personal, and prima facie as to real
estate. The cases decided by the Supreme Court of Florida cited by the majority opinion, supra, refer to wills of both personal and real estate.

The petitioner cites the case of State vs. McGlynn (20 Cal., 233, decided in 1862), in which Justice Norton of the Supreme Court of California, makes the following
review of the nature of probate proceedings in England with respect to wills personal and real property.

In England, the probate of wills of personal estate belongs to the Ecclesiastical Courts. No probate of a will relating to real estate is there necessary. The real estate,
upon the death of the party seized, passes immediately to the devisee under the will if there be one; or if there be no will, to the heir at law. The person who thus
becomes entitled takes possession. If one person claims to be the owner under a will, and another denies the validity of the will and claims to be the owner as heir at
law, an action of ejectment is brought against the party who may be in possession by the adverse claimant; and on the trial of such an action, the validity of the will is
contested, and evidence may be given by the respective parties as to the capacity of the testator to make a will, or as to any fraud practiced upon him, or as to the
actual execution of it, or as to any other circumstance affecting its character as a valid devise of the real estate in dispute. The decision upon the validity of the will in
such action becomes res adjudicata, and is binding and conclusive upon the parties to that action and upon any person who may subsequently acquire the title from
either of those parties; but the decision has no effect upon other parties, and does not settle what may be called the status or character of the will, leaving it subject
to be enforced as a valid will, or defeated as invalid, whenever other parties may have a contest depending upon it. A probate of a will of personal property, on the
contrary, is a judicial determination of the character of the will itself. It does not necessarily or ordinarily arise from any controversy between adverse claimants, but
is necessary in order to authorize a disposition of the personal estate in pursuance of its provisions. In case of any controversy between adverse claimants of the
personal estate, the probate is given in evidence and is binding upon the parties, who are not at liberty to introduce any other evidence as to the validity of the will.

The intervenors, on the other hand, attempt to show that the English law on wills is different from that stated in the case of State vs. McGlynn, supra, citing the
following statutes.

1. The Wills Act, 1837 (7 Will. 4 E 1 Vict. c. 26).

2. The Court of Probate Act, 1857 (20 and 21 Vict. c. 77).

3. The Judicature Act, 1873 (36 and 37 Vict. c. 66).

The Wills Act of 1837 provides that probate may be granted of "every instrumental purporting to be testamentary and executed in accordance with the statutory
requirements . . . if it disposes of property, whether personal or real." The Ecclesiastical Courts which took charge of testamentary causes (Ewells Blackstone [1910],
p. 460), were determined by the Court of Probate Act of 1857, and the Court of Probate in turn was, together with other courts, incorporated into the Supreme Court
of Judicature, and transformed into the Probate Division thereof, by the Judicature Act of 1873. (Lord Halsbury, The Laws of England[1910], pp. 151156.) The
intervenors overlook the fact, however, that the case of Rex vs. Buttery and Macnamarra, supra, upon which they rely in support of their theory that the probate of a
forged will does not protect the forger from punishment, was decided long before the foregoing amendatory statutes to the English law on wills were enacted. The
case of State vs. McGlynn may be considered, therefore, as more or less authoritative on the law of England at the time of the promulgation of the decision in the
case of Rex vs. Buttery and Macnamarra.

In the case of State vs. McGlynn, the Attorney General of California filed an information to set aside the probate of the will of one Broderick, after the lapse of one
year provided by the law of California for the review of an order probating a will, in order that the estate may be escheated to the State of California for the review of
an probated will was forged and that Broderick therefore died intestate, leaving no heirs, representatives or devisees capable of inheriting his estate. Upon these
facts, the Supreme Court of California held.

The fact that a will purporting to be genuine will of Broderick, devising his estate to a devisee capable of inheriting and holding it, has been admitted to probate and
established as a genuine will by the decree of a Probate Court having jurisdiction of the case, renders it necessary to decide whether that decree, and the will
established by it, or either of them, can be set aside and vacated by the judgment of any other court. If it shall be found that the decree of the Probate Court, not
reversed by the appellate court, is final and conclusive, and not liable to be vacated or questioned by any other court, either incidentally or by any direct proceeding,
for the purpose of impeaching it, and that so long as the probate stands the will must be recognized and admitted in all courts to be valid, then it will be immaterial
and useless to inquire whether the will in question was in fact genuine or forged. (State vs. McGlynn, 20 Cal., 233; 81 Am. Dec., 118, 121.).

Although in the foregoing case the information filed by the State was to set aside the decree of probate on the ground that the will was forged, we see no difference
in principle between that case and the case at bar. A subtle distinction could perhaps be drawn between setting aside a decree of probate, and declaring a probated
will to be a forgery. It is clear, however, that a duly probated will cannot be declared to be a forgery without disturbing in a way the decree allowing said will to
probate. It is at least anomalous that a will should be regarded as genuine for one purpose and spurious for another.

The American and English cases show a conflict of authorities on the question as to whether or not the probate of a will bars criminal prosecution of the alleged
forger of the probate will. We have examined some important cases and have come to the conclusion that no fixed standard maybe adopted or drawn therefrom, in
view of the conflict no less than of diversity of statutory provisions obtaining in different jurisdictions. It behooves us, therefore, as the court of last resort, to choose
that rule most consistent with our statutory law, having in view the needed stability of property rights and the public interest in general. To be sure, we have seriously
reflected upon the dangers of evasion from punishment of culprits deserving of the severity of the law in cases where, as here, forgery is discovered after the probate
of the will and the prosecution is had before the prescription of the offense. By and large, however, the balance seems inclined in favor of the view that we have
taken. Not only does the law surround the execution of the will with the necessary formalities and require probate to be made after an elaborate judicial proceeding,
but section 113, not to speak of section 513, of our Code of Civil Procedure provides for an adequate remedy to any party who might have been adversely affected by
the probate of a forged will, much in the same way as other parties against whom a judgment is rendered under the same or similar circumstances. (Pecson vs.
Coronel, 43 Phil., 358.)The aggrieved party may file an application for relief with the proper court within a reasonable time, but in no case exceeding six months after
said court has rendered the judgment of probate, on the ground of mistake, inadvertence, surprise or excusable neglect. An appeal lies to review the action of a court
of first instance when that court refuses to grant relief. (Banco Espaol Filipino vs. Palanca, 37 Phil., 921; Philippine Manufacturing Co. vs. Imperial, 47 Phil., 810;
Samia vs. Medina, 56 Phil., 613.) After a judgment allowing a will to be probated has become final and unappealable, and after the period fixed by section 113 of the
Code of Civil Procedure has expired, the law as an expression of the legislative wisdom goes no further and the case ends there.

. . . The court of chancery has no capacity, as the authorities have settled, to judge or decide whether a will is or is not a forgery; and hence there would be an
incongruity in its assuming to set aside a probate decree establishing a will, on the ground that the decree was procured by fraud, when it can only arrive at the fact
of such fraud by first deciding that the will was a forgery. There seems, therefore, to be a substantial reason, so long as a court of chancery is not allowed to judge of
the validity of a will, except as shown by the probate, for the exception of probate decrees from the jurisdiction which courts of chancery exercise in setting aside
other judgments obtained by fraud. But whether the exception be founded in good reason or otherwise, it has become too firmly established to be disregarded. At
the present day, it would not be a greater assumption to deny the general rule that courts of chancery may set aside judgments procured by fraud, than to deny the
exception to that rule in the case of probate decrees. We must acquiesce in the principle established by the authorities, if we are unable to approve of the reason.
Judge Story was a staunch advocate for the most enlarged jurisdiction of courts of chancery, and was compelled to yield to the weight of authority. He says "No other
excepted case is known to exist; and it is not easy to discover the grounds upon which this exception stands, in point of reason or principle, although it is clearly
settled by authority. (1 Storys Eq. Jur. sec. 440.)" (State vs. McGlynn, 20 Cal., 233; 81 Am. Dec., 118, 129. See, also, Tracy vs. Muir, 121 American State Reports, 118,
125.)

We hold, therefore, that in view of the provisions of sections 306, 333 and 625 of our Code of Civil Procedure, criminal action will not lie in this jurisdiction against the
forger of a will which had been duly admitted to probate by a court of competent jurisdiction.

The resolution of the foregoing legal question is sufficient to dispose of the case. However, the other legal question with reference to the denial to the accused of his
right to a speedy trial having been squarely raised and submitted, we shall proceed to consider the same in the light of cases already adjudicated by this court.

2. The Constitution of the Philippines provides that "In all criminal prosecutions the accused . . . shall enjoy the right . . . to have a speedy . . . trial. . . . (Art. III,
sec. 1, par. 17. See, also, G.O. No. 58, sec. 15, No. 7.) Similar provisions are to be found in the Presidents Instructions to the Second Philippine Commission (par. 11),
the Philippine Bill of July 1, 1902 (sec. 5, par. 2) and the Jones Act of August 29, 1916 (sec. 3, par. 2). The provisions in the foregoing organic acts appear to have been
taken from similar provisions in the Constitution of the United States (6th Amendment) and those of the various states of the American Union. A similar injunction is
contained in the Malolos Constitution (art. 8, Title IV), not to speak of other constitutions. More than once this court had occasion to set aside the proceedings in
criminal cases to give effect to the constitutional injunction of speedy trial. (Conde vs. Judge of First Instance and Fiscal of Tayabas [1923], 45 Phil., 173; Conde vs.
Rivera and Unson[1924], 45 Phil., 650; People vs. Castaeda and Fernandez[1936]), 35 Off. Gaz., 1269; Kalaw vs. Apostol, Oct. 15, 1937, G.R. No. 45591; Esguerra vs.
De la Costa, Aug. 30,1938, G.R. No. 46039.).

In Conde vs. Rivera and Unson, supra, decided before the adoption of our Constitution, we said.

Philippine organic and statutory law expressly guarantee that in all criminal prosecutions the accused shall enjoy the right to have a speedy trial. Aurelia Conde, like
all other accused persons, has a right to a speedy trial in order that if innocent she may go free, and she has been deprived of that right in defiance of law. Dismissed
from her humble position, and compelled to dance attendance on courts while investigations and trials are arbitrarily postponed without her consent, is palpably and
openly unjust to her and a detriment to the public. By the use of reasonable diligence, the prosecution could have settled upon the appropriate information, could
have attended to the formal preliminary examination, and could have prepared the case for a trial free from vexatious, capricious, and oppressive delays.

In People vs. Castaeda and Fernandez, supra, this court found that the accused had not been given a fair and impartial trial. The case was to have been remanded to
the court a quo for a new trial before an impartial judge. This step, however, was found unnecessary. A review of the evidence convinced this court that a judgment
of conviction for theft, as charged, could not be sustained and, having in view the right to a speedy trial guaranteed by the Constitution to every person accused of
crime, entered a judgment acquitting the accused, with costs de oficio. We said.

. . . The Constitution, Article III, section 1, paragraph 17, guarantees to every accused person the right to a speedy trial. This criminal proceeding has been dragging on
for almost five years now. The accused have twice appealed to this court for redress from the wrong that they have suffered at the hands of the trial court. At least
one of them, namely Pedro Fernandez alias Piro, had been con-fined in prison from July 20, 1932 to November 27, 1934, for inability to post the required bond of
P3,000 which was finally reduced to P300. The Government should be the last to set an example of delay and oppression in the administration of justice and it is the
moral and legal obligation of this court to see that the criminal proceedings against the accused come to an end and that they be immediately dis-charged from the
custody of the law. (Conde vs. Rivera and Unson, 45 Phil., 651.)

In Kalaw vs. Apostol, supra, the petitioner invoked and this court applied and gave effect to the doctrines stated in the second Conde case, supra. In granting the writs
prayed for, this court, after referring to the constitutional and statutory provisions guaranteeing to persons accused of crime the right to a speedy trial, said:

Se infiere de los preceptos legales transcritos que todo acusado en causa criminal tiene derecho a ser juzgado pronta y publicamente. Juicio rapido significa un
juicioque se celebra de acuerdo con la ley de procedimiento criminal y los reglamentos, libre de dilaciones vejatorias, caprichosas y opersivas (Burnett vs. State, 76
Ark., 295; 88S. W., 956; 113 AMSR, 94; Stewart vs. State, 13 Ark., 720; Peo. vs. Shufelt, 61 Mich., 237; 28 N. W., 79; Nixon vs. State, 10 Miss., 497; 41 AMD., 601; State
vs. Cole, 4 Okl. Cr., 25; 109 P., 736; State vs. Caruthers, 1 Okl. Cr., 428; 98 P., 474; State vs. Keefe, 17 Wyo., 227, 98 p., 122;22 IRANS, 896; 17 Ann. Cas., 161). Segun los
hechos admitidos resulta que al recurrente se le concedio vista parcial del asunto, en el Juzgado de Primera Instancia de Samar, solo despues de haber transcurrido ya
mas de un ao y medio desde la presentacion de la primera querella y desde la recepcion de la causa en dicho Juzgado, y despues de haberse transferido dos veces la
vista delasunto sin su consentimiento. A esto debe aadirse que laprimera transferencia de vista era claramente injustificadaporque el motivo que se alego consistio
unicamente en laconveniencia personal del ofendido y su abogado, no habiendose probado suficientemente la alegacion del primero de quese hallaba enfermo. Es
cierto que el recurrente habia pedido que, en vez de sealarse a vista el asunto para el mayo de 1936, lo fuera para el noviembre del mismo ao; pero,aparte de que
la razon que alego era bastante fuerte porquesu abogado se oponia a comparecer por compromisos urgentes contraidos con anterioridad y en tal circunstancia
hubiera quedado indefenso si hubiese sido obligado a entraren juicio, aparece que la vista se pospuso por el Juzgado amotu proprio, por haber cancelado todo el
calendario judicial preparado por el Escribano para el mes de junio. Declaramos, con visto de estos hechos, que al recurrents se leprivo de su derecho fundamental de
ser juzgado prontamente.

Esguerra vs. De la Costa, supra, was a petition for mandamus to compel the respondent judge of the Court of First Instance of Rizal to dismiss the complaint filed in a
criminal case against the petitioner, to cancel the bond put up by the said petitioner and to declare the costs de oficio. In accepting the contention that the petitioner
had been denied speedy trial, this court said:

Consta que en menos de un ao el recurrente fue procesado criminalmente por el alegado delito de abusos deshonestos, en el Juzgado de Paz del Municipio de
Cainta, Rizal. Como consecuencia de las denuncias que contra el se presentaron fue arrestado tres veces y para gozar de libertad provisional, en espera de los juicios,
se vio obligado a prestartres fianzas por la suma de P1,000 cada una. Si no se da fin al proceso que ultimamente se ha incoado contra el recurrente la incertidumbre
continuara cerniendose sobre el y las consiguientes molestias y preocupaciones continuaran igualmente abrumandole. El Titulo III, articulo 1, No. 17,de la
Constitucion preceptua que en todo proceso criminalel acusado tiene derecho de ser juzgado pronta y publicamente. El Articulo 15, No. 7, de la Orden General No. 58
dispone asimismo que en las causas criminales el acusado tendra derecho a ser juzgado pronta y publicamente. Si el recurrente era realmente culpable del delito que
se le imputo, tenia de todos modos derechos a que fuera juzgado pronta y publicamente y sin dilaciones arbitrarias y vejatorias. Hemos declarado reiteradamente
que existe un remedio positivo para los casos en que se viola el derecho constitucional del acusado de ser juzgado prontamente. El acusado que esprivado de su
derecho fundomental de ser enjuiciado rapidamente tiene derecho a pedir que se le ponga en libertad, si estuviese detenido, o a que la causa que pende contra el
sea sobreseida definitivamente. (Conde contra Rivera y Unson, 45 Jur. Fil., 682; In the matter of Ford [1911], 160 Cal., 334; U. S. vs. Fox [1880], 3 Mont., 512; Kalaw
contra Apostol, R. G. No. 45591, Oct. 15, 1937; Pueblo contra Castaeda y Fernandez, 35 Gac. Of., 1357.)

We are again called upon to vindicate the fundamental right to a speedy trial. The facts of the present case may be at variance with those of the cases hereinabove
referred to. Nevertheless, we are of the opinion that, under the circumstances, we should consider the substance of the right instead of indulging in more or less
academic or undue factual differentiations. The petitioner herein has been arrested four times, has put up a bond in the sum of P4,000 and has engaged the services
of counsel to undertake his defense an equal number of times. The first arrest was made upon a complaint filed by one of the intervenors herein for alleged
falsification of a will which, sixteen months before, had been probated in court. This complaint, after investigation, was dismissed at the complainant's own request.
The second arrest was made upon a complaint charging the same offense and this complaint, too, was dismissed at the behest of the complainant herself who
alleged the quite startling ground that the petitioner was in poor health. The third arrest was made following the filing of an information by the provincial fiscal of
Pampanga, which information was dismissed, after due investigation, because of insufficiency of the evidence. The fourth arrest was made when the provincial fiscal
secured a reinvestigation of the case against the petitioner on the pretext that he had additional evidence to present, although such evidence does not appear to
have ever been presented.

It is true that the provincial fiscal did not intervene in the case until February 2, 1934, when he presented an information charging the petitioner, for the third time, of
the offense of falsification. This, however, does not matter. The prosecution of offenses is a matter of public interest and it is the duty of the government or those
acting in its behalf to prosecute all cases to their termination without oppressive, capricious and vexatious delay. The Constitution does not say that the right to a
speedy trial may be availed of only where the prosecution for crime is commenced and undertaken by the fiscal. It does not exclude from its operation cases
commenced by private individuals. Where once a person is prosecuted criminally, he is entitled to a speedy trial, irrespective of the nature of the offense or the
manner in which it is authorized to be commenced. In any event, even the actuations of the fiscal himself in this case is not entirely free from criticism. From October
27, 1932, when the first complaint was filed in the justice of the peace court of San Fernando, to February 2, 1934, when the provincial fiscal filed his information with
the justice of the peace of Mexico, one year, three months and six days transpired; and from April 27, 1933, when the second criminal complaint was dismissed by the
justice of the peace of Mexico, to February 2, 1934, nine months and six days elapsed. The investigation following the fourth arrest, made after the fiscal had secured
a reinvestigation of the case, appears also to have dragged on for about a year. There obviously has been a delay, and considering the antecedent facts and
circumstances within the knowledge of the fiscal, the delay may not at all be regarded as permissible. In Kalaw vs. Apostol, supra, we observed that the prosecuting
officer all prosecutions for public offenses (secs. 1681 and 2465 of the Rev. Adm. Code), and that it is his duty to see that criminal cases are heard without vexatious,
capricious and oppressive delays so that the courts of justice may dispose of them on the merits and determine whether the accused is guilty or not. This is as clear
an admonition as could be made. An accused person is entitled to a trial at the earliest opportunity. (Sutherland on the Constitution, p. 664; United States vs. Fox, 3
Mont., 512.) He cannot be oppressed by delaying he commencement of trial for an unreasonable length of time. If the proceedings pending trial are deferred, the
trial itself is necessarily delayed. It is not to be supposed, of course, that the Constitution intends to remove from the prosecution every reasonable opportunity to
prepare for trial. Impossibilities cannot be expected or extraordinary efforts required on the part of the prosecutor or the court. As stated by the Supreme Court of
the United States, "The right of a speedy trial is necessarily relative. It is consistent with delays and depends upon circumstances. It secures rights to a defendant. It
does not preclude the rights of public justice." (Beavers vs. Haubert [1905], 198 U. S., 86; 25 S. Ct., 573; 49 Law. ed., 950, 954.).

It may be true, as seems admitted by counsel for the intervenors, in paragraph 8, page 3 of his brief, that the delay was due to "the efforts towards reaching an
amicable extrajudicial compromise," but this fact, we think, casts doubt instead upon the motive which led the intervenors to bring criminal action against the
petitioner. The petitioner claims that the intention of the intervenors was to press upon settlement, with the continuous threat of criminal prosecution,
notwithstanding the probate of the will alleged to have been falsified. Argument of counsel for the petitioner in this regard is not without justification. Thus after the
filing of the second complaint with the justice of the peace court of Mexico, complainant herself, as we have seen, asked for dismissal of the complaint, on the ground
that "el acusado tenia la salud bastante delicada," and, apparently because of failure to arrive at any settlement, she decided to renew her complaint.

Counsel for the intervenors contend and the contention is sustained by the Court of Appeals that the petitioner did not complain heretofore of the denial of his
constitutional right to a speedy trial. This is a mistake. When the petitioner, for the fourth time, was ordered arrested by the Court of First Instance of Pampanga, he
moved for reconsideration of the order of arrest, alleging, among other things, "Que por estas continuas acusaciones e investigaciones, el acusado compareciente no
obstante su mal estado de salud desde el ao 1932 en que tuvo que ser operado por padecer de tuberculosis ha tenido que sostener litigios y ha sufrido la mar de
humiliaciones y zozobras y ha incudo en enormes gastos y molestias y ha desatendido su quebrantada salud." The foregoing allegation was inserted on page 6 of the
amended petition for certiorari presented to the Court of Appeals. The constitutional issue also appears to have been actually raised and considered in the Court of
Appeals. In the majority opinion of that court, it is stated:

Upon the foregoing facts, counsel for the petitioner submits for the consideration of this court the following questions of law: First, that the respondent court acted
arbitrarily and with abuse of its authority, with serious damage and prejudice to the rights and interests of the petitioner, in allowing that the latter be prosecuted
and arrested for the fourth time, and that he be subjected, also for the fourth time, to a preliminary investigation for the same offense, hereby converting the court
into an instrument of oppression and vengeance on the part of the alleged offended parties, Rosario Basa et al.; . . . .

And in the dissenting opinion, we find the following opening paragraph:

We cannot join in a decision declining to stop a prosecution that has dragged for about five years and caused the arrest on four different occasions of a law abiding
citizen for the alleged offense of falsifying a will that years be competent jurisdiction.

From the view we take of the instant case, the petitioner is entitled to have the criminal proceedings against him quashed. The judgment of the Court of Appeals is
hereby reversed, without pronouncement regarding costs. So ordered.
Republic of the Philippines
SUPREME COURT
Manila

EN BANC



G.R. No. L-28054 June 15, 1972

INTESTATE ESTATE OF ROSINA MARGUERITE WOLFSON, deceased, RICARDO VITO CRUZ, petitioner-appellee.

TESTATE ESTATE of ROSINA MARGUERITE WOLFSON, deceased, MANUEL Y. MACIAS, petitioner-appellant, ARTURO M. DEL ROSARIO, oppositor-appellee.

Quijano & Arroyo for petitioner-appellee.

Manuel Y. Macias in his own behalf.

Ross, Salcedo, Del Rosario, Bito, Misa & Lozada for oppositor-appellee.



MAKASIAR, J.:p

Rosina Marguerite Wolfson died on September 14, 1965 in San Francisco, California, U.S.A.

On January 10, 1966, Atty. Manuel Y. Macias, herein petitioner-appellant, unaware that Rosina died with a will and testament, filed in behalf of Ricardo Vito Cruz a
petition for the issuance of letters of administration in his favor over the estate in the Philippines of the late Rosina, which was docketed as Special Proceedings No.
63866, titled "Intestate Estate of Rosina Marguerite Wolfson, deceased," and was assigned to Branch VIII of the Manila Court of First Instance, then presided over by
then Judge, now Court of Appeals Justice, Manuel P. Barcelona.

Accordingly, Ricardo Vito Cruz was appointed Special Administrator for the estate of Rosina, qualified thereto, took his oath and assumed the duties thereof.

It turned out that Rosina left a will executed in accordance with the laws of the State of New York and through codicils executed in accordance with the laws of the
State of California, U.S.A., naming therein the Wells Fargo Bank as sole executor and the University of Michigan as the residuary beneficiary.

On September 24, 1965, said will and codicils of Rosina were presented for probate in the Superior Court of the State of California, U.S.A.

On October 11, 1965, the said will and codicils were duly probated by said California court (Annex 4-A, pp. 28-35, rec. on appeal).

In a document dated November 10, 1965, duly notarized and authenticated, the Wells Fargo Bank, as the sole executor designated in Rosina's will and codicils,
appointed lawyers James M. Ross, Ewald Selph, Rafael D. Salcedo, Arturo del Rosario, Jesus Bito, Joaquin L. Misa and Mariano Lozada, all of Manila, Philippines, as its
attorney-infact, with authority among others to file ancillary administration proceedings for the estate of Rosina and to act as administrator or administrators of the
estate (Annex "4-A", pp. 28-35, rec. on appeal).

Pursuant to his appointment as attorney-in-fact of executor Wells Fargo Bank, Atty. Arturo del Rosario filed on August 13, 1966, a petition in Special Proceedings No.
63866 before Branch VIII of the Manila Court of First Instance, praying that, inasmuch as the decedent left a will and codicils which were duly probated by the
Superior Court of California, U.S.A., the intestate proceedings in Special Proceedings No. 63866 be converted into a petition for the probate of Rosina's will and
codicils (Annex "4-C", pp. 38-42, rec. on appeal).

On October 25, 1966, petitioner-appellant Macias, in his own behalf and without informing his client Ricardo Vito Cruz, filed a similar but separate and independent
petition, which was docketed as Special Proceedings No. 67302 and assigned to Branch VI of the Manila Court of First Instance, then presided by Judge Gaudencio
Cloribel, alleging that he has a legal interest in Rosina's estate and praying for the probate of Rosina's will and codicils as well as for his (Macias') appointment as
special administrator (pp. 12-17, ROA).

Because of petitioner-appellant's claim that he has a legal interest in Rosina's estate, Judge Cloribel of Branch VI in an order dated October 27, 1966 set the hearing of
the petition on December 17, 1966 and appointed Macias special administrator (Annex "2", pp. 18-20, ROA). Accordingly, petitioner-appellant Macias was issued
letters of special administration on November 12, 1966 (Annex "3", pp. 21-22, ROA).

In a pleading dated December 9, 1966, Atty. Arturo del Rosario filed an opposition to the petition of petitioner-appellant for the probate of the will and codicils of
Rosina Marguerite Wolfson in Special Proceedings No. 67302 on the grounds, among others, that Rosina's estate is the subject of Special Proceedings No. 63866
before Branch VIII previously filed by petitioner-appellant Macias in behalf of respondent Ricardo Vito Cruz and before which he (Atty. Arturo del Rosario) filed on
August 13, 1966 a petition for the conversion of the said intestate proceedings into one for the probate of Rosina's will and codicils, which was then pending
resolution (Annexes "4", "4-A" & "4-B", pp. 23-27, ROA).

Likewise, special administrator Ricardo Vito Cruz filed a motion dated December 13, 1966 to dismiss the said petition of petitioner-appellant in Special Proceedings
No. 67302 on the grounds, among others:

(1) that Rosina's estate is already the subject of Special Proceedings No. 63866 before Branch VIII (invoking Section 1, Rule 73 of the Revised Rules of Court);

(2) that on August 13, 1966, Atty. Arturo del Rosario filed in behalf of Wells Fargo Bank a petition to convert the intestate proceeding in Special Proceedings No.
63866 into a testate proceeding for the probate of the last will and codicils of Rosina;

(3) that two proceedings over the same estate will only complicate matters and delay its closure; and

(4) that in filing his petition for the probate of the will and codicils of Rosina in Special Proceedings No. 67302 before Branch VI, petitioner-appellant concealed from
the presiding judge of Branch VI the fact that appellant previously filed Special Proceedings No. 63866 in his (Ricardo Vito Cruz) behalf as well as the fact that Arturo
del Rosario had already filed his pleading of August 13, 1966 for the conversion of the intestate proceedings into a testate one (Annex "6", pp. 48-59 ROA).

In a pleading dated December 16, 1966, petitioner-appellant filed his reply and opposition respectively to the opposition of Atty. Arturo del Rosario and the motion to
dismiss of Ricardo Vito Cruz, contending:

(1) that the grounds advanced by Atty. Arturo del Rosario and Ricardo Vito Cruz are not legal grounds for the dismissal of Special Proceedings No. 67302, because he
is seeking in this Special Proceedings No. 67302 his own appointment as regular, not ancillary, administrator of Rosina's estate, based simply on his interest in the
estate, without need of any authority from Wells Fargo Bank, which does not and cannot possibly have anything to do with these proceedings because its (WelIs
Fargo Bank) appointment by the California Superior Court as executor of Rosina's estate does not extend ex proprio vigore to the Philippines;

(2) that it is enough that a person has an interest in the will or in the property either as executor or otherwise to justify his intervention in the proceedings, citing
Section 1 of Rule 76 of the Revised Rules of Court and Santos vs. Castillo; 1 (3) that Special Proceedings No. 63866, which is an intestate proceeding merely for the
administration of Rosina's estate, is not a probate proceeding; and

(4) that to dismiss and/or consolidate Special Proceedings No. 67302 with Special Proceedings No. 63866, would frustrate the implementation of Rosina's will to
provide a suitable memorial in the City of Manila to her parents and to provide help and assistance to her former Filipino dependents and those of their parents
(Annex "7", pp. 70-85, ROA).

In an order dated December 17, 1966, Judge Cloribel of Branch VI postponed the hearing of Special Proceedings No. 67302 on the ground that the oppositors had
raised a prejudicial question to the effect that another case involving the very same matter is pending in Branch VIII presided over by Judge Barcelona (Annex "8", p.
86, ROA).

In a pleading dated December 21, 1966, petitioner-appellant filed in both Special Proceedings Nos. 63866 and 67302, a motion for the consolidation and joint hearing
of both cases as they relate to the same estate of Rosina (Annex "9", pp. 87-90, ROA).

In a pleading dated December 22, 1966 filed in both Special Proceedings Nos. 63866 and 67302, Severino Baron, Anselmo A. Reyes, Paulino Andrada, Alfredo V.
Walcott, Narciso S. Villanueva, Leonardo Baron, Godofredo L. Duao and Catalino S. Calimutan all mentioned in Julian's memorandum to Rosina endorsed
petitioner-appellant's petition for his appointment as regular administrator with the will annexed of Rosina's estate (Annex "10", pp. 91-95, ROA).

In a manifestation dated December 23, 1966, Ricardo Vito Cruz stated that he does not object to the transfer of Special Proceedings No. 67302 to Branch VIII, with
the qualification that he does not agree with the allegations in the rest of the prayer of petitioner-appellant in his urgent motion for consolidation of cases filed on
December 21, 1966 for the reasons he (Ricardo Vito Cruz) stated in his motion to dismiss filed on December 13, 1966 (Annex "11", pp. 94-95, ROA).

In an order dated December 23, 1966, Judge Cloribel of Branch VI ordered the transfer of Special Proceedings No. 67302 to Branch VIII if "the Presiding Judge therein
has no objection to said transfer" (Annex "12", p. 96, ROA).

On January 21, 1967, petitioner-appellant filed his opposition to the petition of Ricardo Vito Cruz for appointment as regular administrator of Rosina's estate in
Special Proceedings No. 63866, because, among others, of the alleged corrupt practices of Ricardo Vito Cruz with a view to his unjust enrichment at the expense of
the estate and his alleged wasteful administration of the same (Annex "13", pp. 97-105, ROA).

On January 12, 1967, petitioner-appellant filed an urgent motion for the resolution of his urgent motion for consolidation of the two cases and for their joint hearing
on January 14, 1967 and of his urgent ex parte motion for the probate of the will and codicils of Rosina (Annex "14", pp. 106-109, ROA).

Respondent Ricardo Vito Cruz, through counsel, filed an opposition thereto in a pleading dated January 12, 1967 claiming that he did not agree to the consolidation of
Special Proceedings No. 67302 nor to its joint hearing with Special Proceedings No. 63866, and praying that Special Proceedings No. 67302 be dismissed outright
(Annex "15", pp. 110-114, ROA).

In a manifestation dated January 17, 1967, respondent Ricardo Vito Cruz stated that the order of Judge Cloribel of Branch VI dated December 23, 1966 directing the
transfer of Special Proceedings No. 67302 to Branch VIII if the presiding judge therein has no objection to said transfer, is not an order for the consolidation of the
two cases and that on January 14, 1967 respondent Judge Manuel Barcelona of Branch VIII directed the transfer of Special Proceedings No. 67302 from Branch VI to
Branch VIII; and moved that, the two cases being the same, Special Proceedings No. 67302 should be dismissed, otherwise there will be duplicity even if it will be
heard jointly with Special Proceedings No. 63866 and will complicate matters and violate the prohibition against multiplicity of suits (Annex "15-A", pp. 115-118,
ROA).

On January 21, 1967, petitioner-appellant filed his opposition to the petition of Arturo M. del Rosario dated August 11, 1966 and filed on August 13, 1966 for his
appointment as ancillary administrator on the ground that he has no legal interest in the estate, invoking Section 2 of Rule 79 of the Rules of Court and the case of
Testate Estate of Rosalia Saquitan, Eulogio S. Eusebio vs. Domingo Valmores, Vicenta Siscar, oppositor-appellant 2 (Annex "16", pp. 119-126, ROA).

In an order dated February 11, 1967, pursuant to the agreement of the parties and the order dated December 23, 1966 in Special Proceedings No. 67302 for the
transfer of said special proceedings from Branch VI, Honorable Manuel Barcelona, then Presiding Judge of Branch VIII allowed its consolidation with Special
Proceedings No. 63866 in his Branch VIII (Annex "17", pp. 127-128, ROA).

On the same day, February 11, 1967, Arturo M. del Rosario and Ricardo Vito Cruz filed before Branch VIII a joint motion for an order authorizing the clerk of court to
receive the evidence relative to the probate of the will (Annex "18", pp. 129-130, ROA).


On February 13, 1967, respondent Ricardo Vito Cruz, thru counsel, filed a written manifestation before Branch VIII praying for the resolution of his motion to dismiss
Special Proceedings No. 67302 since it is now transferred to Branch VIII (Annex "18-A", p. 131, ROA).

In a well-reasoned order dated February 20, 1967, Honorable Manuel Barcelona, then Presiding Judge of Branch VIII, dismissed Special Proceedings No. 67302 (Annex
"19", pp. 132-142, ROA).

On March 8, 1967, petitioner-appellant filed a motion for the reconsideration of the aforesaid order of February 20, 1967, followed by an urgent motion dated March
2, 1967 for suspension of hearings until after resolution of his motion for reconsideration (Annexes "20" & "21", pp. 143-158, ROA).

On March 9, 1967, respondent Ricardo Vito Cruz filed his opposition to the motion for the reconsideration of the order of February 20, 1967 1 (Annex "22", pp. 159-
172, ROA), to which petitioner-appellant filed his reply dated March 15, 1967 (Annex "23", pp. 173-188, ROA).

On April 6, 1967, respondent Ricardo Vito Cruz filed a rejoinder to the reply (Annex "24", pp. 189-193, ROA), to which petitioner-appellant filed a surrejoinder dated
April 16, 1967 (Annex "25", pp. 194-203, ROA).

In an order dated April 22, 1967, the Court denied the motion for reconsideration of petitioner-appellant (Annex "26", pp. 204-208, ROA), who filed his notice of
appeal therefrom dated May 19, 1967 (Annex "27", pp. 209-210, ROA).

The appeal is devoid of merit.

Only last May 30, 1972, in Macias vs. Uy Kim, et al., 3 WE reiterated the rule that "Under Section 1 of Rule 73, Rules of Court, 'the court first taking cognizance of the
settlement of the estates of the deceased, shall exercise jurisdiction to the exclusion of all other courts.' Pursuant to this provision, therefore all questions concerning
the settlement of the estate of the deceased Rosina Marguerite Wolfson should be filed before Branch VIII of the Manila Court of First Instance, then presided over by
former Judge, now Justice of the Court of Appeals, Manuel Barcelona, where Special Proceedings No. 63866 for the settlement of the testate estate of the deceased
Rosina Marguerite Wolfson was filed and is still pending."

Paraphrasing the jurisprudence on this score, the salutary purpose of the rule is to prevent confusion and delay. It is not inserted in the law for the benefit of the
parties litigant but in the public interest for the better administration of justice, for which reason the parties have no control over it. 4 Consequently, every challenge
to the validity of the will, any objection to its authentication, every demand or claim by any heir, legatee or party in interest in intestate or testate succession must be
acted upon and decided within the same special proceedings, not in a separate action, and the same judge having jurisdiction in the administration of the estate
should take cognizance of the question raised, for he will be called upon to distribute or adjudicate the property to the interested parties. 5 WE stressed that the
main function of a probate court is to settle and liquidate the estates of the deceased either summarily or through the process of administration; and towards this
end the probate court has to determine who the heirs are and their respective shares in the net assets of the estate. 6 Section 1 of Rule 73, speaking as it does of
"settlement of the estates of the deceased," applies equally to both testate and intestate proceedings. And the conversion of an intestate proceedings into a testate
one is "entirely a matter of form and lies within the sound discretion of the court." 7 Special Proceedings No. 63866 was first instituted on January 10, 1966 by
petitioner-appellant himself as an intestate proceedings because he did not know then that Rosina Marguerite Wolfson died with a will and three codicils, in behalf of
Ricardo Vito Cruz praying for the issuance of letters of administration in favor of the said Ricardo Vito Cruz. Said proceedings was raffled to Brannch VIII of the Manila
Court of First Instance. By virtue of said petition, appellee Ricardo Vito Cruz was appoint special administrator and assumed the duties thereof after qualifying
therefor. On October 11, 1965, the will and dicils of the deceased Rosina were duly probated by to superior court of the State of California, U.S.A. (Annex "4-A", pp.
28-35, ROA). The Wells Fargo Bank, the sole executor designated in Rosina's will and codicils, appoint local lawyers James M. Ross, Ewald Selph, Rafael Salcedo,
Arturo del Rosario, Jesus Bito, Joaquin L. Misa and Mariano Lozada, as its attorney-in-fact, duly authorize among others, to file ancillary administration proceeding for
the estate of Rosina and to act as administrator or administrators of the estate (Annex "4-A", pp. 28-35, ROA). Pursuant to this appointment as such attorney-in-fact
of the executor Wells Fargo Bank, Atty. Arturo del Rosario instituted on August 13, 1966 a petition in Special Proceedings No. 63866, praying that the intestate
proceedings be converted into a petition for probate of Rosina's will and codicils (Annex "4-C", pp. 38-42, ROA). On October 25, 1966, petitioner-appellant Macias, in
his own behalf and without advising his former client Ricardo Vito Cruz, filed an independent petition for the probate of Rosina's will and codicils, which was docketed
as Special Proceedings No. 67302 and assigned to Branch VI of the Manila Court of First Instance. Claiming that he has a legal interest in Rosina's estate, he also
prayed for his appointment as special administrator (pp. 12- 17, ROA).

It is thus patent that the second petition filed on October 25, 1966 by petitioner-appellant was about nine (9) months subsequent to the first petition he filed in
behalf of appellee Ricardo Vito Cruz, now docketed as Special Proceedings No. 63866 and over two months after Arthur del Rosario filed on August 13, 1966 his
petition to convert Special Proceedings No. 63866 from intestate to testate.

As above recounted, petitioner-appellant, on January 21, 1967, filed his opposition to the petition of Atty. Arturo del Rosario for his appointment as ancillary
administrator in Special Proceedings No. 63866 of Rosina's estate, on the ground that del Rosario lacks legal interest, while he, petitioner-appellant, has legal interest
and represents the bigger interest in Rosina's estate and therefore should be appointed regular administrator (Annex "16", pp. 119-126, ROA).

Hence, the appealed orders dated February 20 and April 22, 1967 (Annexes "19" & "26", pp. 132-142, ROA) of the Presiding Judge of Branch VIII dismissing Special
Proceedings No. 67302 after the same was transferred from Branch VI to Branch VIII and consolidated with Special Proceedings No. 63866 pursuant to the order date
February 11, 1967 (Annex "17", pp. 127-128, ROA), should be sustained.

Petitioner-appellant insists that after ordering its consolidation with Special Proceedings No. 63866, the Presiding Judge of Branch VIII has no authority to dismiss
Special Proceedings No. 67302 and should hear jointly said Special Proceedings No. 67302 and Special Proceedings No. 63866.

Generally, consolidation and joint hearing of the two cases would have been proper if they do not involve settlement of the estate of a decedent, which is covered by
a special provision of the Rules of Court, namely Section 1 of Rule 73, the specific command of which should be obeyed. At any rate, motions for consolidation are
addressed to the sound discretion of the court; and WE do not find that the trial judge gravely abused his discretion in reconsidering the prior order for the
consolidation of the two special proceedings for the settlement of the same estate and dismissing Special Proceedings No. 67302, to warrant the excercise of Our
supervisory authority over the lower court which has wide discretion in this regard. 8 As a matter of fact, the Honorable Presiding Judge of Branch VIII exercised
sound discretion in directing the dismissal of Special Proceedings No. 67302. The trial court is not precluded from dismissing one case after ordering the consolidation
and joint hearing of the two cases; because under Section 1 of Rule 31, after ordering consolidation, the court "may make such orders concerning proceedings therein
as may tend to avoid unnecessary costs or delay."

As stated by the trial court in its order dated April 22, 1967

... Moreover, even if there was a consolidation, this Court, perforce, had to dismiss the instant proceedings, otherwise there will be multiplicity of suits.

... It needs no elaboration that a court can never be deprived of its power to dismiss a case pending before it if the subject-matter of such case is the same as the
subject-matter of another case also pending before it. As this Court had pointed out in the order sought to be reconsidered, the filing of Special Proceedings No.
67302 violated that basic and elementary rule on multiplicity of suits which must be avoided. There is no difference between the two proceedings. Both refer to the
Philippine estate of the late Rosina Marguerite Wolfson of which a special administrator has been appointed to temporarily administer this estate pending the
appointment of a regular administrator. Although Special Proceedings No. 63866 is entitled "Re Intestate Estate of Rosina Marguerite Wolfson, etc.", the fact remains
that the hearing on the allowance of the last will and codicils left by the decedent was already terminated in said proceedings and, in fact, petitioner Macias
participated therein. If such will and codicils are allowed to probate, all that would be needed is to change the title of said proceedings from Intestate to Testate.
Special Proceedings No. 67302 (the instant proceedings), therefore, is of no moment and would serve no purpose.

The record that in Special Proceedings No. 63866, petitioner Macias not only participated but presented evidence supporting the admission to probate the decedent's
will and codicils thereto. In the same Special Proceedings No. 63866, Macias also participated by opposing the appointment of Ricardo Vito Cruz as regular
administrator and proposing his own appointment to the position. Viewed in this light, the Court sees no reason to proceed with Special Proceedings No. 67302. If
petitioner Macias wants to be the administrator, he can do so by applying, as in fact he has, in Special Proceedings No. 63866. (Annex "26," pp. 204, 205-207, ROA).

As hinted in the aforequoted portion of the appealed order of April 22, 1967, if petitioner-appellant wants to be appointed as regular administrator, he can file his
petition therefor, as he in fact did, in Special Proceedings No. 63866.

Even if it were true that appellee Vito Cruz intends to suppress, as claimed by petitioner-appellant, Julian's memorandum to his sister Rosina wherein Julian expressed
the hope that Rosina will deliver at her convenience to petitioner-appellant the sum of P500.00 minus taxes, such a design cannot prevent him from presenting the
said memorandum in Special Proceedings No. 63866, a copy of which he apparently possesses because he attached the same as Appendix "A" to his brief (p. 225,
appellant's brief).

As a matter of fact, Julian's memorandum was presented in Special Proceedings No. 57405 over his estate by Severino Baron and Faustino Reis also named therein as
favored recipients of the bounty of Julian (see order of Judge Conrado Vasquez in Special Proceedings No. 57405 on pp. 25-27 of appellant's brief in G.R. No. L-28947).

Appellant himself already presented said memorandum as his Exhibit B-Macias in said Special Proceedings No. 57405 over Julian's estate, which is the subject of his
appeal in G.R. No. L-28947.

It would seem presumptuous on the part of petitioner-appellant that Rosina's desire as expressed in her will to provide a suitable memorial in the City of Manila to
her parents and to provide help and assistance to her former Filipino dependents and those of their parents, could not be adequately implemented by the probate
court.

It should be noted from his said memorandum that the late Julian Wolfson expressly hoped that his sister will deliver to petitioner-appellant at her convenience only
the comparatively meager sum of P500.00, the lowest accorded to his former employees by the late Julian A. Wolfson, who expressed the same hope that Rosina at
her convenience will deliver to three of his employees P10,000 each and to two remaining employees Pl,000.00 each, less taxes. It is likewise significant that the
deceased Julian Wolfson preferred to leave his last instructions to Ricardo Vito Cruz, his accountant, and not to petitioner-appellant, an experienced lawyer of long
standing and Julian's former assistant for several years in his law firm. Considering these circumstances, the unusual interest on the part of petitioner-appellant in
insisting in filing a separate probate proceeding and in seeking his own appointment as administrator of Rosina's estate is rather curious, to say the least.

A respectable and self-respecting member of the Bar would consider indelicate such an act and would restrain his hand in being too officious under the circumstance.
He would not institute several actions to protect his interest, when one suit will suffice, thus minimizing the clogging of the dockets of the courts.

Petitioner-appellant's pretension that he was not aware of the petition filed on August 13, 1966 by Atty. Arturo del Rosario for the probate of the will and codicils of
Rosina in the same Special Proceedings No. 63866, even if true, does not justify his initiating another proceeding for the same purpose, separate from and
independent of Special Proceedings No. 63866. In this connection, his pretense cannot be believed; because he was the one who filed on January 10, 1966, Special
Proceedings No. 63866 in behalf of appellee Ricardo Vito Cruz whose appointment as official administrator he secured and whose appointment as regular
administrator he prayed for. Upon knowing of the existence of Rosina's will and codicils, petitioner-appellee would reasonably be expected to examine and study the
records of Special Proceedings No. 63866 long before filed on October 25, 1966 his separate petition for probate of the same will and codicils assigned to Branch VI,
and consequently he must have necessarily discovered the existence in the record of Special Proceedings No. 63866 the petition for probate filed by Atty. Arturo del
Rosario. Because such a petition would nullify his obvious desire to have a hand in the administration of Rosina's estate through his former client, appellee Ricardo
Vito Cruz, feigning ignorance of the petition of Atty. Arturo del Rosario, he filed a separate independent petition for probate which is docketed as Special Proceedings
No. 67302 hoping thereby to preserve the chances of his claim being recognized by the court and of being appointed regular administrator, instead of filing the same
petition in special Proceedings No. 63866 for the conversion of the said proceedings from intestate to testate. This act on his part reveals a motive that is hardly
flattering to him as a member of the bar and as an officer of the court.

WHEREFORE, the appealed orders dated February 20, 1967 and April 22, 1967 are hereby affirmed and the appeal is hereby dismissed, with costs against petitioner-
appellant.



Republic of the Philippines
SUPREME COURT
Manila

FIRST DIVISION

G.R. No. L-32328 September 30, 1977

TESTATE ESTATE OF THE LATE ADRIANO MALOTO: ALDINA MALOTO CASIANO, CONSTANCIO MALOTO, PURIFICACION MIRAFLOR, ROMAN CATHOLIC CHURCH OF
MOLO, and ASILO DE MOLO, petitioners-appellants
vs.
FELINO MALOTO and FELINO MALOTO, oppositors-appellees.

Ramon C. Zamora, Lorenzo E. Coloso, Jose L. Castigador, Arthur Defensor & Sixto Demaisip and Flores, Macapagal, Ocampo & Balbastro for petitioners-appellants.

Nacianceno G. Rico & Felipe G. Espinosa for oppositors-appellees.



FERNANDEZ, J.:

This is a petition to review the order dated April 13, 1970 of the Court of First Instance of Iloilo, Branch III, in Special Proceeding No. 2176 dismissing the petition for
the probate of a will. 1

One Adriana Maloto died on October 20, 1963 in Iloilo City, her place of residence.

Aldina Maloto Casiano, Constancio Maloto, Panfilo Maloto, and Felino Maloto, niece and nephews, respectively, of Adriana Maloto, in the belief that decedent died
intestate, commenced on November 4, 1963 in the Court of First Instance of iloilo an intestate proceeding docketed as Special Proceeding No. 1736. In the course of
said intestate proceeding, Aldina Maloto Casiano, Constancio Maloto, Panfilo Maloto and Felino Maloto executed an extrajudicial Partition of the estate of Adriana
Maloto on February 1, 1964 whereby they adjudicated said estate unto themselves in the proportion of one-fourth (1/4) share for each. 2 The Court of First Instance
of iloilo, then prescribed by Judge Emigdio V. Nietes, ed he diamond partition on March approve extrajudicial on March 21, 1964. 3

On April 1, 1967, a document dated January 3, 1940 purporting to be the last with and testament of Adriana Maloto was delivered to the Clerk of Art of the Art of
First Instant of Iloilo. 4 It appears that Aldina Maloto Casiano Consent Maloto, Panfilo Maloto, and Felino Maloto are named as heirs but Maloto Casiano and
Constancio Maloto allegedly have shares in said with which are bigger, different and more valuable than what they obtained in the extrajudicial partition. The said will
also allegedly made dispositions to certain devisees and/or legatees, among whom being the Asilo de Molo, the Roman Catholic Church of Molo, and Purificacion
Miraflor.

On May 24, 1967, Aldina Maloto Casiano and Constancio Maloto filed in Special Proceeding No. 1736 a motion (1) for reconsideration; (2) annulment of the
proceedings; and (3) for the allowance of the last will and testament of Adriana Maloto. 5 The Asilo de Molo, the Roman Catholic Church of Molo, and Purificacion
Miraflor also filed in Special Proceeding No. 1736 petitions for the allowance of the will of Adriana Maloto. 6

Panfilo Maloto and Felino Maloto opposed the motion of Aldina Maloto Casiano and Constancio Maloto.

The Court of First Instance of iloilo, through Judge Emigdio V. Nietes, issued an order dated November 16, 1968 denying the motion to reopen the proceedings on the
ground that the said motion had been filed out of time. A motion for reconsideration of said order was denied. Petitioners appealed from the order of denial. On
motion of Panfilo Maloto and Felino Maloto, the lower court dismissed the appeal on the ground that it was filed late. A motion for reconsideration of the order of
dismissal was denied. A supplemental order dated April 1, 1969 stating as additional ground that the appeal is improper was issued.

The petitioners filed a petition for certiorari and mandamus with the Supreme Court docketed as G.R. No. L-30479. This Court dismissed the petition in a resolution
dated May 14, 1969 which reads:

L-010479 (Constancio Maloto, et al, vs. Hon. Emigdio V. Nietes, etc., et al.) THE COURT RESOLVED to dismiss the petition for certiorari and mandamus, without
passing on the issue of whether or not the petitioners appeal from the order of November 16, 1968 of respondent Judge was made on time, it appearing that the
more appropriate remedy of petitioners in the premises stated in the petition is for petitioners to initiate a separate proceeding for the probate of the alleged will in
question. 7

Acting on the petitioners' motion for reconsideration and citation, fl Art issued a resolution dated July 15, 1969 which reads:

Acting on the motion for reconsideration and/or clarification filed by petitioner in G. R. No. L-30479, Constancio Maloto, et al., vs. Hon. Emigdio V. Nietes, etc. et al.,
dated June 11, 1969, the Court resolved to DENY the motion for reconsideration, with the clarification that the matter of whether or not the pertinent findings of
facts of respondent Judge in his herein subject order of November 16, 1968 constitute res adjudicata may be raised in the proceedings for probate of the alleged will
in question indicated in the resolution of this Court of May 14, 1969, wherein such matter will be more appropriately determined. 8

Thereupon, the herein petitioners commenced Special Proceeding No. 2176 in the Court of First Instance of Iloilo for the probate of the alleged last will and
testament of Adriana Maloto. 9

Panfilo Maloto and Felino Maloto filed an opposition with a motion to dismiss on the following grounds:

I. THAT THE ALLEGED WILL SOUGHT TO BE PROBATED HAD BEEN DESTROYED AND REVOKED BY THE TESTATRIX.

II. THAT THE INSTANT PETITION FOR PROBATE IS NOW BARRED BY PRIOR JUDGMENT OR ORDER (OR RES JUDICATA).

III. THAT THE ESTATE OF THE LATE ADRIANA MALOTO HAD ALREADY PASSED OUT OF EXISTENCE AND TITLE THERETO HAD ALREADY ARRESTED IN THE
DISTRIBUTEES OF THEIR ASSIGNS.

IV. THAT PETITIONERS ALDINA MALOTO CASIANO AND CONSTANCIO MALOTO ARE NOW ESTOPPED FROM SEEKING THE REMEDY TENDER THIS PROCEEDING,
THEY HAVING CEASED TO BE INTERESTED PARTIES. 10

In an order dated April 13, 1970, the probate court dismissed the petition for the probate of the with on the basis of the finding of said court in Special Proceeding
No. 1736 that the alleged win sought to be Probated had been destroyed and revoked by the testatrix. The probate court sustained the oppositors' contention that
the petition for probate is now barred by the order of November 16, 1968 in the intestate estate proceeding, Special Proceeding No. 1736. 11

The herein petitioners allege that the probate court committed the following errors:

I

THE LOWER COURT ERRED IN HOLDING THAT THE .kl).NIITTEI)I,Y GENUINE LAST WILL AND TESTAMENT OF THE LATE ADRIANA MALOTO (THE SUBJECT OF PETITION
FOR PROBATE SPECIAL PROCEEDING NO. 2176, CFI ILOILO) HAD PREVIOUSLY BEEN REVOKED BY HER (ADRIANA MALOTO).

II

THE LOWER COURT ERRED IN HOLDING THAT SAID PETITION (FOR PROBATE OF THE AFORESAID LAST WILL AND TESTAMENT OF THE LATE ADRIANA MALOTO) IS NOW
BARRED BY PRIOR JUDGMENT. I. E., THAT THE MATTER CONCERNED IS NOW RES ADJUDICATA

III

THE LOWER COURT, THEREFORE, ERRED IN DISMISSING THE AFORESAID PETITION FOR PROBATE OF THE LAST WILL AND TESTAMENT OF THE LATE ADRIANA MALOTO
AND IN NOT, INSTEAD, GIVING IT (THE PETITION ABOVE-CITED DUE COURSE.12

The instant petition for review is meritorious.
The probate court had no jurisdiction to entertain the petition for the probate of the alleged with of Adriana Maloto in Special Proceeding No. 1736. Indeed, the
motion to reopen the was denied because the same was filed out of time. Moreover, it is not proper to make a finding in an intestate estate proceeding that the
discovered will has been revoked. As a matter of fact, the probate court in Special Proceeding No. 1736 stated in the order of November 16, 1968 that "Movants
should have filed a separate action for the probate of the Will." 13 And this court stated in its resolution of May 14, 1969 that "The more appropriate remedy of the
petitioners in the premises stated in the petition is for petitioners to initiate a separate proceeding for the probate of the alleged with in question."
In view of the foregoing, the order of November 16, 1968 in Special Proceeding No. 1736 is not a bar to the present petition for the probate of the alleged will of
Adriana Maloto. WHEREFORE, the order dated April 13, 1970 dismissing the petition for the probate of the alleged will of Adriana Maloto is hereby set aside and the
lower court is directed to proceed with the hearing of the petition in Special Proceeding No. 2176 on the merits, with costs against the respondents. SO ORDERED.

Вам также может понравиться